nat res digested

173
CASES IN NATURAL RESOURCES SUBMITTED TO: Atty . Ismael Manaligod  SUBMITTED BY : Sophia E. Matote  Jonna Maye S. Canindo Maureen Margareth D. Eslava Sony Berth Daluping Eric Gonayon  CSU Ll.B- II

Upload: ellen-glae-daquipil

Post on 02-Jun-2018

228 views

Category:

Documents


0 download

TRANSCRIPT

Page 1: Nat Res Digested

8/10/2019 Nat Res Digested

http://slidepdf.com/reader/full/nat-res-digested 1/173

CASES

INNATURAL RESOURCES

SUBMITTED TO: Atty. Ismael Manaligod

SUBMITTED BY:

Sophia E. Matote Jonna Maye S. CanindoMaureen Margareth D. EslavaSony Berth DalupingEric Gonayon

CSU Ll.B- II

Page 2: Nat Res Digested

8/10/2019 Nat Res Digested

http://slidepdf.com/reader/full/nat-res-digested 2/173

G.R. No. 79538. Octobe !8" !99#

$ELI%E YSMAEL" &R. ' CO." INC." petitioner "

vs .T(E DE%UTY E)ECUTI*E SECRETARY" T(E SECRETARY O$EN*IRONMENT AND NATURAL RESOURCES" T(E DIRECTOR O$ T(EBUREAU O$ $OREST DE*ELO%MENT +, T IN %EA/S DE*ELO%MENTAND REALTY COR%ORATION"respondents .

FAC S!

"n "cto#er $%& $'()& petitioner entered into a tim#er license agreement*ith the Department o+ Agriculture and ,atural -esources& represented #ythen Secretary Jose Feliciano& *herein it *as issued an e clusive license tocut& collect and remove tim#er e cept prohi#ited species *ithin a speci/edportion o+ pu#lic +orest land *ith an area o+ )0&'%1 hectares located in themunicipality o+ Maddela& province o+ ,ueva 2i3caya +rom "cto#er $%& $'()until June 41& $''1.

5o*ever& on August $6& $'64& the Director o+ the Bureau o+ ForestDevelopment 7Bureau8& Director Edmundo Cortes& issued a memorandumorder stopping all logging operations in ,ueva 2i3caya and 9uirino provinces&and cancelling the logging concession o+ petitioner and nine other +orest

concessionaires& pursuant to presidential instructions and a memorandumorder o+ the Minister o+ ,atural -esources eodoro :ena.

Su#se;uently& petitioner<s tim#er license agreement *as cancelled. 5e senta letter addressed to then :resident Ferdinand Marcos *hich soughtreconsideration o+ the Bureau=s directive& citing in support thereo+ itscontri#utions to +orest conservation and alleging that it *as not given theopportunity to #e heard prior to the cancellation o+ its logging operations& #utno +avora#le action *as ta>en on his letter?

Barely one year therea+ter& appro imately one@hal+ o+ the area +ormerlycovered #y petitioner<s A *as re@a*arded to *in :ea>s Development and-ealty Corporation under a ne* A *hich *as set to e pire on July 4$& %11'&*hile the other hal+ *as allo*ed to #e logged #y Filipinas oggers& Inc.*ithout the #ene/t o+ a +ormal a*ard or license. he latter entities *erecontrolled or o*ned #y relatives or cronies o+ deposed :resident FerdinandMarcos.

Page 3: Nat Res Digested

8/10/2019 Nat Res Digested

http://slidepdf.com/reader/full/nat-res-digested 3/173

Soon a+ter the change o+ government in Fe#ruary $'6(& petitioner sent aletter dated March $ & $'6( to the " ce o+ the :resident& and another letterdated April %& $'6( to Minister Ernesto Maceda o+ the Ministry o+ ,atural-esources M,- & see>ing! 7$8 the reinstatement o+ its tim#er licenseagreement *hich *as cancelled in August $'64 during the Marcos

administration? 7%8 the revocation o+ A ,o. 4)( *hich *as issued to *in:ea>s Development and -ealty Corporation *ithout pu#lic #idding and inviolation o+ +orestry la*s& rules and regulations? and& 748 the issuance o+ anorder allo*ing petitioner to ta>e possession o+ all logs +ound in theconcession area. 5o*ever& petitioner=s re;uest *as denied. :etitionermoved +or reconsideration reiterating& among others& its re;uest that thetim#er license agreement issued to private respondent #e declared null andvoid. he M,- ho*ever denied this motion. :etitioner su#se;uentlyappealed +rom the orders o+ the M,- to the " ce o+ the :resident. he" ce o+ the :resident& acting through then Deputy E ecutive SecretaryCatalino Macaraig& denied petitioner=s appeal +or lac> o+ merit. :etitioner /led

*ith the Court a petition +or certiorari, *ith prayer +or the issuance o+ arestraining order or *rit o+ preliminary in unction&

ISS E! Hhether or not petitioner has the right to see> the nulli/cation o+ the Bureau orders cancelling his tim#er license agreement and the grantingo+ A to private respondent& *hich *ere issued *ay #ac> in $'64 and $'60&respectively.

5E D!

,". he +ailure o+ petitioner to /le the petition +or certiorari *ithin a

reasona#le period o+ time renders the petitioner suscepti#le to the adverselegal conse;uences o+ laches. aches is de/ned as the +ailure or neglect +oran unreasona#le and une plained length o+ time to do that *hich #ye ercising due diligence& could or should have #een done earlier& or to asserta right *ithin a reasona#le time& *arranting a presumption that the partyentitled thereto has either a#andoned it o+ declined to assert it. he rule isthat unreasona#le delay on the part o+ a plainti in see>ing to en+orce analleged right may& depending upon the circumstances& #e destructive o+ theright itsel+. 2erily& the la*s did these *ho are vigilant& not those *ho sleepupon their rights. In the case at #ar& petitioner *aited +or at least threeyears #e+ore it /nally /led a petition +or certiorari *ith the Court attac>ingthe validity o+ the assailed Bureau actions in $'64 and $'60. Consideringthat petitioner& throughout the period o+ its inaction& *as not deprived o+ theopportunity to see> relie+ +rom the courts *hich *ere normally operating atthe time& its delay constitutes unreasona#le and ine cusa#le neglect&tantamount to laches. Accordingly& the *rit o+ certiorari re;uiring thereversal o+ these orders *ill not lie. here is a more signi/cant +actor *hich#ars the issuance o+ a *rit o+ certiorari in +avor o+ petitioner and againstpu#lic respondents herein. A long line o+ cases esta#lish the #asic rule that

Page 4: Nat Res Digested

8/10/2019 Nat Res Digested

http://slidepdf.com/reader/full/nat-res-digested 4/173

the courts *ill not inter+ere in matters *hich are addressed to the sounddiscretion o+ government agencies entrusted *ith the regulation o+ activitiescoming under the special technical >no*ledge and training o+ such agencies.More so *here& as in the present case& the interests o+ a private loggingcompany are pitted against that o+ the pu#lic at large on the pressing pu#lic

policy issue o+ +orest conservation. For this Court recogni3es the *idelatitude o+ discretion possessed #y the government in determining theappropriate actions to #e ta>en to preserve and manage natural resources&and the proper parties *ho should en oy the privilege o+ utili3ing theseresources. im#er licenses& permits and license agreements are the principalinstruments #y *hich the State regulates the utili3ation and disposition o+ +orest resources to the end that pu#lic *el+are is promoted. And it can hardly#e gainsaid that they merely evidence a privilege granted #y the State to;uali/ed entities& and do not vest in the latter a permanent or irrevoca#leright to the particular concession area and the +orest products therein. heymay #e validly amended& modi/ed& replaced or rescinded #y the Chie+

E ecutive *hen national interests so re;uire. hus& they are not deemedcontracts *ithin the purvie* o+ the due process o+ la* clause.

Page 5: Nat Res Digested

8/10/2019 Nat Res Digested

http://slidepdf.com/reader/full/nat-res-digested 5/173

BE,G E C"-:"-A I",& G.-. ,o. $(4$1$:etitioner&

@ versus @

DE:A- ME, "F E,2I-",ME, A,D ,A -A -ES" -CES@MI,ESADJ DICA I", B"A-D and J.G. -EA A,D MI,I,G C"-:"-A I",&-espondents.

FAC S!

Benguet and J.G. -ealty entered into a -oyalty Agreement *ith "ption to:urchase 7-AH":8 & *herein J.G. -ealty *as ac>no*ledged as the o*ner o+ +our mining claims *ith a total area o+ %66.6()( hectares. he parties alsoe ecuted a Supplemental Agreement. he mining claims *ere covered #y

Mineral :roduction Sharing Agreement 7M:SA8 Application ,o. A:SA@2@111' ointly /led #y J.G. -ealty as claim@o*ner and Benguet as operator.

A+ter some time& the E ecutive 2ice@:resident o+ Benguet& Antonio ,. achuling& issued a letter in+orming J.G. -ealty o+ its intention to develop themining claims. 5o*ever& J.G. -ealty& through its :resident& Johnny . an&then sent a letter to the :resident o+ Benguet in+orming the latter that it *asterminating the -AH":. he latter alleged that petitioner violated some o+ the provisions o+ the -AH":& speci/cally on non@payment o+ royalties andnon@+ul/llment o+ o#ligations stipulated therein.

J.G. -ealty /led a :etition +or Declaration o+ ,ullityKCancellation o+ the-AH":. :"A issued a Decision& cancelling the -AH": and its SupplementalAgreement. BE,G E *as su#se;uently e cluded +rom the oint M:SAApplication over the mineral claims. Su#se;uent M- *as denied. Saiddecision *as upheld #y DE,-@MAB.

5ence this instant petition.

ISS E!

Hhether or not petitioner the /ling o+ the petition *ith the SupremeCourt is proper.

5E D!

Page 6: Nat Res Digested

8/10/2019 Nat Res Digested

http://slidepdf.com/reader/full/nat-res-digested 6/173

,". the instant petition can #e denied outright as Benguet resorted toan improper -emedy.

he last paragraph o+ Section ' o+ -epu#lic Act ,o. 7-A8 '0% or the

L:hilippine Mining Act o+ $'') states& LA petition +or revie* #y certiorari and;uestion o+ la* may #e /led #y the aggrieved party *ith the Supreme Court*ithin thirty 7418 days +rom receipt o+ the order or decision o+ the MAB .

he -evised -ules o+ Civil :rocedure included -ule 04 to provide auni+orm rule on appeals +rom ;uasi@ udicial agencies. nder the rule& appeals+rom their udgments and /nal orders are no* re;uired to #e #rought to theCA on a veri/ed petition +or revie*. A ;uasi@ udicial agency or #ody has #eende/ned as an organ o+ government& other than a court or legislature& *hicha ects the rights o+ private parties through either ad udication or rule@ma>ing. MAB +alls under this de/nition? hence& it is no di erent +rom the

other ;uasi@ udicial #odies enumerated under -ule 04. Besides& theintroductory *ords in Section $ o+ Circular ,o. $@'$NNLamong these agenciesare NNindicate that the enumeration is not e clusive or conclusive andac>no*ledge the e istence o+ other ;uasi@ udicial agencies *hich& though note pressly listed& should #e deemed included therein.

T he udicial policy o+ o#serving the hierarchy o+ courts dictates that directresort +rom administrative agencies to this Court *ill not #e entertained&unless the redress desired cannot #e o#tained +rom the appropriate lo*ertri#unals& or unless e ceptional and compelling circumstances usti+yavailment o+ a remedy +alling *ithin and calling +or the e ercise o+ our

primary urisdiction. hus Benguet should have /led the appeal *ith the CA.

:etitioner having +ailed to properly appeal to the CA under -ule 04& thedecision o+ the MAB has #ecome /nal and e ecutory. "n this ground alone&the instant petition must #e denied.

Page 7: Nat Res Digested

8/10/2019 Nat Res Digested

http://slidepdf.com/reader/full/nat-res-digested 7/173

G.R. No. 148267. August 8, 2002

ARMANDO C. CARPIO, petitioner, vs. SULU RESOURCES DEVELOPMENTCORPORATION, respondent.

FACTS:

This case originated from a petition filed by respondent [Sulu Resources DevelopmentCorporation] for Mines Production Sharing Agreement MPSA! "o# MPSA$%&$'(') coveringcertain areas in Antipolo) Ri*al# Petitioner [Armando C# Carpio] filed an opposition+adverseclaim thereto) alleging) inter alia, that his landholdings in Cupang and Antipolo) Ri*al ,ill becovered by respondent-s claim) thus he en.oys a preferential right to e/plore and e/tract the0uarry resources on his properties#

After due proceedings ,ere held) the Panel of Arbitrators of the Mines and 1eo$Sciences 2ureauof the D3"R rendered a Resolution upholding petitioner-s opposition+adverse claim#Respondent appealed the foregoing Resolution to the Mines Ad.udication 2oard# Mean,hile)

petitioner filed a motion to dismiss appeal on the ground of respondent-s failure to comply ,iththe re0uirements of the "e, Mining Act-s %mplementing Rules and Regulations# The MinesAd.udication 2oard rendered the assailed 4rder dismissing petitioner-s opposition+adverse claim#Petitioner filed a motion for reconsideration of said 4rder ,hich ,as denied by the 2oard# Anappeal ,as filed ,ith the CA but same ,as denied#

%SS536

7hether or not appeals from the Decision or 8inal 4rders of the Mines Ad.udication 2oardshould be made directly to the Supreme Court as contended by the respondent and the Court of Appeals) or such appeals be first made to the Court of Appeals as contended by herein petitioner#

93:D6

The petition is meritorious#

8actual controversies are usually involved in administrative actions; and the CA is preparedto handle such issues because) unli<e this Court) it is mandated to rule on 0uestions of fact# i %n

Metro Construction, ,e observed that not only did the CA have appellate .urisdiction over C%AC

Page 8: Nat Res Digested

8/10/2019 Nat Res Digested

http://slidepdf.com/reader/full/nat-res-digested 8/173

decisions and orders) but the revie, of such decisions included 0uestions of fact and la,# ii At thevery least ,hen factual findings of the MA2 are challenged or alleged to have been made ingrave abuse of discretion as in the present case) the CA may revie, them) consistent ,ith theconstitutional duty iii of the .udiciary#

To summari*e) there are sufficient legal footings authori*ing a revie, of the MA2 Decisionunder Rule =( of the Rules of Court# First, Section (> of Article &% of the '?@ Constitution)mandates that B[n]o la, shall be passed increasing the appellate .urisdiction of the SupremeCourt as provided in this Constitution ,ithout its advice and consent# 4n the other hand)Section ? of RA "o# ?= provides that decisions of the MA2 may be revie,ed by this Courton a Bpetition for revie, by certiorari# This provision is obviously an e/pansion of the Court-sappellate .urisdiction) an e/pansion to ,hich this Court has not consented# %ndiscriminateenactment of legislation enlarging the appellate .urisdiction of this Court ,ould unnecessarily

burden it# iv

Second, ,hen the Supreme Court) in the e/ercise of its rule$ma<ing po,er) transfers to theCA pending cases involving a revie, of a 0uasi$.udicial body-s decisions) such transfer relates

only to procedure; hence) it does not impair the substantive and vested rights of the parties# Theaggrieved party-s right to appeal is preserved; ,hat is changed is only the procedure by ,hichthe appeal is to be made or decided# v The parties still have a remedy and a competent tribunal togrant this remedy#

Third, the Revised Rules of Civil Procedure included Rule =( to provide a uniform rule onappeals from 0uasi$.udicial agencies# vi 5nder the rule) appeals from their .udgments and finalorders are no, re0uired to be brought to the CA on a verified petition for revie,# vii A 0uasi$

.udicial agency or body has been defined as an organ of government) other than a court or legislature) ,hich affects the rights of private parties through either ad.udication or rule$ma<ing#viii MA2 falls under this definition; hence) it is no different from the other 0uasi$.udicial

bodies enumerated under Rule =(# 2esides) the introductory ,ords in Section ' of Circular "o#'$?' $$ Bamong these agencies are $$ indicate that the enumeration is not e/clusive or conclusiveand ac<no,ledge the e/istence of other 0uasi$.udicial agencies ,hich) though not e/presslylisted) should be deemed included therein# i/

Fourth, the Court reali*es that under 2atas Pambansa 2P! 2lg# ' ? / as amended by RA "o#?> ) /i factual controversies are usually involved in decisions of 0uasi$.udicial bodies; and theCA) ,hich is li<e,ise tas<ed to resolve 0uestions of fact) has more elbo, room to resolve them#2y including 0uestions of fact /ii among the issues that may be raised in an appeal from 0uasi$

.udicial agencies to the CA) Section 3 of Revised Administrative Circular "o# '$?E and Section (of Rule =( e/plicitly e/panded the list of such issues#

According to Section ( of Rule =() B[a]n appeal under this Rule may be ta<en to the Court of Appeals ,ithin the period and in the manner herein provided ,hether the appeal involves0uestions of fact) of la,) or mi/ed 0uestions of fact and la,# 9ence) appeals from 0uasi$.udicialagencies even only on 0uestions of la, may be brought to the CA#

8ifth) the .udicial policy of observing the hierarchy of courts dictates that direct resort fromadministrative agencies to this Court ,ill not be entertained) unless the redress desired cannot beobtained from the appropriate lo,er tribunals) or unless e/ceptional and compellingcircumstances .ustify availment of a remedy falling ,ithin and calling for the e/ercise of our

Page 9: Nat Res Digested

8/10/2019 Nat Res Digested

http://slidepdf.com/reader/full/nat-res-digested 9/173

primary .urisdiction# /iii

Consistent ,ith these rulings and legal bases) ,e therefore hold that Section ? of RA ?=is li<e,ise to be understood as having been modified by Circular "o# '$?') 2P 2lg# ' ? asamended by RA ?> ) Revised Administrative Circular '$?E) and Rule =( of the Rules of Court#%n brief) appeals from decisions of the MA2 shall be ta<en to the CA through petitions for revie,in accordance ,ith the provisions of Rule =( of the '?? Rules of Court#

Page 10: Nat Res Digested

8/10/2019 Nat Res Digested

http://slidepdf.com/reader/full/nat-res-digested 10/173

DESAMA vs ELISEA GOZUN G.R. No. 157882, Marc !", 2""#

FACTS:

E$%c&'(v% Or)%r No. 27*, +ro &- a'%) /0 ' % r%s()% ' Cora3o A4&( o,a&' or(3%) ' % DENR S%cr%'ar0 'o acc%+', co s()%r a ) %va-&a'% +ro+osa-s roor%( 6o %) cor+ora'(o s or or%( ( v%s'ors or co 'rac's o a r%% % 's ( vo-v(%(' %r '%c (ca- or ( a c(a- ass(s'a c% or -ar %6sca-% %$+-ora'(o , )%v%-o+ % ', a )&'(-(3a'(o o ( %ra-s, (c , &+o a++ro+r(a'% r%co % )a'(o o ' % S%cr%'ar0, ' %r%s()% ' a0 %$%c&'% (' ' % or%( +ro+o % '.

A '%r so % '( %, r%s()% ' F()%- . Ra os s( %) ( 'o -a R%+. Ac' No. 7*92% '('-%), A Ac' I s'('&'( A N% S0s'% o M( %ra- R%so&rc%s E$+-ora'(o ,D%v%-o+ % ', U'(-(3a'(o a ) Co s%rva'(o ,; o' %r (s% < o as ' % (-(++( % M( (

Ac' o 1**5. T % DENR S%cr%'ar0 (c'or O. Ra os (ss&%) DENR A) ( (s'ra'(v%Or)%r =DAO> No. 2!, S%r(%s o 1**5, co 'a( ( ' % ( +-% % '( &()%-( %s o R%+. Ac'No. 7*92. T (s as soo s&+%rs%)%) /0 DAO No. *#69", s. 1**#, (c 'oo< % %c' o2! ?a &ar0 1**7 a '%r )&% +&/-(ca'(o .

r%v(o&s-0, o %v%r, or s+%c( (ca--0 o 2" ?& % 1**9, r%s()% ' Ra os %$%c&'%)a FTAA (' Ar( co M( ( Cor+ora'(o =AMC> ov%r a 'o'a- -a ) ar%a o !7,"""%c'ar%s cov%r( ' % +rov( c%s o N&%va (3ca0a a ) @&(r( o. I c-&)%) ( ' (s ar%a (sara a0 D(+()(o, Bas(/&, N&%va (3ca0a. Co& s%-s or +%'('(o %rs (-%) a )% a )-%''%r a))r%ss%) 'o ' % DENR S%cr%'ar0 % %rso A-var%3, or ' % ca c%--a'(o o ' %CAMC FTAA or ' % +r( ar0 r%aso ' a' R%+. Ac' No. 7*92 a ) ('s I +-% % '( R&-%sa ) R% &-a'(o s DAO *#69" ar% & co s'('&'(o a-.

S&/s%4&% '-0, AMC co so-()a'%) (' C-( a$ M( ( L( ('%) 'o or a s( -%co +a 0 ' a' o o%s & )%r ' % % a % o C-( a$6Ar( co M( ( Cor+ora'(o=CAMC>, ' % co 'ro--( ** o s'oc< o-)%rs o (c ar% A&s'ra-(a a'(o a-s.MG r% %c'%) ' % )% a ) o co& s%-s or +%'('(o %rs or ' % ca c%--a'(o o ' % CAMCFTAA.

%'('(o %rs ' &s (-%) ' % +r%s% ' +%'('(o or +ro (/('(o a ) mandamus , (' a+ra0%r or a '% +orar0 r%s'ra( ( or)%r.

ISSUE:

%' %r or o' ' % ( ( ac' a ) ('s ( +-% % '( r&-%s a ) r% &-a'(o s ar% vo()a ) & co s'('&'(o a- s+%c( (ca--0 S%c'(o 7# o R%+. Ac' No. 7*92 a ) S%c'(o 1"7 o DAO *#69" FOR IT a--o s ' % & -a &- a ) & &s' 'a<( ; o +r(va'% +ro+%r'0 or +r(va'% +&r+os% ( co 'ra)(c'(o (' S%c'(o *, Ar'(c-% III o ' % 1*87 Co s'('&'(o .

ELD:

NO. S%c'(o 7# o R%+&/-(c Ac' No. 7*92 a ) S%c'(o 1"7 o DAO *#69"R%+&/-(c Ac' No. 7*92 a ) ('s I +-% % '( R&-%s a ) R% &-a'(o s co 'a( %) ( DAO

Page 11: Nat Res Digested

8/10/2019 Nat Res Digested

http://slidepdf.com/reader/full/nat-res-digested 11/173

*#69" H ( so ar as ' %0 r%-a'% 'o ( a c(a- a ) '%c (ca- ass(s'a c% a r%% % 's r% %rr%)'o ( +ara ra+ 9 o S%c'(o 2 o Ar'(c-% II o ' % Co s'('&'(o ar% NOTUNCONSTITUTIONAL.

(-% ' (s Co&r' )%c-ar%s ' a' ' % assa(-%) +rov(s(o (s a 'a<( +rov(s(o , ' (s

)o%s o' %a ' a' (' (s & co s'('&'(o a- o ' % ro& ) ' a' (' a--o s 'a<( o +r(va'%+ro+%r'0 (' o&' ' % )%'%r ( a'(o o +&/-(c &s% a ) ' % +a0 % ' o &s' co +% sa'(o .

T % 'a<( 'o /% va-() &s' /% or +&/-(c &s%. &/-(c &s% as a r%4&(r% % ' or ' %va-() %$%rc(s% o ' % +o %r o % ( % ' )o a( (s o s0 o 0 o&s (' +&/-(c ( '%r%s',+&/-(c /% % (', +&/-(c %- ar% a ) +&/-(c co v% (% c%. I' ( c-&)%s ' % /roa)%r o'(o o ( )(r%c' +&/-(c /% % (' or a)va 'a %. &/-(c &s% as 'ra)('(o a--0 & )%rs'oo) as ac'&a-&s% /0 ' % +&/-(c; as a-r%a)0 /%% a/a )o %).

M( ( ( )&s'r0 +-a0s a +(vo'a- ro-% ( ' % %co o (c )%v%-o+ % ' o ' % co& 'r0a ) (s a v('a- 'oo- ( ' % ov%r % 'Js ' r&s' o acc%-%ra'%) r%cov%r0. Irr% ra a/-0, ( (

(s a ( )&s'r0 (c (s o +&/-(c /% % ('. T a' +&/-(c &s% (s % a'%) /0 ' % ac' ' a' ' %s'a'% o&-) /% 'a<( +r(va'% +ro+%r'(%s or ' % /% % (' o +r(va'% ( ( (r s or ( (co 'rac'ors (s o' a' a-- 'r&%.

T %r% (s a-so o /as(s or ' % c-a( ' a' ' % M( ( La a ) ('s ( +-% % '(r&-%s a ) r% &-a'(o s )o o' +rov()% or &s' co +% sa'(o ( %$+ro+r(a'( +r(va'%+ro+%r'(%s. S%c'(o 7# o R%+. Ac' No. 7*92 a ) S%c'(o 1"7 o DAO *#69" +rov()% or ' % +a0 % ' o &s' co +% sa'(o

Page 12: Nat Res Digested

8/10/2019 Nat Res Digested

http://slidepdf.com/reader/full/nat-res-digested 12/173

LA UGAL6 JLAAN TRI AL ASSOCIATION, I c. vs RAMOS

G.R. No. 127882 ?a &ar0 27, 2""9

FACTS:

4n March () '??E) President Ramos signed into la, R#A# "o# ?= # Section 'E thereof declares that the Act Bshall govern the e/ploration) development) utili*ation) and processing of all mineral resources# Such declaration not,ithstanding) R#A# "o# ?= does not actually cover all the modes through ,hich the State may underta<e the e/ploration) development) andutili*ation of natural resources#

The State) being the o,ner of the natural resources) is accorded the primary po,er andresponsibility in the e/ploration) development and utili*ation thereof# As such) it may underta<ethese activities through four modes6

The State may directly underta<e such activities#

!The State may enter into co$production) .oint venture or production$sharing agreements,ith 8ilipino citi*ens or 0ualified corporations#

(!Congress may) by la,) allo, small$scale utili*ation of natural resources by 8ilipinociti*ens#

=!8or the large$scale e/ploration) development and utili*ation of minerals) petroleum andother mineral oils) the President may enter into agreements ,ith foreign$o,ned corporationsinvolving technical or financial assistance#

R#A# "o# ?= primarily concerns itself ,ith the second and fourth modes#

Petitioners submit that) in accordance ,ith the te/t of Section ) Article F%% of the

Constitution) 8TAAs should be limited to B technical or financial assistance only# Theyobserve) ho,ever) that) contrary to the language of the Constitution) the 7MCP 8TAA allo,s7MCP) a fully foreign$o,ned mining corporation) to e/tend more than mere financial or technical assistance to the State) for it permits 7MCP to manage and operate every aspect of themining activity

4n Ganuary '>) '?? ) counsels for petitioners sent a letter to the D3"R Secretarydemanding that the D3"R stop the implementation of R#A# "o# ?= and DA4 "o# ?H$=>)giving the D3"R fifteen days from receipt to act thereon# The D3"R) ho,ever) has yet torespond or act on petitioners- letter#

%'('(o %rs ' &s (-%) ' % +r%s% ' +%'('(o or +ro (/('(o a ) a )a &s, (' a

+ra0%r or a '% +orar0 r%s'ra( ( or)%r. T %0 a--% % ' a' a' ' % '( % o ' % (-( o ' %+%'('(o , 1"" FTAA a++-(ca'(o s a) a-r%a)0 /%% (-%), cov%r( a ar%a o 8.9 (--(o%c'ar%s, #9 o (c a++-(ca'(o s ar% /0 &--0 or%( 6o %) cor+ora'(o s cov%r( a'o'a- o 5.8 (--(o %c'ar%s, a ) a' -%as' o % /0 a &--0 or%( 6o %) ( ( co +a 0ov%r o s or% ar%as.

Page 13: Nat Res Digested

8/10/2019 Nat Res Digested

http://slidepdf.com/reader/full/nat-res-digested 13/173

ISSUE:

ET ER OR NOT R%+&/-(c Ac' No. 7*92 IS UNCONSTITUTIONAL.

ELD:

The Court hereby declares unconstitutional and void the follo,ing6

'! provisions of Republic Act "o# ?= 6

a!The proviso in Section ( a0!)

b!Section ()

c!Section (( to =')

d!Section EH)

e!The second and third paragraphs of Section @') and

f! Section ?>#! All provisions of Department of 3nvironment and "atural Resources Administrative

4rder ?H$=>) s# '??H ,hich are not in conformity ,ith this Decision) and

(! The 8inancial and Technical Assistance Agreement bet,een the 1overnment of theRepublic of the Philippines and 7MC Philippines) %nc#

%t is undisputed that R#A# "o# ?= and DA4 "o# ?H$=> contain provisions that are morefavorable to 7MCP) hence) these la,s) to the e/tent that they are favorable to 7MCP) governthe 8TAA#

%n addition) R#A# "o# ?= e/plicitly ma<es certain provisions apply to pre$e/isting

agreements#R#A# "o# ?= is invalid insofar as said Act authori*es service contracts# Although the

statute employs the phrase Bfinancial and technical agreements in accordance ,ith the '?@Constitution) it actually treats these agreements as service contracts that grant beneficialo,nership to foreign contractors contrary to the fundamental la,#

Page 14: Nat Res Digested

8/10/2019 Nat Res Digested

http://slidepdf.com/reader/full/nat-res-digested 14/173

APEX MINING CO., INC.,

%'('(o %r,

6 versus 6

SOUTHEAST MINDANAO GO D MINING CORP. ET A ,

R%s+o )% 's.

! " " " " " " " " " " " " " " " " " " " " " " " " " !

#A ITE COMMUNA PORTA MINING COOPERATI$E,

%'('(o %r,

6 versus 6

SOUTHEAST MINDANAO GO D MINING CORP., APEX MINING CO., INC., ET A ,

R%s+o )% 's.$ 6 6 6 6 6 6 6 6 6 6 6 6 6 6 6 6 6 6 6 6 6 6 6 6 $THE MINES AD%UDICATION #OARD AND ITS MEM#ERS, THE HON. $ICTOR O.RAMOS &C'()*+( -, UNDERSECRETAR $IRGI IO MARCE O &M/+ /*- (DIRECTOR HORACIO RAMOS &M/+ /*-,

%'('(o %rs,

6 versus 6

SOUTHEAST MINDANAO GO D MINING CORPORATION,

R/s o / t.

3ACTS

T (s r%so-v%s ' % o'(o or r%co s()%ra'(o )a'%) 12 ?&-0 2""#, (-%) /0 So&' %as'M( )a ao Go-) M( ( Cor+ora'(o =SEM>, o ' (s Co&r'Js D%c(s(o )a'%) 2! ?& % 2""#

Page 15: Nat Res Digested

8/10/2019 Nat Res Digested

http://slidepdf.com/reader/full/nat-res-digested 15/173

=Assa(-%) D%c(s(o >. T % Assa(-%) D%c(s(o %-) ' a' ' % ass( % ' o E$+-ora'(o%r (' =E > 1!! ( avor o SEM v(o-a'%) o % o ' % co )('(o s s'(+&-a'%) ( ' % +%r (',

i.e. , ' a' ' % sa % s a-- /% or ' % %$c-&s(v% &s% a ) /% % (' o Marco++%r M( (Cor+ora'(o =MMC> or ('s )&-0 a&' or(3%) a % 's. S( c% SEM )() o' c-a( or s&/ ('%v()% c% ' a' (' as a )%s( a'%) a % ' o MMC, ' % -a''%r ca o' /% co s()%r%) as aa % ' o ' % or %r ' a' ca &s% E 1!! a ) /% % (' ro ('. I' a-so r&-%) ' a' ' %'ra s %r o E 1!! v(o-a'%) r%s()% '(a- D%cr%% No. 9#!, (c r%4&(r%s ' a' ' %ass( % ' o a ( ( r( ' /% a)% (' ' % +r(or a++rova- o ' % S%cr%'ar0 o ' %D%+ar' % ' o E v(ro % ' a ) Na'&ra- R%so&rc%s =DENR>. Mor%ov%r, ' % Assa(-%)D%c(s(o +o( '%) o&' ' a' E 1!! %$+(r%) /0 o 6r% % a- s( c% (' as o' r% % %)/% or% or a '%r ('s %$+(ra'(o .

T % Assa(-%) D%c(s(o -(<% (s% &+ %-) ' % va-()('0 o roc-a a'(o No. 2*7a/s% ' a 0 4&%s'(o a a( s' ('s va-()('0. I v(% o ' (s, a ) co s()%r( ' a' & )%r S%c'(o 5 o R%+&/-(c Ac' No. 7*92, o' %r (s% < o as ' % M( ( Ac' o 1**5,;( ( o+%ra'(o s ( ( %ra- r%s%rva'(o s a0 /% & )%r'a<% )(r%c'-0 /0 ' % S'a'% or ' ro& a co 'rac'or, ' % Co&r' )%% %) ' % (ss&% o o %rs (+ o +r(or('0 r( ' ov%r ' %co '%s'%) D( a- a- Go-) R&s Ar%a as av( /%% ov%r'a<% /0 ' % sa()+roc-a a'(o . T &s, (' as %-) ( ' % Assa(-%) D%c(s(o ' a' (' (s o (' ( ' %+r%ro a'(v% o ' % E$%c&'(v% D%+ar' % ' 'o & )%r'a<% )(r%c'-0 ' % ( ( o+%ra'(o s o ' % )(s+&'%) ar%a or 'o a ar) ' % o+%ra'(o s 'o +r(va'% % '('(%s ( c-&)( +%'('(o %rs

A+%$ a ) a-('%, s&/ %c' 'o a++-(ca/-% -a s, r&-%s a ) r% &-a'(o s, a ) +rov()%) ' a'' %s% +r(va'% % '('(%s ar% 4&a-( (%).

SEM a-so (-%) a Mo'(o or R% %rra- o Cas% 'o ' % Co&r'En Banc a ) or Ora- Ar & % 's )a'%) 22 A& &s' 2""#.

Page 16: Nat Res Digested

8/10/2019 Nat Res Digested

http://slidepdf.com/reader/full/nat-res-digested 16/173

A+%$, or ('s +ar', (-%) a Mo'(o or C-ar( (ca'(o o ' % Assa(-%) D%c(s(o , +ra0(' a' ' % Co&r' %-&c()a'% o ' % D%c(s(o Js +ro o& c% % ' ' a' ( ( o+%ra'(o s, ar%o , ' %r% or% (' ( ' % &-- co 'ro- o ' % S'a'% ' ro& ' % %$%c&'(v% /ra c .;

Mor%ov%r, A+%$ as<s ' (s Co&r' 'o or)%r ' % M( %s a ) G%osc(% c%s oar) =MG > 'oacc%+' ('s a++-(ca'(o or a %$+-ora'(o +%r ('.

I ('s Ma ( %s'a'(o a ) Mo'(o )a'%) 28 ?&-0 2""#, a-('% %c o%s ' % sa %co c%r as ' a' o A+%$ o ' % ac'&a- 'a<%ov%r /0 ' % S'a'% o ' % ( ( ( )&s'r0 ( ' %)(s+&'%) ar%a 'o ' % %$c-&s(o o ' % +r(va'% s%c'or. I a))('(o , a-('% +ra0s or ' (sCo&r' 'o )(r%c' MG 'o acc%+' ('s a++-(ca'(o or a %$+-ora'(o +%r ('.

Ca (-o a a), et al ., -(<% (s% (-%) a o'(o or r%co s()%ra'(o a ) +ra0%) ' a'' % )(s+&'%) ar%a /% a ar)%) 'o ' % .

ISSUEKS:

Whether Southeast Mindanao Mining Corp. (SEM) acquired a vested right over the disputed area, which constitutes a property right protected y theConstitution.

ELD:

NO. SEM )o%s o' av%r or +rov% ' a' ('s ( ( r( 's a) /%% +%r %c'%) a )co +-%'%) % ' % (-(++( % (-- o 1*"2 as s'(-- ' % o+%ra'(v% -a . S&r%-0, (' (s( +oss(/-% or SEM 'o s&cc%ss &--0 ass%r' ' a' (' ac4&(r%) ( ( r( 's ov%r ' %)(s+&'%) ar%a ( accor)a c% (' ' % sa % /(--, s( c% (' as o -0 ( 1*89 ' a' MMC,SEMJs +r%)%c%ssor6( 6( '%r%s', (-%) ('s )%c-ara'(o o -oca'(o s a ) ('s +ros+%c'(+%r (' a++-(ca'(o ( co +-(a c% (' r%s()% '(a- D%cr%% No. 9#!. I' as o 1 ?&-0

Page 17: Nat Res Digested

8/10/2019 Nat Res Digested

http://slidepdf.com/reader/full/nat-res-digested 17/173

1*85 a ) 1" Marc 1*8# ' a' a ros+%c'( %r (' a ) E 1!!, r%s+%c'(v%-0, %r%(ss&%) 'o MMC. Co s()%r( ' %s% ac's, ' %r% (s o +oss(/(-('0 ' a' MMC or SEM co&-)av% ac4&(r%) a +%r %c'%) ( ( c-a( & )%r ' % a&s+(c%s o ' % (-(++( % (-- o

1*"2. a'%v%r ( ( r( 's MMC a) ' a' (' ( va-()-0 'ra s %rr%) 'o SEM ca o', /0a 0 s'r%'c o ( a ( a'(o , /% co s()%r%) ( ( r( 's; as co '% +-a'%) & )%r ' %(-(++( % (-- o 1*"2 a ) ( or'a-(3%) ( Mc!aniel a ) "old Cree# Mining .

SEM -(<% s E 1!! (' a /&(-)( +%r ('. SEM -(<% (s% %4&a'%s ('s s&++os%)r( 's a''ac %) 'o ' % %$+-ora'(o +%r (' (' ' % r( 's ' a' a +r(va'% +ro+%r'0 -a )o %r as 'o sa() -a ) o-)( . T (s a a-o 0 as o /as(s ( -a . As %ar-(%r )(sc&ss%),

& )%r ' % 1*!5, 1*7! a ) 1*87 Co s'('&'(o s, a'(o a- %a-' , s&c as ( %ra-r%so&rc%s, ar% o %) /0 ' % S'a'% a ) o' /0 ' %(r )(scov%r%r. T % )(scov%r%r or -oca'or ca o -0 )%v%-o+ a ) &'(-(3% sa() ( %ra-s or (s o /% % (' ( % as co +-(%)(' a-- ' % r%4&(r% % 's s%' or' /0 a++-(ca/-% -a s a ) ( ' % S'a'% as co %rr%) o( s&c r( ' ' ro& +%r ('s, co c%ss(o s or a r%% % 's. I o' %r or)s, (' o&' ' %( +r( a'&r o ' % S'a'%, a 0 ( ( as+(ra ' )o%s o' av% a 0 )% ( ('(v% r( ' ov%r ' %( %ra- -a ) /%ca&s%, & -(<% a +r(va'% -a ) o-)( , ( %ra- -a ) (s o %) /0 ' % S'a'%,a ) ' % sa % ca o' /% a-(% a'%) 'o a 0 +r(va'% +%rso as %$+-(c('-0 s'a'%) ( S%c'(o 2,

Ar'(c-% I o ' % 1*87 Co s'('&'(o :

A-- -a )s o +&/-(c )o a( , a'%rs, +) /*(5s ! ! ! ( (55 ot'/* (tu*(5 */sou* /s (*/ o / t'/ St(t/ . (' ' % %$c%+'(o o a r(c&-'&ra- -a )s, a-- o' %r (tu*(5 */sou* /s s'(55 ot / (5)/ (t/ .=E + as%s s&++-(%).>

F&r' %r, a c-os%r scr&'( 0 o ' % )%%) o ass( % ' ( avor o SEM r%v%a-s ' a'MMC ass( %) 'o ' % or %r ' % r( 's a ) ( '%r%s's (' a) ( E 1!!, ' &s:

Page 18: Nat Res Digested

8/10/2019 Nat Res Digested

http://slidepdf.com/reader/full/nat-res-digested 18/173

1. T a' or ONE ESO = 1.""> a ) o' %r va-&a/-% co s()%ra'(or%c%(v%) /0 ' % ASSIGNOR ro ' % ASSIGNEE, ' % ASSIGNOR %r%/0ASSIGNS, TRANS3ERS ( CON$E S u to t'/ ASSIGNEE '(t/9/* *)g'ts o* ) t/*/st t'/ ASSIGNOR +( '(9/ ) t'/ (*/( s)tu(t/ )Mo :( o, D(9(o /5 No*t/ ( C(t//5, D(9(o O*)/ t(5, ) / t);)/ (sE! 5o*(t)o P/*+)t No. 1<< a ) A++-(ca'(o or a %r (' 'o ros+%c' (& a a , A &sa )%- S&r r%s+%c'(v%-0. =E + as(s s&++-(%).>

I' (s %v()% ' ' a' a' MMC a) ov%r ' % )(s+&'%) ar%a )&r( ' % ass( % 'as a %$+-ora'(o +%r ('. C-%ar-0, ' % r( ' ' a' SEM ac4&(r%) as -( ('%) 'o%$+-ora'(o , o -0 /%ca&s% MMC as a %r% o-)%r o a %$+-ora'(o +%r ('. As+r%v(o&s-0 %$+-a( %), SEM )() o' ac4&(r% ' % r( 's ( %r% ' ( ' % +%r (', as ' %ass( % ' /0 MMC 'o SEM as )o % ( v(o-a'(o o ' % co )('(o s'(+&-a'%) (' % +%r (', a ) ' % ass( % ' as % %c'%) (' o&' ' % a++rova- o ' % +ro+%r a&' or('0 ( co 'rav% '(o o ' % +rov(s(o o ' % ( ( -a ov%r ( a' ' a''( %. I a))('(o , ' % +%r (' %$+(r%) o # ?&-0 1**9. I' (s, ' %r% or%, 4&('% c-%ar ' a' SEM as o r( ' ov%r ' % ar%a.

Page 19: Nat Res Digested

8/10/2019 Nat Res Digested

http://slidepdf.com/reader/full/nat-res-digested 19/173

CELESTIAL NICBEL MINING G.R. No. 1#*"8"E LORATION COR ORATION,

%'('(o %r,

6 v%rs&s 6

MACROASIA COR ORATION= or %r-0 INFANTA MINERAL AND INDUSTRIALCOR ORATION>,LUE RIDGE MINERAL COR ORATION, a ) LE AC MINING COR ORATION,

R%s+o )% 's.

FACTS:

T % S%cr%'ar0 o A r(c&-'&r% a ) Na'&ra- R%so&rc%s a ) I a 'a M( %ra- a )I )&s'r(a- Cor+ora'(o =I a 'a> % '%r%) ( 'o a M( ( L%as% Co 'rac' 61"5".

I a 'aJs cor+ora'% a % as ' % c a %) 'o Co/%r'so o-)( s Cor+ora'(oa ) s&/s%4&% '-0 'o ('s +r%s% ' a %, Macroas(a Cor+ora'(o .

A '%r so %'( %, C%-%s'(a- (-%) a %'('(o 'o Ca c%- ' % s&/ %c' ( ( -%as%co 'rac's a ) o' %r ( ( c-a( s o Macroas(a ( c-&)( ' os% cov%r%) /0 M( (L%as% Co 'rac' No. 61"5", /% or% ' % a %- o Ar/('ra'ors = OA> o ' % M( %s a )G%o6Sc(% c%s &r%a& =MG > o ' % DENR.

-&% R() %, ( a %ar-(%r -%''%r6+%'('(o , a-so ro'% ' % D(r%c'or o M( %s 'o s%%<ca c%--a'(o o ( ( -%as% co 'rac's a ) o' %r ( ( r( 's o Macroas(a a ) a o' %r % '('0, L%/ac M( ( Cor+ora'(o =L%/ac >, ( ( ( ar%as ( roo<%Js o( '.

C%-%s'(a- (s ' % ass( %% o 199 ( ( c-a( s cov%r( s&c ar%as co '( &o&s 'oI a 'aJs = o Macroas(a> ( ( -o)% c-a( s. C%-%s'(a- a-so o-)s a M SA (' ' %ov%r % ' (c cov%rs 2,8!5 %c'ar%s -oca'%) a' I+(-a KMaas( , roo<%Js o( ',a-a a a ) ' o +% )( a++-(ca'(o s cov%r( a o' %r 9,"9" %c'ar%s ( ara a0Ma( (' a-so ( roo<%Js o( '.

C%-%s'(a- so& ' ' % ca c%--a'(o o Macroas(aJs -%as% co 'rac's.

Macroas(a r% &'%) ' % ro& )s or ca c%--a'(o ( vo<%) /0 C%-%s'(a-.

as%) o ' % r%cor)s o ' % &r%a& o M( %s a ) ( )( s o ' % (%-)( v%s'( a'(o s, ' % OA ra '%) ' % +%'('(o o C%-%s'(a- 'o ca c%- ' % M( ( L%as%Co 'rac's o Macroas(a a ) o& ) ' % c-a( s o ' % o' %rs ( )&/('a/-0 %r('or(o&s. I'av% C%-%s'(a- ' % +r% %r% '(a- r( ' 'o Macroas(aJs ( ( ar%as.1 I' &+ %-) -&%

R() %Js +%'('(o , /&' o -0 as a a( s' ' % M( ( L%as% Co 'rac' ar%as o L%/ac , a )' % sa() -%as%) ar%as %r% )%c-ar%) a&'o a'(ca--0 a/a )o %). I' av% -&% R() %

1

Page 20: Nat Res Digested

8/10/2019 Nat Res Digested

http://slidepdf.com/reader/full/nat-res-digested 20/173

+r(or('0 r( ' 'o ' % a or%sa() L%/ac Js ar%asK ( ( c-a( s. -&% R() % a ) Macroas(aa++%a-%) /% or% ' % MA .

L%/ac )() o' (-% a 0 o'(c% o a++%a- (' ' % r%4&(r%) % ora )& o a++%a- ' &s, (' r%s+%c' 'o L%/ac , ' % a/ov% r%so-&'(o /%ca % ( a- a ) %$%c&'or0.

T % MA a)% a )%c(s(o &+ o-)( ' % D%c(s(o o ' % OA 'o ca c%- ' %M( ( Lo)%KL%as% Co 'rac's o Macroas(a.

o %v%r, ' % MA , s&/s%4&% '-0 (ss&%) a r%so-&'(o vaca'( ('s +r%v(o&s)%c(s(o , o-)( ' a' %(' %r ' % OA or ' % MA a) ' % +o %r 'o r%vo<% a ( %ra-a r%% % ' )&-0 % '%r%) ( 'o /0 ' % DENR S%cr%'ar0. T % MA &r' %r %-) ' a' ' %+o %r 'o ca c%- or r%vo<% a ( %ra- a r%% % ' as %$c-&s(v%-0 -o) %) (' ' % DENRS%cr%'ar0.

C%-%s'(a- a ) -&% R() % a)% a a++%a-.

T % CA S+%c(a-12' D(v(s(o a (r %) ' % MA R%so-&'(o (c &+ %-) ' %%$c-&s(v% a&' or('0 o ' % DENR S%cr%'ar0 'o a++rov%, ca c%-, a ) r%vo<% ( %ra-a r%% % 's. T % CA a-so )% (%) C%-%s'(a-Js Mo'(o or R%co s()%ra'(o .

(-% ' % CA S+%c(a- 1"' D(v(s(o ra '%) -&% R() %Js +%'('(o r%v%rs%) a )s%' as()% ' % R%so-&'(o s o ' % MA a ) 'r%a'%) ' % ca c%--a'(o o a ( ( -%as%a r%% % ' as a ( ( )(s+&'% (' ( ' % %$c-&s(v% &r(s)(c'(o o ' % OA & )%r S%c.77 o RA 7*92, %$+-a( ( ' a' ' % +o %r 'o r%so-v% ( ( )(s+&'%s, (c (s ' %r%a'%r +o %r, %c%ssar(-0 ( c-&)%s ' % -%ss%r +o %r 'o ca c%- ( ( a r%% % 's.

ISSUE:

%' %r or o' (' (s o -0 ' % S%cr%'ar0 o ' % DENR o as ' % &r(s)(c'(o 'oca c%- ( ( co 'rac's a ) +r(v(-% %s

ELD:

ES. I' (s o -0 ' % S%cr%'ar0 o ' % DENR o as &r(s)(c'(o 'o ca c%- ( (co 'rac's a ) +r(v(-% %s.

A '%r a scr&'( 0 o ' % +rov(s(o s o D 9#!, EO 211, EO 27*, RA 7*92 a ) ('s( +-% % '( r&-%s a ) r% &-a'(o s, %$%c&'(v% (ss&a c%s, a ) cas% -a , % r&-% ' a' ' %DENR S%cr%'ar0, o' ' % OA, as ' % &r(s)(c'(o 'o ca c%- %$(s'( ( %ra- -%as%co 'rac's or ( %ra- a r%% % 's /as%) o ' % o--o ( r%aso s:

T % +o %r o ' % DENR S%cr%'ar0 'o ca c%- ( %ra- a r%% % 's % a a'%s ro(s a) ( (s'ra'(v% a&' or('0, s&+%rv(s(o , a a % % ', a ) co 'ro- ov%r ( %ra-

Page 21: Nat Res Digested

8/10/2019 Nat Res Digested

http://slidepdf.com/reader/full/nat-res-digested 21/173

Page 22: Nat Res Digested

8/10/2019 Nat Res Digested

http://slidepdf.com/reader/full/nat-res-digested 22/173

[G.R. No. 86889. December 4, 1990]

LUZ FARMS, petitioner , vs . THE H N RA!LE SE"RETAR# F THE

DE$ARTMENT F AGRAR%AN REF RM, respondent .

FACTS: On June 10, 1988, the President of the Philippines approved R A !o

""#$, %hi&h in&ludes the raisin' of livesto&(, poultr) and s%ine in its

&overa'e On Januar) *, 1989, the Se&retar) of A'rarian Refor+

pro+ul'ated the uidelines and Pro&edures -+ple+entin' Produ&tion and

Profit Sharin' as e+.odied in Se&tions 1/ and /* of R A !o ""#$ On

Januar) 9, 1989, the Se&retar) of A'rarian Refor+ pro+ul'ated its Rules

and Re'ulations i+ple+entin' Se&tion 11 of R A !o ""#$ Co++er&ial

Far+s2u3 Far+s, petitioner in this &ase, is a &orporation en'a'ed in the

livesto&( and poultr) .usiness and to'ether %ith others in the sa+e .usiness

alle'edl) stands to .e adversel) affe&ted .) the enfor&e+ent of Se&tion

/ . , Se&tion 11, Se&tion 1/, Se&tion 1" d and 1$ and Se&tion /* of R A

!o ""#$ other%ise (no%n as Co+prehensive A'rarian Refor+ 2a% and of

the uidelines and Pro&edures -+ple+entin' Produ&tion and Profit Sharin'

under R A !o ""#$ pro+ul'ated on Januar) *, 1989 and the Rules and

Re'ulations -+ple+entin' Se&tion 11 thereof as pro+ul'ated .) the 4AR on

Januar) 9, 19895en&e, this petition pra)in' that aforesaid la%s, 'uidelines and rules

.e de&lared un&onstitutional 6ean%hile, it is also pra)ed that a %rit of

preli+inar) in7un&tion or restrainin' order .e issued en7oinin' pu.li&

respondents fro+ enfor&in' the sa+e, insofar as the) are +ade to appl) to2u3 Far+s and other livesto&( and poultr) raisers This Court in its

Resolution dated Jul) , 1989 resolved to den), a+on' others, 2u3 Far+s

pra)er the issuan&e of a preli+inar) in7un&tion in its 6anifestation dated 6a)

*" and /1, 1989

Page 23: Nat Res Digested

8/10/2019 Nat Res Digested

http://slidepdf.com/reader/full/nat-res-digested 23/173

2ater, ho%ever, this Court in its Resolution dated Au'ust * , 1989

resolved to 'rant said 6otion for Re&onsideration re'ardin' the in7un&tive

relief, after the filin' and approval .) this Court of an in7un&tion .ond in the

a+ount of P100,000 00 This Court also 'ave due &ourse to the petition and

re uired the parties to file their respe&tive +e+oranda-SS;<: =O! Se&tion / . , Se&tion 11, Se&tion 1/, Se&tion 1" d and 1$ and

Se&tion /* of R A !o ""#$ other%ise (no%n as Co+prehensive A'rarian

Refor+ 2a% and of the uidelines and Pro&edures -+ple+entin' Produ&tion

and Profit Sharin' under R A !o ""#$ is un&onstitutional5<24: ><S -t is evident fro+ the fore'oin' dis&ussion that Se&tion -- of R A

""#$ %hi&h in&ludes ?private a'ri&ultural lands devoted to &o++er&ial

livesto&(, poultr) and s%ine raisin'@ in the definition of &o++er&ial far+sis invalid, to the eBtent that the afore&ited a'ro industrial a&tivities are +ade

to .e &overed .) the a'rarian refor+ pro'ra+ of the State There is si+pl)

no reason to in&lude livesto&( and poultr) lands in the &overa'e of a'rarian

refor+5en&e, there is +erit in 2u3 Far+s ar'u+ent that the re uire+ent in

Se&tions 1/ and /* of R A ""#$ dire&tin' ?&orporate far+s@ %hi&h in&lude

livesto&( and poultr) raisers to eBe&ute and i+ple+ent ?produ&tion sharin'plans@ pendin' final redistri.ution of their landholdin's %here.) the) are

&alled upon to distri.ute fro+ three per&ent /D of their 'ross sales and

ten per&ent 10D of their net profits to their %or(ers as additional

&o+pensation is unreasona.le for .ein' &onfis&ator), and therefore violative

of due pro&ess

Page 24: Nat Res Digested

8/10/2019 Nat Res Digested

http://slidepdf.com/reader/full/nat-res-digested 24/173

E R !o "1*9/ Fe.ruar) 1#, 1990

4O6-! O G 6A44;6GA and A!-TA C 6A44;6GA, petitioners , vsOH<R!6<!T S<RH-C< -!S;RA!C< S>ST<6, Represented .) its Chair+an,

Goard of Trustees, 5O!ORAG2< 2<O!-2O OCA6PO, respondent

FACTS: On 4e&e+.er 10, 1980, respondent S-S &ondu&ted a pu.li& .iddin'

of several fore&losed properties -n&luded in the properties offered to the

pu.li& %as a house and lot situated at //$$ !e% Panaderos Street, Sta Ana,

6anila, &overed .) Transfer Certifi&ate of Title !o $ 9 of the Re'ister of

4eeds of 6anilaPetitioner 4o+in'o G 6addu+.a parti&ipated in the pu.li& .iddin' and

su.+itted his sealed .id in the a+ount of P98,000 00 in Philippine &urren&)

The .id %as su.7e&t to the &ondition that there should .e a do%n pa)+ent of

/#D of the a+ount thereof, the 10D &onstitutin' the proposal .ond %ith the

re+ainin' *#D to .e paid after the re&eipt of the noti&e of a%ard or

a&&eptan&e of the .id A&&ordin'l), petitioner en&losed %ith his sealed .id a

+ana'erIs &he&( in the a+ount of P9,#00 00 and &ash in the a+ount of

P/00 00 to &o+plete the P9,800 00 proposal .ond;pon the re&eipt of the noti&e of a%ard, petitioner offered to pa) the

additional *#D in 2and Gan( .onds at their fa&e value These .onds %ere

issued to petitioner as pa)+ent for his ri&eland &onsistin' of t%ent) siB

he&tares lo&ated in Cordon, -sa.ela a& uired .) the overn+ent fro+ hi+

under Presidential 4e&ree !o *$ 5o%ever, the S-S re7e&ted the offer,

hen&e it %as %ithdra%n .) petitioner Petitioner then offered to pa) in &ashthe re+ainin' *#D do%n pa)+ent and all future install+ents Thereafter,

on !ove+.er 1", 1981, petitioner paid in &ash the .alan&e of the re uired

do%n pa)+entA 4eed of Conditional Sale %as eBe&uted .) the parties on !ove+.er 19,

1981, %here the petitioner as vendee a'reed to pa) the vendor S-S the

Page 25: Nat Res Digested

8/10/2019 Nat Res Digested

http://slidepdf.com/reader/full/nat-res-digested 25/173

.alan&e of the pur&hase pri&e of S- T> T5R<< T5O;SA!4 S<H<! 5;!4R<4

F-H< K #0L100 P"/,$0# #0 P<SOS Philippine &urren&), in S- T> "0

+onthl) install+ents of O!< T5O;SA!4 FO;R 5;!4R<4 S- T<<! K "9L100

P1, 1" "9 P<SOS Philippine &urren&), at t%elve 1*D per&ent interest

per annu+, &o+pounded +onthl), .e'innin' 4e&e+.er 1, 1981The first install+ent in the a+ount of P1, 1" 00 %as paid .) petitioner on

4e&e+.er /, 1981 =hen the se&ond +onthl) install+ent .e&a+e due,

petitioner sent a letter dated Januar) #, 198*, to the S-S Goard of Trustees

re uestin' that he .e allo%ed to pa) the +onthl) a+orti3ations %ith his

2and Gan( .onds &o++en&in' in Januar), 198* until the eBhaustion of the

said .onds Petitioner invo(ed the provisions of Se&tion 8# of Repu.li& A&t

!o /8 , as a+ended .) Presidential 4e&ree !o *#1The S-S Goard of Trustees, in its Resolution !o 91 adopted on Januar) **,

198*, denied petitionerIs offer The .oard ?resolved to reiterate the poli&)

that 2and Gan( .onds shall .e a&&epted as pa)+ent onl) at a dis&ounted

rate to )ield the S)ste+ 18D at +aturit) @ -n a letter dated Fe.ruar) 1*, 198*, petitioner as(ed the Goard of Trustees

to re&onsider Resolution !o 91 Petitioner reiterated his relian&e on Se&tion

8# of Repu.li& A&t !o /8 , as a+ended, and further supported his position%ith the &ontention that the poli&) of the S-S &ontravenes the rulin' in the

&ase of on3ales, et al vs The overn+ent -nsuran&e S)ste+, et& , et al

2i(e%ise, petitioner su.+itted an opinion of the 6inistr) of A'rarian Refor+,

dated Fe.ruar) 1*, 198*, %herein it %as stated, inter alia, that ?if the S-S

a&&epts the 2and Gan( .onds as pa)+ent thereof, it +ust a&&ept the sa+e

at par or fa&e value To a&&ept said .onds at a dis&ounted rate %ould lessen

the &redi.ilit) of the .onds as instru+ents of inde.tedness @ -n a letter dated 6a) /1, 198*, petitioner %as advised .) the 6ana'er,

A& uired Assets 4epart+ent, S-S that Resolution !o 1# %as adopted on

6a) 18, 198* .) the S-S Goard of Trustees den)in' the re uest of

petitioner 5en&e, on Au'ust #, 198*, the instant ori'inal a&tion for

mandamus %as filed .) petitioner

Page 26: Nat Res Digested

8/10/2019 Nat Res Digested

http://slidepdf.com/reader/full/nat-res-digested 26/173

-SS;<: =hether or not under the provisions of Se&tion 8# of Repu.li& A&t

!o /8 , as a+ended .) Presidential 4e&ree !o *#1 effe&tive Jul) *1,

19$/, the S-S +a) .e &o+pelled to a&&ept 2and Gan( .onds at their fa&e

value in pa)+ent for a residential house and lot pur&hased .) the

.ondholder fro+ the S-S5<24: >es -t is not disputed that under the a.ove uoted provisions, a

'overn+ent o%ned or &ontrolled &orporation, li(e the S-S, is &o+pelled to

a&&ept 2and Gan( .onds as pa)+ent for the pur&hase of its assets As a

+atter of fa&t, the .idder %ho offers to pa) in .onds of the 2and Gan( is

entitled to preferen&e =hat respondent S-S is resistin', ho%ever, is its

.ein' &o+pelled to a&&ept said .onds at their fa&e value Respondent, in

support of its stan&e that it &an dis&ount the .onds, avers that a P4 *#1

has a+ended Se&tion 8# of RA /8 .) deletin' and eli+inatin' the ori'inal

provision that 2and Gan( .onds shall .e a&&epted Min the a+ount of their

fa&e value N and . to a&&ept the said .onds at their fa&e value %ill i+pair

the a&tuarial solven&) of the S-S and thorou'hl) pre7udi&e its &apa&it) to

pa) death, retire+ent, insuran&e, dividends and other .enefits and &lai+s to

its +ore than a +illion +e+.ers, the +a7orit) of %ho+ are lo% salaried

'overn+ent e+plo)ees and %or(ers

Page 27: Nat Res Digested

8/10/2019 Nat Res Digested

http://slidepdf.com/reader/full/nat-res-digested 27/173

Page 28: Nat Res Digested

8/10/2019 Nat Res Digested

http://slidepdf.com/reader/full/nat-res-digested 28/173

E R !o 1/ 9#8 Januar) /1, *001

PATR-C-O C;TARA!, 4AH-4 4A! =AS and PAC-O 4OS-2, petitioners, vs.4<PART6<!T OF <!H-RO!6<!T and !AT;RA2 R<SO;RC<S, herein

represented .) S<C H-CTOR O RA6OS, OSCAR 6 5A6A4A and

;-22<R6O S F-A! A, in his &apa&it) as Chair+an of Co++unit) Spe&ial

Tas( For&e on An&estral 2ands CSTFA2 , Ga'uio Cit), respondents.

FACTS: Cutaran et al assails the validit) of 4<!R Spe&ial Order/1, Spe&ial

Order *#, and 4epart+ent Ad+inistrative Order * for .ein' issued %ithout

prior le'islative authorit) Spe&ial Order SO /1 1990 : ?Creation of a

Spe&ial Tas( for&e on a&&eptan&e, identifi&ation, evaluation and delineation of

an&estral land &lai+s in the Cordillera Ad+inistrative Re'ion@ Spe&ial Order

SO *#: ?Creation of Spe&ial Tas( For&es provin&ial and &o++unit)

environ+ent and natural resour&es offi&es for the identifi&ation, delineation

and re&o'nition of an&estral land &lai+s nation%ide@ 4AO *: -+ple+entin'

Rules and uidelines of Spe&ial Order no *# The sa+e )ear SO /1 %as

issued, relatives of petitioners filed separate appli&ations for Certifi&ate of

An&estral 2and Clai+ CA2C for the land the) o&&up) inside the Ca+p John

5a) Reservation These petitions %ere denied Also pursuant to the SO s,

the heirs of A pe' Carantes filed appli&ation for CA2C for so+e portions of

land in the Ca+p John 5a) Reservation, overlappin' so+e of the land

o&&upied .) the petitioners The petitioners &ontend that if not for the

respondent s ti+el) resistan&e to the Orders, the petitioners %ould .e totall)

evi&ted fro+ their land Petitioners filed in the CA petition to en7oin

respondents fro+ i+ple+entin' Orders on 'round that the) are void for la&(

of le'al .asis CA ruled that SO/1 has no for&e and effe&t for pree+ptin'

le'islative prero'ative for it %as issued prior to the effe&tivit) of RA$#8"

!ational -nte'rated Prote&ted S)ste+s , .ut it sustained SO*#and 4AO *

Page 29: Nat Res Digested

8/10/2019 Nat Res Digested

http://slidepdf.com/reader/full/nat-res-digested 29/173

on the 'round that the) %ere issued pursuant to po%ers dele'ated to 4<!R

under RA$#8" Petitioners no% &ontend that CA erred in upholdin' the

validit) of SO*# and 4AO * and see( to en7oin the 4<!R fro+ pro&essin' the

appli&ation of CA2C of 5eirs of Carantes-SS;<: =O! SO *# and 4AO * are valid5<24: !ot a 7usti&ia.le &ontrovers) The petition %as pre+aturel) filed

There is )et no 7usti&ia.le &ontrovers) for the &ourt to resolve The adverse

le'al interests involved are the &o+petin' &lai+s of the petitioners and heirs

of Carantes to possess a &o++on pie&e of land Sin&e the CA2C appli&ation

of the 5eirs of Carantes has not )et .een 'ranted or issued, and %hi&h the

4<!R +a) or +a) not 'rant, there is )et no a&tual or i++inent violation of

petitioner s asserted ri'ht to possess the disputed land 4efinition of 7usti&ia.le &ontrovers): a definite and &on&rete dispute tou&hin' on the le'al

relations of parties havin' adverse le'al interests %hi&h +a) .e resolved .)

a &ourt of la% throu'h the appli&ation of a la% Su.7e&t to &ertain %ell

defined eB&eptions, the &ourts %ill not tou&h an issue involvin' the validit) of

a la% unless there has .een a 'overn+ental a&t a&&o+plished or perfor+ed

that has a dire&t adverse effe&t on the le'al ri'ht of the person &ontestin' its

validit) This Court &annot rule on the .asis of petitioners spe&ulation thatthe 4<!R %ill approve the appli&ation of the heirs of Carantes There +ust

.e an a&tual 'overn+ental a&t %hi&h dire&tl) &auses or %ill i++inentl) &ause

in7ur) to the alle'ed le'al ri'ht of the petitioner to possess the land .efore

the 7urisdi&tion of this Court +a) .e invo(ed There is no sho%in' that the

petitioners %ere .ein' evi&ted fro+ the land .) the heirs of Carantes under

orders fro+ the 4<!R

Page 30: Nat Res Digested

8/10/2019 Nat Res Digested

http://slidepdf.com/reader/full/nat-res-digested 30/173

Page 31: Nat Res Digested

8/10/2019 Nat Res Digested

http://slidepdf.com/reader/full/nat-res-digested 31/173

E R !o #9"0/ April *9, 198$

< PORT PROC<SS-! O!< A;T5OR-T>, petitioner , vs 5O! C<F<R-!O <4;2A>, in his &apa&it) as the Presidin' Jud'e, Court of First -nstan&e of

Ce.u, Gran&h H-, 2apu 2apu Cit), and SA! A!TO!-O 4<H<2OP6<!T

CORPORAT-O!, respondents

F'c()& The four par&els of land %hi&h are the su.7e&t of this &ase is %here

the 6a&tan <Bport Pro&essin' one Authorit) in Ce.u <P A is to .e

&onstru&ted Private respondent San Antonio 4evelop+ent Corporation San

Antonio, for .revit) , in %hi&h these lands are re'istered under, &lai+ed that

the lands %ere eBpropriated to the 'overn+ent %ithout the+ rea&hin' the

a'ree+ent as to the &o+pensation Respondent Jud'e 4ula) then issued an

order for the appoint+ent of the &o++issioners to deter+ine the 7ust

&o+pensation -t %as later found out that the pa)+ent of the 'overn+ent to

San Antonio %ould .e P1# per s uare +eter, %hi&h %as o.7e&ted to .) the

latter &ontendin' that under P4 1#//, the .asis of 7ust &o+pensation shall

.e fair and a&&ordin' to the fair +ar(et value de&lared .) the o%ner of the

propert) sou'ht to .e eBpropriated, or .) the assessor, %hi&hever is lo%er

Su&h o.7e&tion and the su.se uent 6otion for Re&onsideration %ere denied

and hearin' %as set for the re&eption of the &o++issioner s report <P A

then filed this petition for &ertiorari and +anda+us en7oinin' the respondent

fro+ further hearin' the &ase

%))*e& =hether or !ot the eB&lusive and +andator) +ode of deter+inin'

7ust &o+pensation in P4 1#// is un&onstitutional

He+ & The Supre+e Court ruled that the +ode of deter+ination of 7ust

&o+pensation in P4 1#// is un&onstitutional

Page 32: Nat Res Digested

8/10/2019 Nat Res Digested

http://slidepdf.com/reader/full/nat-res-digested 32/173

The +ethod of as&ertainin' 7ust &o+pensation &onstitutes i+per+issi.le

en&roa&h+ent to 7udi&ial prero'atives -t tends to render the &ourts inutile in

a +atter in %hi&h under the Constitution is reserved to it for finan&ial

deter+ination The valuation in the de&ree +a) onl) serve as 'uidin'

prin&iple or one of the fa&tors in deter+inin' 7ust &o+pensation, .ut it +a)

not su.stitute the &ourt s o%n 7ud'+ent as to %hat a+ount should .e

a%arded and ho% to arrive at su&h a+ount The deter+ination of 7ust

&o+pensation is a 7udi&ial fun&tion The eBe&utive depart+ent or the

le'islature +a) +a(e the initial deter+ination .ut %hen a part) &lai+s a

violation of the 'uarantee in the Gill of Ri'hts that the private part) +a) not

.e ta(en for pu.li& use %ithout 7ust &o+pensation, no statute, de&ree, or

eBe&utive order &an +andate that its o%n deter+ination shall prevail over

the &ourt s findin's 6u&h less &an the &ourts .e pre&luded fro+ loo(in' into

the 7ustness of the de&reed &o+pensation

Page 33: Nat Res Digested

8/10/2019 Nat Res Digested

http://slidepdf.com/reader/full/nat-res-digested 33/173

E R !o 1*$*9" Januar) **, 1998<4;G- -S OR4;2A, C<2SO H F<R!A!4< , JR , C<2SO A F<R!A!4< ,

!ORA <22<! <STR<22A4O, 4<H<2OP6<!T GA!Q OF T5< P5-2-PP-!<S, J F

F<ST<JO A!4 CO , -!C A!4 R< -ST<R OF 4<<4S OF 2A ;!A, petitioners , vs. T5< 5O!ORAG2< CO;RT OF APP<A2S and R<P;G2-C OF T5<

P5-2-PP-!<S represented .) the !ational Po%er Corporation , respondents.

FACTS: For+er President Ferdinand < 6ar&os issued Pro&la+ation !o

#$/E/ %ithdra%in' fro+ sale and settle+ent and settin' aside as

per+anent forest reserves, su.7e&t to private ri'hts, &ertain par&els of the

pu.li& do+ain %hi&h in&luded Par&el !o 9 Calira)a 2u+ot River Forest

Reserve The) %ere pri+aril) for use as %atershed area The par&el of land

su.7e&t of the &ase at .ar is, .) petitionersI eBpli&it ad+ission,E %ithin

Par&el !o 9, the Calira)a 2u+ot River Forest Reserve Petitioner <du.i'is

ordula filed %ith the Gureau of 2ands, an Appli&ationE# for a Free Patent

over the land 6anuel Fernande3 and several others also filed free patent

appli&ations &overin' other par&els of land in the area 6r Antonio A uino,

Jr , the Civil Se&urit) Offi&er of the Cavinti reservoir &o+pleB, sent a

6e+orandu+ to the President of the !apo&or infor+in' hi+ of the fen&es

and roads .ein' &onstru&ted in the saddle area, +ore parti&ularl), in the lots

sold .) petitioner Fernande3 to petitioner <strellado Respondent Repu.li&,

throu'h the !apo&or, filed a'ainst petitioners a Co+plaint for Annul+ent of

Free Patent and Can&ellation of Titles and Reversion %ith =rit of Preli+inar)

-n7un&tion in the RTC of Sta Cru3, 2a'una The trial &ourt rendered

7ud'+ent in favor of petitioners Respondent Repu.li&, throu'h the !apo&or,

elevated the &ase to the respondent Court of Appeals On June *0, 199", the

respondent Court of Appeals ruled a'ainst petitioners 5en&e, this petition-SS;<: =hether or not the su.7e&t par&els of land are non disposa.le and

inaliena.le pu.li& land

Page 34: Nat Res Digested

8/10/2019 Nat Res Digested

http://slidepdf.com/reader/full/nat-res-digested 34/173

5<24: The t%o * par&els of land %ere pu.li& disposa.le and aliena.le lands

.efore the issuan&e, .) the for+er President, of Pro&la+ation !o #$/, on

June *", 19" The propert) %as, ho%ever, later reserved, under

Pro&la+ation !o #$/, as a per+anent forest, on June*", 19"E9 Sin&e

then, the propert) .e&a+e non disposa.le and inaliena.le pu.li& land G)

their ver) nature or .) eBe&utive or statutor) fiat, the) are outside the

&o++er&e of +an, unsus&epti.le of private appropriation in an) for+ and

in&onverti.le into an) &hara&ter less than of inaliena.le pu.li& do+ain,

re'ardless of their a&tual state, for as lon' as the reservation su.sists and is

not revo(ed .) a su.se uent valid de&lassifi&ation Petitioners do not &ontest

the nature of the land in the &ase at .ar -t is ad+itted that it lies in the

heart of the Calira)a 2u+ot River Forest Reserve, %hi&h Pro&la+ation !o

#$/ &lassified as inaliena.le and in disposa.le !o pu.li& land &an .e

a& uired .) private persons %ithout an) 'rant, eBpress or i+plied fro+ the

'overn+entN it is indispensa.le that there .e a sho%in' of a title fro+ the

state The fa&ts sho% that petitioner ordula, did not a& uire title to the

su.7e&t land prior to its reservation under Pro&la+ation !o #$/ 5e filed his

appli&ation for free patent onl) in Januar), 19$/, +ore than three / )earsafter the issuan&e of Pro&la+ation !o #$/ in June, 19"9 At that ti+e, the

land, as part of the Calira)a 2u+ot River Forest Reserve, %as no lon'er open

to private o%nership as it has .een &lassified as pu.li& forest reserve for the

pu.li& 'ood

Page 35: Nat Res Digested

8/10/2019 Nat Res Digested

http://slidepdf.com/reader/full/nat-res-digested 35/173

J 6 T;ASO! K CO , -!C v 2A!4 T<!;R< A46-!-STRAT-O!

FACTS: R A *"1" authori3ed eBpropriation of the Tatalon <state in ue3onCit) o%ned .) petitioner and * others 2ands %ere to .e divided to lots to .e

sold The) pra)ed that it .e de&lared un&onstitutional .e&ause violative of

e ual prote&tion &lause sin&e statute applies onl) to Tatalon estate-SS;<: =O!5<24: !o person shall .e denied e ual prote&tion A 7udi&ial .ein' is

in&luded %ithin its ter+s Those adversel) affe&ted +a) under su&h

&ir&u+stan&es invo(e the e ual prote&tion &lause onl) if the) &an sho% that

the 'overn+ental a&t assailed %as pro+pted .) the spirit of hostilit), or at

the ver) least dis&ri+ination that finds no support in reason Petitioner failed

to prove denial of e ual prote&tion O&&upants .elieve in 'f that veterans

su.division is the real o%ner Onl) %hen the pla&e vastl) i+proved %ith

.uildin' of roads, infrastru&ture did petitioner &lai+ed for the first ti+e that

the) are the o%ners

Page 36: Nat Res Digested

8/10/2019 Nat Res Digested

http://slidepdf.com/reader/full/nat-res-digested 36/173

E R !o 8 " $ 6a) */, 1991

6AR-A A2-C-A 2<;T<R-O, petitioner , vs CO;RT OF APP<A2S and 5<-RS OFG<!-TO 2<;T<R-O, respondents

FACTS: Pa.lo 2euterio died in San 2uis, Pa+pan'a on June 1#, 19#0, leavin'

a lar'e estate &onsistin' of several par&els of land in Pa+pan'a 5is %ido%,

Ana 6a'lan ue %ho had .een one of his do+esti& servants and later his

+istress, and %ho+ he had +arried a fe% +onths .efore his death, +ore

pre&isel), on Fe.ruar) *#, 19#0 too( possession of his estate and

ad+inistered itOn Jul) */, 19#$, Patro&inio Apostol, a nie&e of Pa.lo 2euterio, filed a

petition in the Court of First -nstan&e of Pa+pan'a for her appoint+ent as

'uardian of 6aria Ali&ia 2euterio, then 1" )ears of a'e, alle'ed to .e the

le'iti+ated dau'hter of said Pa.lo 2euterioOn !ove+.er *0, 19#$, Genito 2euterio, a .rother of Pa.lo 2euterio of the

full .lood, instituted pro&eedin's for the settle+ent of the de&edentIs

intestate estate in the sa+e Court of First -nstan&e of Pa+pan'a, pra)in' forhis appoint+ent as ad+inistrator Genito 2euterioIs petition pertinentl)

alle'ed that Pa.lo 2euterio had died %ithout leavin' a %illN that he %as

survived, not onl) .) said Genito 2euterio, .ut also .) a the &hildren of

<lena 2euterio, de&eased, sister of the full .lood of the de&edentN . Hi&ente

4 2euterio, the son of re'oria 2euterio, also de&eased, and also a sister of

the full .lood of Pa.lo 2euterio That Pa.lo 2euterio died a %ido%erN and that

the &lai+ of Patro&inio Apostol, a nie&e of the de&edent, that the latter hadleft a le'iti+ate dau'hter %as %ithout foundation in fa&t and in la%The petition %as opposed .) Ana 6a'lan ue and 6aria Ali&ia 2euterio the

latter .ein' represented .) the a.ove na+ed Patro&inio Apostol After

hearin', the Pro.ate Court appointed Ana 6a'lan ue ad+inistratriB of Pa.lo

2euterioIs estate

Page 37: Nat Res Digested

8/10/2019 Nat Res Digested

http://slidepdf.com/reader/full/nat-res-digested 37/173

The event leadin' dire&tl) to the appellate pro&eedin's at .ar %as the filin'

in the settle+ent pro&eedin' .) 6aria Ali&ia 2euterio on O&to.er 19, 19"* of

a pleadin' entitled "Assertion of Rights," in %hi&h she averred that she %as

the onl) for&ed heir of Pa.lo 2euterio and therefore entitled to su&&eed to the

latterIs entire estate, su.7e&t onl) to the ri'hts a&&orded .) la% to her

+other, Ana 6a'lan ue-n respe&t of this &lai+, the parties entered into a stipulation of fa&ts and

issues, as re'ards the &ele.ration and the validit) of the +arria'e of Pa.lo

2euterio and Ana 6a'lan ueN the identit) of the de&edentIs relatives .)

&onsan'uinit), supra N the &hara&ter of the de&edentIs estate as .ein' his

o%n separate, eB&lusive properties and, therefore, his &apital@

-SS;<: =O! the Pro.ate Court had erred

1 in re7e&tin' as spurious <Bhi.it 4, %hi&h is the &ertifi&ate of the re&ordof .irth of 6aria Ali&ia 2euterio in the Civil Re'istr) of San 2uis, Pa+pan'aN

* in not 'ivin' full faith and &reden&e to the testi+onies of erva&io Ga'tasand Paula Pun3alan %ho are disinterested %itnesses and %ho are s&hooltea&hers at the San 2uis <le+entar) S&hool %here appellant 6aria Ali&ia2euterio %as stud)in'N

/ in holdin' that the testi+on) of 4on Sotero Galu)ut 'iven in the for+ of a deposition appears to .e in the for+ of an a&&o++odationN

?in not de&larin' on the .asis of the eviden&e that 6aria Ali&ia 2euteriohas .een in the possession of the status of a natural &hild .efore and afterthe +arria'e of her parents

5<24: -n this &ase, the Court is not in&lined to &on&lude that there %as aneBpress desire on the part of Pa.lo to re&o'ni3e 6aria Ali&ia as his natural&hild As previousl) adverted to, the .irth &ertifi&ate, .aptis+al &ertifi&ateand the photo'raphs do not .ear the si'natures of Pa.lo eBpressin' hisa&(no%led'+ent of 6aria Ali&ia as his natural dau'hter %ith Ana6a'lan ue -ndeed, 6aria Ali&ia is said to have .een .orn, reared andraised in the house of Pa.lo Appellees eBplain this .) statin' that Ana %asa househelp in the house of Pa.lo Pa.lo has no &hild %ith his previous %ife,and it is not unusual if he loo(ed upon 6aria Ali&ia as if she %ere his o%ndau'hter in or outside his residen&e ;pon these &onsiderations, the &ourt auo %as &orre&t in re7e&tin' the testi+onies of 4ar Juan, Paula Pun3alan and

Page 38: Nat Res Digested

8/10/2019 Nat Res Digested

http://slidepdf.com/reader/full/nat-res-digested 38/173

erva&io Ga'tas, and the deposition of Sotero Galu)ut =ith respe&t to 4arJuan, Pun3alan and Ga'tas, the lo%er &ourt sa% and o.served theirde+eanor in the %itness stand and o.7e&ted to their vital &lai+s =ithrespe&t to the testi+on) of Sotero Galu)ut, petitioners ad+it that he andPa.lo %ere ver) &lose friends

=hat &lin&hes the &ase in favor of appellees, to Our +ind, is the a.solutela&( of a do&u+ent or %ritin', su&h as re&eipts of pa)+ent of s&hool fees inthe na+e of Pa.lo, si'natures in s&hool &ards, or a letter to relatives orfriends na+in' 6aria Ali&ia as his dau'hter, despite the lapse of 9 )ears fro+the .irth of 6aria Ali&ia in 19 1 up to his death in 19#0

-n her appeal to this Court, petitioner 6aria Ali&ia 2euterio su.+its that the4e&ision of the Court of Appeals should .e reversed .e&ause it %as &learand patent error on its part

1 to sur+ise that the a&tion of the petitioner for le'iti+ation is .ased onvoluntar) re&o'nition, and

* to hold that the fa&ts and the la%s involved pla&e this &ase s uarel) onall fours %ith the &ase of Colorado et al vs Court of Appeals, R !o 2/99 8, Fe.ruar) *8, 198#, althou'h the a&tion of herein petitioner is one for&o+pulsor) re&o'nition and for le'iti+ation

The petition is %ithout +erit, and &annot .e 'ranted

-t see+s to this Court that .oth the Court of Appeals and the Pro.ate Court%ere a%are of the pre&ise nature of the petitionerIs re&ourse: a 7udi&ialde&laration of her &o+pulsor) or involuntar) re&o'nition as Pa.lo 2euterioIsnatural &hild The re&ord dis&loses that the Pro.ate Court %ent to so+elen'ths to stress the distin&tion .et%een voluntar) and &o+pulsor)re&o'nition, and to +a(e petitionerIs &ounsel identif) the eBa&t &hara&ter ofthe re+ed) that she %as see(in' %hether it .e voluntar), or &o+pulsor),re&o'nition uotin' in this &onne&tion, the eB&han'e .et%een the Jud'eand petitionerIs attorne), %hi&h &ul+inated in the latterIs des&ription of thedesired relief as not voluntar) a&(no%led'+ent in the sense that thede&edent did not eBe&ute a pu.li& do&u+ent eBpressl) a&(no%led'in' thepetitioner 6aria Ali&ia 2euterio as his natural &hild Ge&ause %e .elieve thata pu.li& do&u+ent is one of the eviden&e of &o+pulsor) a&(no%led'+ent -tsaid:

There should not .e &onfusion in ter+s: one thin' is the a&(no%led'+entof a &hild .) the father, +ade voluntaril)N another is the a&tion that should.e instituted .) the &hild a'ainst the father to &o+pel the latter to

Page 39: Nat Res Digested

8/10/2019 Nat Res Digested

http://slidepdf.com/reader/full/nat-res-digested 39/173

a&(no%led'e hi+ as a natural &hild The &ontinuous possession of the statusof a natural &hild, tolerated .) his father and 7ustified .) dire&t a&ts of thelatter, does not, of itself, &onstitute eviden&e of a&(no%led'+ent that he isso in effe&t -t is, at +ost, an eviden&e to &o+pel the father to a&(no%led'ehi+ 5o%ever, the a&tion for this purpose should .e .rou'ht %ithin the

periods of ti+e pres&ri.ed in Arti&le 1/$ of the old Civil Code no% Arti&le*8# of the ne% Civil Code itt vs itt, "8 Phil /8#

The Pro.ate CourtIs state+ents &orre&tl) refle&t the state of the la% at theti+e -n fa&t, it is &onsistent %ith the state+ent of the la% atte+pted .)petitionerIs o%n distin'uished &ounsel, &itin' Concepcion vs. Untaran , /8Phil , $/$, $/8, viz :

The father of a natural &hild +a) re&o'ni3e it in t%o different %a)s: a .) avoluntar) re&o'nition Art, 1/1, &iv &ode N * .) an involuntar) re&o'nitionenfor&ed .) either a &ivil or &ri+inal a&tion Art 1/#, Civ CodeN Art 99,Pen Code

A voluntar) re&o'nition of a natural &hild +a) .e +ade: a in the re&ord of.irthsN . .) %illN and & .) an) other pu.li& instru+ent Art 1/1, CivilCode

An involuntar) re&o'nition of a natural &hild is +ade: a .) anin&ontroverti.le paper %ritten .) the parent eBpressl) re&o'ni3in' hispaternit)N . .) 'ivin' su&h &hild the status of a natural &hild of the father,

7ustified .) dire&t a&t of the &hild of the father or his fa+il) art 1/#, Civ

Code N and & .) a &ri+inal a&tion for rape, sedu&tion or a.du&tion par *,art 9, Pen Code

-t %as in this sense, too, that the Court of Appeals appeared to haveunderstood and applied the la% to the &ase As +u&h is apparent fro+ itsde&laration that r e&o'nition under the Civil Code of 1889 M+ust .e pre&ise,eBpress and sole+n 2i+ vs Court of Appeals, "# SCRA 1"1 , %hethervoluntar) or &o+pulsor) Garon vs Garon, "/ O !o *, Jan 9, 19"$ 2i(ethe Pro.ate Court, %hose 7ud'+ent it affir+ed, the Court of Appeals ruledthat the eviden&e failed to prove either the eBisten&e of an in&ontroverti.lepaper %ritten .) the parent eBpressl) re&o'ni3in' his paternit), or the'ivin' to su&h &hild of the status of a natural &hild of the father&onfor+a.l) %ith Arti&le 1/# of the Civil Code of 1889 5en&e, there %as nofa&tual .asis on %hi&h to rest a de&laration of involuntar) re&o'nition .)Pa.lo 2euterio of 6aria Ali&ia as his natural dau'hter

!o%, the findin's of fa&t of the Court of Appeals are, .) fa+iliar do&trine,&on&lusive on this Court and are not thus su.7e&t of revie%, spe&iall) %here

Page 40: Nat Res Digested

8/10/2019 Nat Res Digested

http://slidepdf.com/reader/full/nat-res-digested 40/173

those findin's are the sa+e as those +ade .) the Trial Court There are, of&ourse, eB&eptions to this rule, .ut none o.tains in the &ase at .ar

The petitioner also &ontests the Appellate CourtIs holdin' that Arti&le *8/ ofthe present 19#0 Civil Code has no retroa&tive effe&t That &on&lusion %as

no dou.t .ased on the fa&t that Arti&le **"0 of the sa+e Code eBpressl)a&&ords su&h effe&t onl) to voluntar) re&o'nition thus .) inferen&e eB&ludin'&o+pulsor) re&o'nition for the &auses or under the &ir&u+stan&esenu+erated in Arti&le *8/, %ith its &at&h all provision that re&o'nition +a).e &o+pelled if the &hild has in his favor an) eviden&e or proof that thedefendant is his father =hile a &ontrar) vie%, i e , in favor of retroa&tivit),+a) find support in the eB&eptin' &lause of Arti&le **#/, also of the CivilCode, %hi&h 'ives effe&t to ri'hts de&lared for the first ti+e therein, thou'harisin' fro+ a&ts done or events o&&urred under prior la% provided no vestedor a& uired ri'hts of the sa+e ori'in are pre7udi&ed there.), there is littlepoint in pursuin' that uestion insofar as the resolution of this appeal is&on&erned

=hether Arti&le *8/ has retroa&tive effe&t or it operates onl) prospe&tivel),the fa&t is that .oth the Pro.ate Court and the Court of Appeals re7e&ted inits entiret) as variousl), insuffi&ient, unpersuasive and spurious petitionerIs eviden&e .oth oral and do&u+entar) .earin' on her alle'edstatus as a natural &hild of Pa.lo 2euterio That re7e&tion fore&loses the&lai+ of petitioner to either voluntar) or &o+pulsor) re&o'nition, .e it +adeunder the Civil Code of 1889 %hi&h %as in for&e at the ti+e of her asserted.irth or, in the &ase of &o+pulsor) re&o'nition, under the +ore li.eral Arti&le

*8/ of the present Code -t &an hardl) .e disputed that in openin' the doorto an) eviden&e of paternit) in an a&tion to &o+pel a&(no%led'+ent,Arti&le *8/ .) no +eans did a%a) %ith the usual tests of &o+peten&e,suffi&ien&) and &redi.ilit) to %hi&h su&h eviden&e is su.7e&t %hen offered in a&ourt of la%, or strip the &ourts of their fun&tion and prero'ative of passin'upon its a&&epta.ilit) after appl)in' su&h tests Su&h eviden&e here havin'.een found %antin' after due assess+ent as alread) stated, petitionerIs&lai+ %as properl) denied

Page 41: Nat Res Digested

8/10/2019 Nat Res Digested

http://slidepdf.com/reader/full/nat-res-digested 41/173

E R !o 109 90 Fe.ruar) 1 , 199

PATROC-!-O < 6AR O22<S, H-R -!-A < H-22O! CO, <4;AR4O C<SP-!OSA, 2;C-A < 2AP<RA2, !OR6A C <SP-!OSA, T<R<S-TA < CASA2,A2-C< < SOTTO, petitioners, vs 5O! CO;RT OF APP<A2S, F-R<STO!<C<RA6-CS, -!C , GOO6TO=! 4<H<2OP6<!T CORPORAT-O!, SPO;S<SC>!T5-A 4 C5-! and C5-! T-O! Q<! , SPO;S<S CAR6<! SOCO and2OR<! O O! <! C5O! , SPO;S<S SO2<4A4 G >; and >; S> C5-A,and 2<T-C-A !OCO! C5A!, respondents.

On 11 Jul) 198#, Firestone, Goo+to%n, spouses C)nthia 4 Chin' and Chin'Tion' Qen', spouses Car+en So&o and 2oren3o On' <n' Chon', spousesSoledad >u and >u S) Chia, and 2eti&ia !o&on Chan filed %ith the Re'ional

Trial Court, Gran&h #8, 6a(ati, 6etro 6anila, a &o+plaint for annul+ent oftitles, re&over) of possession, and uietin' of titles a'ainst Patro&inio <6ar'olles, Hir'inia < Hillon'&o, <d'ardo C <spinosa, 2u&ia < 2aperal,!or+a < <spinosa, Teresita < Casal, Ali&e < Sotto, Heroni&a ana, and< uita.le Gan(in' Corporation Also in&luded a+on' the defendants %erethe 2and Re'istration Co++issioner and the Re'ister of 4eeds of Pasa) Cit)

The &o+plaint averred that the par&els of land in uestion %ere re'istered inthe na+es of Genito on3ales and <+eterio <spiritu %as issued pursuant toa de&ision in 2and Re'istration Case !o ! ""*#, dated ** Jul) 19"9 On 0Fe.ruar) 19$", the propert) %as su.divided .) on3ales and <spiritu intofive lots, resultin' in the issuan&e of five TCT

6onths after plaintiffs too( possession of the pre+ises, the defendantsde+anded that the plaintiffs va&ate the pre+ises Clai+in' o%nership, thedefendants, on their part, tra&ed their titles fro+ Ori'inal Certifi&ate of Title!o *1" issued to the spouses 2oren3o ana and 6a Juliana Carlos on *"6ar&h 19*9 pursuant to 4e&ree !o /#18/ in 2and Re'istration Case 2RC!o "$* of the Court of First -nstan&e of Ri3al, 2 R O Re&ord !o /0 0"

On 0 April 19#", OCT !o *1" %as &an&elled and, in its pla&e, TCT !o/### %as issued to 2oren3o ana and Heroni&a ana +arried to Ra+onRodri'ue3 TCT !o /### %as itself li(e%ise &an&elled on the sa+e da)and TCT !o /##" %as issued, this ti+e in the na+e of Heroni&a anaalone On 1/ Au'ust 19#", Heroni&a ana sold the land to Patro&inio6ar'olles, resultin' in the issuan&e of TCT !o "/0*

Page 42: Nat Res Digested

8/10/2019 Nat Res Digested

http://slidepdf.com/reader/full/nat-res-digested 42/173

Page 43: Nat Res Digested

8/10/2019 Nat Res Digested

http://slidepdf.com/reader/full/nat-res-digested 43/173

To support their &lai+ that OCT !o *1" is 'enuine, the petitioners havesu.+itted, a+on' other thin's, the follo%in' pie&es of do&u+entar)eviden&e:

1 The ori'inal of OCT !o *1", as %ell as the o%ners dupli&ate

&ertifi&ates, on file %ith the Offi&e of the Re'ister of 4eeds of Ri3alN

* The pu.li&ations in the <n'lish and Spanish versions of the Offi&iala3ette 19*$ editions , &ontainin' noti&es of the initial hearin' in 2andRe'istration Case !o "$* 2RO Re&ord !o /0 0" , instituted .) thespouses 2oren3o ana and 6aria Juliana Carlos, &overin' a par&el of land inTindi' na 6an''a, 2as PiUasN

/ The order of then CF- Jud'e Ce&ilia 6uUo3 Pal+a, dated */ 6ar&h 19"1,in 2RC Case !o ! *1*" 2RO Re&ord !o ! "#" , den)in' the re'istrationof a par&el of land .) reason of the &ertifi&ation, dated *" June 19#9, of the2and Re'istration Co++issioner, Antonio ! !o.le7as, that a portion of thepropert) &overed in this post %ar land &ase had .een de&reed under 4e&ree!o /#18*/, issued on 0# 6ar&h 19*9, in the na+e of the spouses 2oren3oana and 6aria Juliana A Carlos in 2RC Case !o "$* 2RO Re&ord !o/0 0" , and %hile said &ase &overed onl) a part of the propert) in dispute, itdid sho%, ho%ever, that the de&ree %as, in fa&t, issued to the spouses anaand CarlosN

The Report, dated 0$ June 198/, of the 2and Re'istration Co++issionIsHerifi&ation Co++ittee, sustainin' the validit) of 4e&ree !o /#18*/ in favor

of 2oren3o J ana and 6aria Juliana A CarlosN

# Pa'e *09 of the Goo( of 4e&rees Old Goo( of the 2and Re'istrationCo++ission, sho%in' that a de&ree %as o(a)ed in 2RO Re&ord !o /0 0"2RC Case !o "$* , under the entr) 4ate O Q for 4e&ree on 1 ** *9** Januar) 19*9 and that a de&ree %as issued under the entr) 4ate4e&ree -ssued on / # *9 0# 6ar&h 19*9 N

" The &ertified true +i&rofil+ reprodu&tion of plan Psu 9*$/ &overin' apar&el of land in Garrio Tindi' na 6an''a, 2as PiUas, surve)ed for 2oren3oana and 6aria Juliana Carlos, approved .) the Gureau of 2ands in 19*"N

$ The de&ision of this Court in ui&o vs San Pedro, $* Phil 1#, pointin'to the de&ision rendered .) the Court of First -nstan&e of Ri3al in 2RC Case!o "$* in favor of the spouses 2oren3o ana and 6aria Juliana CarlosN and

8 The letters of Soli&itor eneral <stelito 6endo3a and Soli&itor eneralFran&is&o Chave3, statin' that the infor+ation and do&u+ents su.+itted to

Page 44: Nat Res Digested

8/10/2019 Nat Res Digested

http://slidepdf.com/reader/full/nat-res-digested 44/173

the Offi&e of the Soli&itor eneral .) the Gureau of 2ands and the 2andRe'istration Co++ission %ere not suffi&ient to support an a&tion for&an&ellation of OCT !o *1" and the derivative titles thereof

5<24: The a.ove do&u+entar) eviden&e is +u&h too over%hel+in' to .e

si+pl) .rushed aside -t is our &onsidered vie% that the appellate &ourt has&o++itted serious error in refusin' to 'ive an) pro.ative value to su&heviden&e All that the private respondents &ould .asi&all) proffer a'ainstOCT *1" are that

1 The title is invalid, fa(e and spurious, %hi&h +ust have .een the %or( ofso+e uns&rupulous ele+ents %ho &ould have a&&ess to the Re'istr) Goo(of the Offi&e of the Re'ister of 4eeds of the Provin&e of Ri3al, that eBplainspetitionersI failure to present a &op) of the de&ision in 2and Re'istrationCase !o "$* or 4e&ree !o /#18*/N and

* Assu+in' OCT !o *1" to have .een issued, the sa+e is invalid havin'.een issued on still un&lassified land of the pu.li& do+ain

Se&tion /, Rule 1/0, of the Revised Rules of Court, ta(en fro+ Se&tion /*1of A&t !o 190, states:

Se& / Ori'inal do&u+ent +ust .e produ&edN eB&eptions =hen thesu.7e&t of in uir) is the &ontents of a do&u+ent, no eviden&e shall .ead+issi.le other than the ori'inal do&u+ent itself, eB&ept in the follo%in'&ases:

? a =hen the ori'inal has .een lost or destro)ed, or &annot .eprodu&ed in &ourt, %ithout .ad faith on the part of the offerorN

? . =hen the ori'inal is in the &ustod) or under the &ontrol of thepart) a'ainst %ho+ the eviden&e is offered, and the latter fails toprodu&e it after reasona.le noti&eN

? & =hen the ori'inal &onsists of nu+erous a&&ounts or otherdo&u+ents %hi&h &annot .e eBa+ined in &ourt %ithout 'reat lossof ti+e and the fa&t sou'ht to .e esta.lished fro+ the+ is onl)the 'eneral result of the %holeN and

? d =hen the ori'inal is a pu.li& re&ord in the &ustod) of a pu.li&offi&er or is re&orded in a pu.li& offi&e

-t has .een plainl) sho%n that the failure of the petitioners to produ&e the4e&ree is due to the .urnin' of the Ar&hives of the Court of First -nstan&e of

Page 45: Nat Res Digested

8/10/2019 Nat Res Digested

http://slidepdf.com/reader/full/nat-res-digested 45/173

Ri3al durin' the li.eration of Pasi', in &onse uen&e of %hi&h all pre %ar landre'istration &ases in Ri3al have .een destro)ed The respondents o%n%itness, <duardo Santos, Jr , has testified that the re&ords of pre %arre'istration &ases are thus in&o+plete as &an .e eBpe&ted The Certifi&ation,dated 0* 6a) 1980, of Re)naldo S Her'ara, A&tin' Chief of the 4o&(et

4ivision of the 2and Re'istration Authorit), states that the pre %ar re&ord of2RC Case !o "$*, 2RO Re&ord !o 0/0 0" for the provin&e of Ri3al, is nota+on' the re&ords on file %ith the Hault Se&tion of the 4o&(et 4ivision sin&ethe sa+e +ust have .een lost or destro)ed as a &onse uen&e of the last%orld %ar Certainl), the petitioners &annot .e held to a&&ount for those lostor destro)ed re&ords

The private respondents ar'ue that the petitioners should have as(edfor the re&onstitution of the 2RC &ase and the de&ree in a&&ordan&e %ith A&t!o /110 and Repu.li& A&t !o *", or that the) &ould have opposed, orintervened in, the pro&eedin's in 2RC Case !o ! ""*# 2RC Re&ord !o !/"#$9 %here OCT !o A S $ has .een de&reed For failin' to do so, thepetitioners, it is no% &ontended, should .e held .ound .) the order ofdefault issued .) the land re'istration &ourt The ar'u+ent isuna&&epta.le The petitioners are not &overed .) the 'eneral order ofdefault in 2RC Case !o ! ""*# Repu.li& A&t !o *" onl) &overs lost ordestro)ed &ertifi&ates of title The ori'inal of OCT !o *1" is not eBtantN ithas, in fa&t, .een presented in eviden&e A&t !o /110, on the other hand,applies onl) to pendin' 7udi&ial pro&eedin's This Court has heretofore held,thus

The %hole theor) of re&onstitution is to reprodu&e or repla&e re&ords lost ordestro)ed so that said re&ords +a) .e &o+plete and &ourt pro&eedin's +a)&ontinue fro+ the point or sta'e %here said pro&eedin's stopped due to theloss of the re&ords B B B

BBB BBB BBB

-f the re&ords up to a &ertain point or sta'e are lost and the) are notre&onstituted, the parties and the &ourt should 'o .a&( to the neBt pre&edin'sta'e %here re&ords are availa.le, .ut not .e)ond thatN other%ise to i'noreand 'o .e)ond the sta'e neBt pre&edin' %ould .e voidin' and unne&essaril)i'norin' pro&eedin's %hi&h are dul) re&orded and do&u+ented, to the 'reatpre7udi&e not onl) of the parties and their %itnesses, .ut also of the &ourt%hi&h +ust 'ain perfor&e ad+it pleadin's, rule upon the+ and then tr) the&ase and de&ide it ane%, all of these, %hen the re&ords up to said point orsta'e are inta&t and &o+plete, and un&ontroverted

Page 46: Nat Res Digested

8/10/2019 Nat Res Digested

http://slidepdf.com/reader/full/nat-res-digested 46/173

B B B A&t !o /110, %as not pro+ul'ated to penali3e people for failure too.serve or invo(e its provisions -t &ontains no penal san&tion -t %asena&ted rather to aid and .enefit liti'ants, so that %hen &ourt re&ords aredestro)ed at an) sta'e of 7udi&ial pro&eedin's, instead of institutin' a ne%&ase and startin' all over a'ain, the) +a) re&onstitute the re&ords lost and

&ontinue the &ase -f the) fail to as( for re&onstitution, the %orst that &anhappen to the+ is that the) lose the advanta'es provided .) there&onstitution la% B B B

B B B T o re uire the parties to file their a&tion ane% and in&ur theeBpenses and suf f er the anno)an&e and veBation in&ident to the filin' ofpleadin's and the &ondu&t of hearin's, aside fro+ the possi.ilit) that so+eof the %itnesses +a) have died or left the 7urisdi&tion, and also to re uirethe &ourt to a'ain rule on the pleadin's and hear the %itnesses and thende&ide the &ase, %hen all alon' and all the ti+e the re&ord of the for+erpleadin's of the trial and eviden&e and de&ision are there and are notdisputed, all this %ould appear to .e not eBa&tl) lo'i&al or reasona.le, or fairand 7ust to the parties, in&ludin' the trial &ourt %hi&h has not &o++itted an)ne'li'en&e or fault at all @

Further+ore, Se&tion # of A&t !o /110, provides that n othin' &ontainedin the A&t shall .e &onstrued to repeal or +odif) the provisions of Se&tionThree 5undred and T%ent) One of A&t !u+.ered One 5undred and !inet)Se&tion /*1 of A&t !o 190 is no% Se&tion / afore uoted , Rule 1/0, of theRevised Rules of Court, other%ise (no%n as the ?.est eviden&e rule 5en&e,even if the petitioners have failed to have the re&ords of the 2RC &ase

re&onstituted, the) are not pre&luded fro+ esta.lishin' .) other eviden&ethe re uisite proof of validit) of OCT !o *1"

uite re&entl), in =ido%s and Orphans Asso&iation, -n& =-4ORA vs Courtof Appeals, this Court, spea(in' throu'h 6r Justi&e Florentino Feli&iano,said:

B B B . The &op) of OCT !o /#1 offered .) Orti'as %as a &ertified true &op)of the ori'inal thereof found in the Re'istration Goo( of the Re'ister of4eeds of Ri3al The ad+issi.ilit) of su&h a &op) in &ourt pro&eedin's is aneB&eption to the ordinar) rule on se&ondar) eviden&eN su&h ad+issi.ilit) is infa&t +andated .) Se&tion $ of A&t !o 9" The 2and Re'istration A&t;nder the 2and Re'istration A&t %hi&h %as in for&e at the ti+e OCT !o /#1issued, the ori'inal thereof found in the Re'istration Goo( of the Re'ister of4eeds of Ri3al %as an official transcript of Decree No. 14 !, ith respect tothe land covered #$ such decree situated in the %rovince of Rizal

Page 47: Nat Res Digested

8/10/2019 Nat Res Digested

http://slidepdf.com/reader/full/nat-res-digested 47/173

Page 48: Nat Res Digested

8/10/2019 Nat Res Digested

http://slidepdf.com/reader/full/nat-res-digested 48/173

;nfortunatel), for all &on&erned, no authenti& &op) of 2C 6ap !o $"",Pro7e&t 1/, &ould .e presented, al.eit understanda.l), &onsiderin' that eventhe re&ords of the !ational 6appin' and Resour&e Authorit) !A6R<A haveapparentl) .een lost or destro)ed durin' the se&ond =orld =ar

-n Sta 6oni&a -ndustrial and 4evelop+ent Corporation vs Court ofAppeals a &ase to annul a 191* de&ision of the land re'istration &ourt , theRepu.li& sou'ht to prove that, at the ti+e an ori'inal &ertifi&ate of title %asissued, the land &overed there.) %as still %ithin the forest 3one -t offeredas eviden&e a land &lassifi&ation +ap prepared .) the 4ire&tor of Forestr) in19"1 The Court ruled:

?B B B =hen the pro&eedin's %ere ori'inall) filed .) the Repu.li& .eforethe Court of Appeals, the petitioner &ontended that %hen the de&ree in favorof 4e Perio %as issued .) Jud'e Ostrand in 191* the par&els of land %erestill part of the inaliena.le pu.li& forests 5o%ever, petitionerIs &ase restedsolel) on land &lassifi&ation +aps dra%n several )ears after the issuan&e ofthe de&ree in 191* These +aps failed to &on&lusivel) esta.lish the a&tual&lassifi&ation of the land in 191* and the )ears prior to that Gefore thisCourt, petitioner reiterates said &ontention and refers, for the first ti+e, to a1908 pro&la+ation reservin' the land in a+.ales as a naval reservationand alle'in' that the su.7e&t par&els of land are parts thereof These B B Bare insuffi&ient to over&o+e the le'al presu+ption in favor of the de&reeIsre'ularit) B B B

Further+ore, FAO !o 11 1, si'ned .) then Se&retar) of A'ri&ulture and

!atural Resour&es Arturo R Tan&o, Jr , on 0/ Januar) 19"8, provides:

1 Pursuant to the provisions of Se&tion 18*$ of the RevisedAd+inistrative Code, - here.) de&lare as aliena.le or disposa.leand pla&e the sa+e under the &ontrol of the Gureau of 2ands forad+inistration and disposition in a&&ordan&e %ith the Pu.li& 2andA&t, su.7e&t to private ri'hts, if an) there .e and to the &onditionsherein spe&ified, the portions of the pu.li& do+ain situated in the6uni&ipalities of B B B 2as PiUas, B B B Provin&e of Ri3al B B B%hi&h are desi'nated and des&ri.ed as aliena.le or disposa.le onGureau of Forestr) 6ap 2C *"*/, approved on Januar) /, 19"8;nders&orin' supplied

The issuan&e of OCT !o *1" in 19*9, &onferrin' a private ri'ht, is thena+pl) prote&ted .) FAO !o 11 1N other%ise, &ertifi&ates of title issuedprior to 19"8 &ould possi.l) .e all nullified

Page 49: Nat Res Digested

8/10/2019 Nat Res Digested

http://slidepdf.com/reader/full/nat-res-digested 49/173

Finall), the private respondents raise estoppel .) la&hes on the part of thepetitioners 2a&hes is the failure or ne'le&t for an unreasona.le anduneBplained len'th of ti+e, to do that %hi&h .) eBer&isin' due dili'en&e&ould or should have .een done earlier, or the ne'li'en&e or o+ission toassert a ri'ht %ithin a reasona.le ti+e, %arrantin' a presu+ption that the

part) entitled to assert it either has a.andoned it or has de&lined to assertit @

Contrar) to private respondents &lai+ that no a&tion %as ta(en .) thepetitioners until a petition for uietin' of title %as filed in 198# .) the privaterespondents the+selves, the re&ords %ould indi&ate that upon thesu.division of the lots in uestion .) <spiritu and on3ales, and thesu.se uent transfers of the sa+e to the private respondents in 19$", ade+and %as seasona.l) +ade .) the petitioners for the private respondentsto va&ate the pre+ises Fro+ the ti+e OCT !o A S $ %as issued to theprivate respondents in 19"9 until the de+and %as +ade in 19$", onl) seven$ )ears had elapsed

2astl), it is a settled rule that %hen t%o &ertifi&ates of title are issued todifferent persons &overin' the sa+e land in %hole or in part, the earlier indate +ust prevail, and, in &ase of su&&essive re'istrations %here +ore thanone &ertifi&ate is issued over the land, the person holdin' a prior &ertifi&ateis entitled to the land as a'ainst a person %ho relies on a su.se uent&ertifi&ate The titles of the petitioners, havin' e+anated fro+ an older title,should thus .e upheld

Page 50: Nat Res Digested

8/10/2019 Nat Res Digested

http://slidepdf.com/reader/full/nat-res-digested 50/173

Page 51: Nat Res Digested

8/10/2019 Nat Res Digested

http://slidepdf.com/reader/full/nat-res-digested 51/173

S*-be'm "o- e-/e-ce Foo ), %-c. ). "A

G.R. No. 0464, '-. 29, 1990

FA"TS&

Sun.ea+ Convenien&e Foods, -n& is the re&ipient of a Sales Patent issued

.) the Gureau of 2ands over t%o par&els of land in Gataan An OCT %as

there.) issued The Soli&itor eneral filed an a&tion for reversion on the

'round that the lots %ere forest lands and therefore inaliena.le

CA ruled, upholdin' the Soli&itor eneralIs &ontention

%SSUE&

• =hether or not land is aliena.le

HELD&

The SC affir+ed

Our adheren&e to the Re'alian 4o&trine su.7e&ts all a'ri&ultural, ti+.er, and

+ineral lands to the do+inion of the State Thus, .efore an) land +a) .e

de&lassified fro+ the forest 'roup and &onverted into aliena.le or disposa.le

land for a'ri&ultural purposes, there +ust .e a positive a&t fro+ theovern+ent <ven rules on the &onfir+ation of i+perfe&t titles do not appl)

unless and until the land &lassified as forest land is released in an offi&ial

pro&la+ation to that effe&t so that it +a) for+ part of the disposa.le

a'ri&ultural lands of the pu.li& do+ain

Page 52: Nat Res Digested

8/10/2019 Nat Res Digested

http://slidepdf.com/reader/full/nat-res-digested 52/173

The +ere fa&t that a title %as issued .) the 4ire&tor of 2ands does not

&onfer an) validit) on su&h title if the propert) &overed .) the title or patent

is part of the pu.li& forest

Page 53: Nat Res Digested

8/10/2019 Nat Res Digested

http://slidepdf.com/reader/full/nat-res-digested 53/173

2and Titles and 4eeds Case 4i'est: 4ire&tor of 2ands v -AC 198"

2a.els: 198", Case 4i'est, Juris 4o&tor, 2and Titles and 4eeds, 2and Titles

and 4eeds Case 4i'estR !o $/00* 4e&e+.er *9, 198"

2essons Appli&a.le: Se& / Art --, 198$ Constitution 2and Titles and

4eeds

FACTS:A&+e Pl)%ood K Heneer Co , -n& , a &orp represented .) 6r Rodolfo

!a3ario, a& uired fro+ 6ariano and A&er -nfiel, +e+.ers of the 4u+a'at

tri.e # par&els of landpossession of the -nfiels over the landdates .a&( .efore the Philippines %as

dis&overed .) 6a'ellanland sou'ht to .e re'istered is a private land pursuant to RA /8$* 'rantin'

a.solute o%nership to +e+.ers of the non Christian Tri.es on land o&&upied

.) the+ or their an&estral lands, %hether %ith the aliena.le or disposa.le

pu.li& land or %ithin the pu.li& do+ainA&+e Pl)%ood K Heneer Co -n& , has introdu&ed +ore than P #6 %orth of

i+prove+ents

o%nership and possession of the land sou'ht to .e re'istered %as dul)re&o'ni3ed .) the 'overn+ent %hen the 6uni&ipal Offi&ials of 6a&ona&on,

-sa.eladonated part of the land as the to%nsite of 6a&ona&on -sa.ela-AC affir+ed CF-: in favor of-SS;<S:=L! the land is alread) a private land ><S=L! the &onstitutional prohi.ition a'ainst their a& uisition .) private

&orporations or asso&iations applies !O

5<24: -AC affir+ed A&+e Pl)%ood K Heneer Co , -n&><Salread) a& uired, .) operation of la% not onl) a ri'ht to a 'rant, .ut a 'rant

of the overn+ent, for it is not ne&essar) that a &ertifi&ate of title should .e

issued in order that said 'rant +a) .e san&tioned .) the &ourts, an

appli&ation therefore is suffi&ient

Page 54: Nat Res Digested

8/10/2019 Nat Res Digested

http://slidepdf.com/reader/full/nat-res-digested 54/173

it had alread) &eased to .e of the pu.li& do+ain and had .e&o+e private

propert), at least .) presu+ptionThe appli&ation for &onfir+ation is +ere for+alit), the la&( of %hi&h does not

affe&t the le'al suffi&ien&) of the title as %ould .e eviden&ed .) the patent

and the Torrens title to .e issued upon the stren'th of said patentThe effe&t of the proof, %herever +ade, %as not to &onfer title, .ut si+pl) to

esta.lish it, as alread) &onferred .) the de&ree, if not .) earlier la% * !O-f it is a&&epted as it +ust .e that the land %as alread) private land to %hi&h

the -nfiels had a le'all) suffi&ient and transfera.le title on O&to.er *9, 19"*

%hen A&+e a& uired it fro+ said o%ners, it +ust also .e &on&eded that

A&+e had a perfe&t ri'ht to +a(e su&h a& uisition

The onl) li+itation then eBtant %as that &orporations &ould not a& uire, holdor lease pu.li& a'ri&ultural lands in eB&ess of 1,0* he&tares

Page 55: Nat Res Digested

8/10/2019 Nat Res Digested

http://slidepdf.com/reader/full/nat-res-digested 55/173

Restituto >not vs -nter+ediate Appellate Court

There had .een an eBistin' la% %hi&h prohi.ited the slau'hterin' of &ara.aos <O "*" To stren'then the la%, 6ar&os issued <O "*" A %hi&h

not onl) .anned the +ove+ent of &ara.aos fro+ interprovin&es .ut as %ell

as the +ove+ent of &ara.eef On 1/ Jan 198 , >not %as &au'ht transportin'

" &ara.aos fro+ 6as.ate to -loilo 5e %as then &har'ed in violation of <O

"*" A >not averred <O "*" A as un&onstitutional for it violated his ri'ht to

.e heard or his ri'ht to due pro&ess 5e said that the authorit) provided .)

<O "*" A to outri'htl) &onfis&ate &ara.aos even %ithout .ein' heard isun&onstitutional The lo%er &ourt ruled a'ainst >not rulin' that the <O is a

valid eBer&ise of poli&e po%er in order to pro+ote 'eneral %elfare so as to

&ur. do%n the indis&ri+inate slau'hter of &ara.aos%SSUE& =hether or not the la% is validHELD& The SC ruled that the <O is not valid as it indeed violates due

pro&ess <O "*" A &treated a presu+ption .ased on the 7ud'+ent of

the eBe&utive The +ove+ent of &ara.aos fro+ one area to the other does

not +ean a su.se uent slau'hter of the sa+e %ould ensue >not should .e

'iven to defend hi+self and eBplain %h) the &ara.aos are .ein' transferred

.efore the) &an .e &onfis&ated The SC found that the &hallen'ed +easure

is an invalid eBer&ise of the poli&e po%er .e&ause the +ethod e+plo)ed to

&onserve the &ara.aos is not reasona.l) ne&essar) to the purpose of the la%

and, %orse, is undul) oppressive 4ue pro&ess is violated .e&ause the o%ner

of the propert) &onfis&ated is denied the ri'ht to .e heard in his defense and

is i++ediatel) &onde+ned and punished The &onfer+ent on

the ad+inistrative authorities of the po%er to ad7ud'e the 'uilt of the

supposed offender is a &lear en&roa&h+ent on 7udi&ial fun&tions and +ilitates

a'ainst the do&trine of separation of po%ers There is, finall), also an invalid

dele'ation of le'islative po%ers to the offi&ers +entioned therein %ho are

Page 56: Nat Res Digested

8/10/2019 Nat Res Digested

http://slidepdf.com/reader/full/nat-res-digested 56/173

'ranted unli+ited dis&retion in the distri.ution of the properties ar.itraril)

ta(en

Page 57: Nat Res Digested

8/10/2019 Nat Res Digested

http://slidepdf.com/reader/full/nat-res-digested 57/173

#A 3S. $"GG [22 S"RA 68 G.R. No. 96 41 24 A*5 199 ]

Frida), Januar) /0, *009 Posted .) Coffeeholi& =rites

2a.els: Case 4i'ests , Politi&al 2a%

F'c()& On 9 Au'ust 1990, 6ateo A T Caparas, then Chair+an of PC ,

%rote then President Cora3on C A uino, re uestin' her for authorit) to si'n

the proposed Consi'n+ent A'ree+ent .et%een the Repu.li& of the

Philippines throu'h PC and Christie, 6anson and=oods -nternational, -n&

&on&ernin' the s&heduled sale on 11 Januar) 1991 of ei'ht) t%o Old

6asters Paintin's and anti ue silver%are sei3ed fro+ 6ala&aUan' and the6etropolitan 6useu+ of 6anilaalle'ed to .e part of the ill 'otten %ealth of

the late President 6ar&os, his relatives and &ronies On 1 Au'ust 1990,

then President A uino, throu'h for+er <Be&utive Se&retar) Catalino

6a&arai', Jr, authori3ed Chair+an Caparas to si'n

the Consi'n+ent A'ree+ent allo%in' ChristieIs of !e% >or( to au&tion off

the su.7e&t art pie&es for and in .ehalf of the Repu.li& of the Philippines On

1# Au'ust 1990, PC , throu'h Chair+an Caparas, representin' theovern+ent of the Repu.li& of the Philippines, si'ned

the Consi'n+ent A'ree+ent %ith ChristieIs of !e% >or( A&&ordin' to the

a'ree+ent, PC shall &onsi'n to C5R-ST-<IS for sale at pu.li& au&tion the

ei'ht) t%o Old 6asters Paintin's then found at the 6etropolitan 6useu+

of 6anila as %ell as the silver%are &ontained in sevent) one &artons in the

&ustod) of the Central Gan( of the Philippines, and su&h other propert) as

+a) su.se uentl) .e identified .) PC and a&&epted .) C5R-ST-<IS to .esu.7e&t to the provisions of the a'ree+ent

On *" O&to.er 1990, the Co++ission on Audit throu'h then Chair+an

<ufe+io C 4o+in'o su.+itted to President A uino the audit findin's and

Page 58: Nat Res Digested

8/10/2019 Nat Res Digested

http://slidepdf.com/reader/full/nat-res-digested 58/173

o.servations of COA on the Consi'n+ent A'ree+ent of 1# Au'ust 1990 to

the effe&t that: the authorit) of for+er PC Chair+an Caparas to enter into

the Consi'n+ent A'ree+ent %as of dou.tful le'alit)N the &ontra&t %as hi'hl)

disadvanta'eous to the 'overn+entN PC had a poor tra&( re&ord in asset

disposal .) au&tion in the ; S N and, the assets su.7e&t of au&tion %ere

histori&al reli&s and had &ultural si'nifi&an&e, hen&e, their disposal %as

prohi.ited .) la%

After the oral ar'u+ents of the parties on 9 Januar) 1991, %e issued

i++ediatel) our resolution den)in' the appli&ation for preli+inar) in7un&tion

to restrain the s&heduled sale of the art%or(s on the 'round that petitioners

had not presented a &lear le'al ri'ht to a restrainin' order and that proper

parties had not .een i+pleaded

On 11 Januar) 1991, the sale at pu.li& au&tion pro&eeded as s&heduled and

the pro&eeds of V1/,/0*,"0 8" %ere turned over to the Gureau of

Treasur)

%))*e)&

1 =hether or not petitioners have le'al standin'

* =hether or not the Old 6asters Paintin's and anti ue silver%are are

e+.ra&ed in the phrase &ultural treasure of the nation

/ =hether or not the paintin's and silver%are are properties of pu.li&

do+inion on %hi&h &an .e disposed of throu'h the 7oint &on&urren&e of the

President and Con'ress

Page 59: Nat Res Digested

8/10/2019 Nat Res Digested

http://slidepdf.com/reader/full/nat-res-digested 59/173

=hether or not PC has &o+plied %ith the due pro&ess &lause and

other statutor) re uire+ents for the eBportation and sale of the su.7e&t

ite+s

# =hether or not the petition has .e&o+e +oot and a&ade+i&, and if so,

%hether the a.ove -ssue %arrant resolution fro+ this Court

He+ & This is pre+ised on Se& *, Rule /, of the Rules of Court %hi&h

provides that ever) a&tion +ust .e prose&uted and defended in the na+e of

the real part) in interest, and that all persons havin' interest in the su.7e&t

of the a&tion and in o.tainin' the relief de+anded shall .e 7oined as

plaintiffs The Court %ill eBer&ise its po%er of 7udi&ial revie% onl) if the &ase

is .rou'ht .efore it .) a part) %ho has the le'al standin' to raise

the &onstitutional or le'al uestion 2e'al standin' +eans a personal and

su.stantial interest in the &ase su&h that the part) has sustained or %ill

sustain dire&t in7ur) as a result of the 'overn+ental a&t that is .ein'&hallen'ed The ter+ interest is +aterial interest, an interest in issue and

to .e affe&ted .) the de&ree, as distin'uished fro+ +ere interest in the

uestion involved, or a +ere in&idental interest 6oreover, the interest of the

part) plaintiff +ust .e personal and not one .ased on a desire to vindi&ate

the &onstitutional ri'ht of so+e third and related part)

There are &ertain instan&es ho%ever %hen this Court has allo%ed eB&eptionsto the rule on le'al standin', as %hen a &iti3en .rin's a &ase for +anda+us

to pro&ure the enfor&e+ent of a pu.li& dut) for the fulfill+ent of a pu.li&

ri'ht re&o'ni3ed .) the Constitution, and %hen a taBpa)er uestions the

validit) of a 'overn+ental a&t authori3in' the dis.urse+ent of pu.li& funds

Page 60: Nat Res Digested

8/10/2019 Nat Res Digested

http://slidepdf.com/reader/full/nat-res-digested 60/173

PetitionersI ar'u+ents are devoid of +erit The) la&( .asis in fa&t and in la%

The o%nership of these paintin's le'all) .elon's to the foundation or

&orporation or the +e+.ers thereof, althou'h the pu.li& has .een 'iven the

opportunit) to vie% and appre&iate these paintin's %hen the) %ere pla&ed

on eBhi.it

The &onfis&ation of these properties .) the A uino ad+inistration ho%ever

should not .e understood to +ean that the o%nership of these paintin's has

auto+ati&all) passed on the 'overn+ent %ithout &o+pl)in'

%ith &onstitutional and statutor) re uire+ents of due pro&ess and 7ust

&o+pensation -f these properties %ere alread) a& uired .) the 'overn+ent,

an) &onstitutional or statutor) defe&t in their a& uisition and their

su.se uent disposition +ust .e raised onl) .) the proper parties the true

o%ners thereof %hose authorit) to re&over e+anates fro+ their proprietar)

ri'hts %hi&h are prote&ted .) statutes and the Constitution 5avin' failed to

sho% that the) are the le'al o%ners of the art%or(s or that the valued

pie&es have .e&o+e pu.li&l) o%ned, petitioners do not possess an) &learle'al ri'ht %hatsoever to uestion their alle'ed unauthori3ed disposition

!either &an this petition .e allo%ed as a taBpa)erIs suit O.viousl),

petitioners are not &hallen'in' an) eBpenditure involvin' pu.li& funds .ut

the disposition of %hat the) alle'e to .e pu.li& properties -t is %orth) to

note that petitioners ad+it that the paintin's and anti ue silver%are %ere

a& uired fro+ private sour&es and not %ith pu.li& +one)Anent the se&ond re uisite of a&tual &ontrovers), petitioners ar'ue that this

&ase should .e resolved .) this Court as an eB&eption to the rule on +oot

and a&ade+i& &asesN that althou'h the sale of the paintin's and silver has

lon' .een &onsu++ated and the possi.ilit) of retrievin' the treasure trove is

Page 61: Nat Res Digested

8/10/2019 Nat Res Digested

http://slidepdf.com/reader/full/nat-res-digested 61/173

nil, )et the novelt) and i+portan&e of the -ssue raised .) the petition

deserve this CourtIs attention The) su.+it that the resolution .) the Court

of the -ssue in this &ase %ill esta.lish future 'uidin' prin&iples and do&trines

on the preservation of the nationIs pri&eless artisti& and &ultural possessions

for the .enefit of the pu.li& as a %hole

For a &ourt to eBer&ise its po%er of ad7udi&ation, there +ust .e an a&tual

&ase of &ontrovers) W one %hi&h involves a &onfli&t of le'al ri'hts, an

assertion of opposite le'al &lai+s sus&epti.le of 7udi&ial resolutionN the &ase

+ust not .e +oot or a&ade+i& or .ased on eBtra le'al or other si+ilar

&onsiderations not &o'ni3a.le .) a &ourt of 7usti&e A &ase .e&o+es +oot

and a&ade+i& %hen its purpose has .e&o+e stale, su&h as the &ase .efore

us Sin&e the purpose of this petition for prohi.ition is to en7oin respondent

pu.li& offi&ials fro+ holdin' the au&tion sale of the art%or(s on a parti&ular

date W 11 Januar) 1991 W %hi&h is lon' past, the -ssue raised in the

petition have .e&o+e +oot and a&ade+i&

The &ultural properties of the nation %hi&h shall .e under the prote&tion of the state are &lassified as the i+portant &ultural properties and the

national &ultural treasures On the other hand, a national &ultural

treasures is a uni ue o.7e&t found lo&all), possessin' outstandin' histori&al,

&ultural, artisti& andLor s&ientifi& value %hi&h is hi'hl) si'nifi&ant and

i+portant to this &ountr) and nation This Court ta(es note of the

&ertifi&ation issued .) the 4ire&tor of the 6useu+ that the -talian paintin's

and silver%are su.7e&t of this petition do not &onstitute prote&ted &ulturalproperties and are not a+on' those listed in the Cultural Properties Re'ister

of the !ational 6useu+

Page 62: Nat Res Digested

8/10/2019 Nat Res Digested

http://slidepdf.com/reader/full/nat-res-digested 62/173

=5<R<FOR<, for la&( of +erit, the petition for prohi.ition and +anda+us is

4-S6-SS<4

Page 63: Nat Res Digested

8/10/2019 Nat Res Digested

http://slidepdf.com/reader/full/nat-res-digested 63/173

Page 64: Nat Res Digested

8/10/2019 Nat Res Digested

http://slidepdf.com/reader/full/nat-res-digested 64/173

refle&ts the vi.ran&) of Philippine herita'e and &ulture -t is a proud le'a&) of

an earlier 'eneration of Filipinos %ho .elieved in the no.ilit) and sa&redness

of independen&e and its po%er and &apa&it) to release the full potential of

the Filipino people To all intents and purposes, it has .e&o+e a part of the

national patri+on) " Petitioner also ar'ues that sin&e #1D of the shares of

the 65C &arries %ith it the o%nership of the .usiness of the hotel %hi&h is

o%ned .) respondent S-S, a 'overn+ent o%ned and &ontrolled

&orporation, the hotel .usiness of respondent S-S .ein' a part of the

touris+ industr) is un uestiona.l) a part of the national e&ono+)

%))*e& =hether or !ot the sale of 6anila 5otel to Renon' Gerhad is violative

of the Constitutional provision of Filipino First poli&) and is therefore null and

void

He+ & The 6anila 5otel or, for that +atter, #1D of the 65C, is not 7ust an)

&o++odit) to .e sold to the hi'hest .idder solel) for the sa(e of

privati3ation The 6anila 5otel has pla)ed and &ontinues to pla) a si'nifi&ant

role as an authenti& repositor) of t%entieth &entur) Philippine histor) and&ulture This is the plain and si+ple +eanin' of the Filipino First Poli&)

provision of the Philippine Constitution And this Court, heedin' the &larion

&all of the Constitution and a&&eptin' the dut) of .ein' the elderl) %at&h+an

of the nation, %ill &ontinue to respe&t and prote&t the san&tit) of the

Constitution -t %as thus ordered that S-S a&&epts the +at&hin' .id of

petitioner 6A!-2A PR-!C< 5OT<2 CORPORAT-O! to pur&hase the su.7e&t

#1D of the shares of the 6anila 5otel Corporation at P 00 per share andthereafter to eBe&ute the ne&essar) &learan&es and to do su&h other a&ts and

deeds as +a) .e ne&essar) for purpose

The Supre+e Court dire&ted the S-S and other respondents to &ease and

desist fro+ sellin' the #1D shares of the 65C to the 6ala)sian fir+ Renon'

Page 65: Nat Res Digested

8/10/2019 Nat Res Digested

http://slidepdf.com/reader/full/nat-res-digested 65/173

Gerhad, and instead to a&&ept the +at&hin' .id of the petitioner 6anila

Prin&e 5otel

A&&ordin' to Justi&e Gellosillo, ponente of the &ase at .ar, Se&tion 10, se&ond

para'raph, Arti&le 11 of the 198$ Constitution is a +andator) provision, apositive &o++and %hi&h is &o+plete in itself and needs no further 'uidelines

or i+ple+entin' la%s to enfor&e it The Court <n Gan& e+phasi3ed that

ualified Filipinos shall .e preferred over forei'ners, as +andated .) the

provision in uestion

The 6anila 5otel had lon' .een a land+ar(, therefore, +a(in' the #1D of

the e uit) of said hotel to fall %ithin the purvie% of the &onstitutional shelter

for it e+prises the +a7orit) and &ontrollin' sto&( The Court also reiterated

ho% +u&h of national pride %ill vanish if the nation s &ultural herita'e %ill fall

on the hands of forei'ners

-n his dissentin' opinion, Justi&e Puno said that the provision in uestion

should .e interpreted as pro Filipino and, at the sa+e ti+e, not anti alien in

itself .e&ause it does not prohi.it the State fro+ 'rantin' ri'hts, privile'es

and &on&essions to forei'ners in the a.sen&e of ualified Filipinos 5e alsoar'ued that the petitioner is estopped fro+ assailin' the %innin' .id of

Renon' Gerhad .e&ause the for+er (ne% the rules of the .iddin' and that

the forei'ners are ualified, too

Page 66: Nat Res Digested

8/10/2019 Nat Res Digested

http://slidepdf.com/reader/full/nat-res-digested 66/173

6anos&a Hs CA Case 4i'est

M'-o)c' 3). "o*r( A e'+)

2 2 S"RA 412G.R. No. 106440'-*'r: 29, 1996

F'c()& The !ational 5istori&al -nstitute de&lared the par&el of land o%ned .)

Petitioners as a national histori&al land+ar(, .e&ause it %as the site of the

.irth of FeliB 6analo, the founder of -'lesia ni Cristo The Repu.li& of the

Philippines filed an a&tion to appropriate the land Petitioners ar'ued that the

eBpropriation %as not for a pu.li& purpose

%))*e& =hether or !ot the ta(in' or eBer&ise of e+inent do+ain +a) .e

'ranted

He+ & Pu.li& use should not .e restri&ted to the traditional uses The ta(in'

is for a pu.li& use .e&ause of the &ontri.ution of FeliB 6analo to the &ulture

and histor) of the Philippines

Page 67: Nat Res Digested

8/10/2019 Nat Res Digested

http://slidepdf.com/reader/full/nat-res-digested 67/173

MMDA 3). !e+;A/r 3/++'5e [ 28 S"RA 8 6 G.R. No. 1 962 27 M'r

2000]

Frida), Januar) /0, *009 Posted .) Coffeeholi& =rites

2a.els: Case 4i'ests , Politi&al 2a%

F'c()& 6etropolitan 6anila 4evelop+ent Authorit) 664A , petitioner

herein, is a overn+ent A'en&) tas(ed %ith the deliver) of .asi& servi&es in

6etro 6anila Gel Air Hilla'e Asso&iation GAHA , respondent herein, re&eived

a letter of re uest fro+ the petitioner to open !eptune Street of Gel

Air Hilla'e for the use of the pu.li& The said openin' of !eptune Street %ill.e for the safe and &onvenient +ove+ent of persons and to re'ulate the

flo% of traffi& in 6a(ati Cit) This %as pursuant to 664A la% or Repu.li& A&t

!o $9* On the sa+e da), the respondent %as appraised that the

peri+eter %all separatin'the su.division and Qala)aan Avenue %ould .e

de+olished

The respondent, to stop the openin' of the said street and de+olition of the%all, filed a preli+inar) in7un&tion and a te+porar) restrainin' order

Respondent &lai+ed that the 664A had no authorit) to do so and the lo%er

&ourt de&ided in favor of the Respondent Petitioner appealed the de&ision of

the lo%er &ourts and &lai+ed that it has the authorit) to open !eptune

Street to pu.li& traffi& .e&ause it is an a'ent of the State that &an pra&ti&e

poli&e po%er in the deliver) of .asi& servi&es in 6etro 6anila

%))*e& =hether or not the 664A has the +andate to open !eptune Street

to pu.li& traffi& pursuant to its re'ulator) and poli&e po%ers

Page 68: Nat Res Digested

8/10/2019 Nat Res Digested

http://slidepdf.com/reader/full/nat-res-digested 68/173

He+ & The Court held that the 664A does not have the &apa&it)

toeBer&ise poli&e po%er Poli&e po%er is pri+aril) lod'ed in the !ational

2e'islature 5o%ever, poli&e po%er +a) .e dele'ated to 'overn+ent units

Petitioner herein is a develop+ent authorit) and not a politi&al 'overn+ent

unit Therefore, the 664A &annot eBer&ise poli&e po%er .e&ause it &annot .e

dele'ated to the+ -t is not a le'islative unit of the 'overn+ent Repu.li&

A&t !o $9* does not e+po%er the 664A to ena&t ordinan&es, approve

resolutions and appropriate funds for the 'eneral %elfare of the inha.itants

of 6anila There is no s)lla.le in the said a&t that 'rants 664A poli&e po%er

-t is an a'en&) &reated for the purpose of la)in' do%n poli&ies and

&oordinatin' %ith various national 'overn+ent a'en&ies, people s

or'ani3ations, non 'overn+ental or'ani3ations and the private se&tor for the

effi&ient and eBpeditious deliver) of .asi& servi&es in the vast +etropolitan

area

Page 69: Nat Res Digested

8/10/2019 Nat Res Digested

http://slidepdf.com/reader/full/nat-res-digested 69/173

Te&hnolo') vs CA 19/ s&ra 1 $

Fa&ts: Te&hnolo') 4evelopers -n& is en'a'ed in +anufa&turin' and

eBportin' &har&oal .ri uette On Fe.ruar) 1", 1989, the) re&eived a letter

fro+ respondent A&tin' 6a)or Pa.lo Cru3, orderin' the full &essation of the

operation of the petitioner s plant in Sta 6aria, Gula&an The letter also

re uested the &o+pan) to sho% to the offi&e of the +a)or so+e do&u+ents,

in&ludin' the Guildin' per+it, +a)or s per+it, and Re'ion --- Pollution of

<nviron+ental and !atural Resour&es Anti Pollution Per+it Sin&e the

&o+pan) failed to &o+pl) in .rin'in' the re uired do&u+ents, respondent

A&tin' 6a)or, %ithout noti&e, &aused the padlo&( of &o+pan) s plant

pre+ises, effe&tivel) &ausin' stoppa'e of its operation Te&hnolo')

4evelopers then instituted an a&tion for &ertiorari, prohi.ition, +anda+us

%ith preli+inar) in7u&tion a'ainst respondents, alle'in' that the &losure

order %as issued in 'rave a.use of dis&retion The lo%er &ourt ruled a'ainst

the &o+pan) The CA affir+ed the lo%er &ourt s rulin'-ssue: 1 =hether or not the +a)or has authorit) to order the &losure of the

plant ><S* =hether or not the &losure order %as done %ith 'rave a.use of

dis&retion !O Rulin': 1 !o +a)orIs per+it had .een se&ured =hile it is true that the +atter of

deter+inin' %hether there is a pollution of the environ+ent that re uires

&ontrol if not prohi.ition of the operation of a .usiness is essentiall)

addressed to the then !ational Pollution Control Co++ission of the 6inistr)of 5u+an Settle+ents, no% the <nviron+ental 6ana'e+ent Gureau of the

4epart+ent of <nviron+ent and !atural Resour&es, it +ust .e re&o'ni3ed

that the +a)or of a to%n has as +u&h responsi.ilit) to prote&t its inha.itants

fro+ pollution, and .) virture of his poli&e po%er, he +a) den) the

Page 70: Nat Res Digested

8/10/2019 Nat Res Digested

http://slidepdf.com/reader/full/nat-res-digested 70/173

appli&ation for a per+it to operate a .usiness or other%ise &lose the sa+e

unless appropriate +easures are ta(en to &ontrol andLor avoid in7ur) to the

health of the residents of the &o++unit) fro+ the e+issions in the operation

of the .usiness* The A&tin' 6a)or, in the letter, &alled the attention of petitioner to the

pollution e+itted .) the fu+es of its plant %hose offensive odor not onl)

pollute the air in the lo&alit) .ut also affe&t the health of the residents in the

area, so that petitioner %as ordered to stop its operation until further orders

and it %as re uired to .rin' the follo%in': a Guildin' per+itN . 6a)orIs

per+itN and & Re'ion --- 4epart+ent of <nviron+ent and !atural Resour&es

Anti Pollution per+it

/ This a&tion of the A&tin' 6a)or %as in response to the &o+plaint of theresidents of Garan'a) u)on', Sta 6aria, Gula&an, dire&ted to the Provin&ial

overnor throu'h &hannels The &losure order of the A&tin' 6a)or %as issued onl) after an

investi'ation %as +ade -t found that the fu+es e+itted .) the plant of

petitioner 'oes dire&tl) to the surroundin' houses and that no proper air

pollution devi&e has .een installed# Petitioner failed to produ&e a .uildin' per+it fro+ the +uni&ipalit) of Sta

6aria, .ut instead presented a .uildin' per+it issued .) an offi&ial of 6a(ati " =hile petitioner %as a.le to present a te+porar) per+it to operate .) the

then !ational Pollution Control Co++ission on 4e&e+.er 1#, 198$, the

per+it %as 'ood onl) up to 6a)*#, 1988 Petitioner had not eBerted an) effort to eBtend or validate its per+it +u&h

less to install an) devi&e to &ontrol the pollution and prevent an) ha3ard to

the health of the residents of the &o++unit) Petitioner ta(es note of theplea of petitioner fo&usin' on its hu'e invest+ent in this dollar earnin'

industr) -t +ust .e stressed ho%ever, that &on&o+itant %ith the need to

pro+ote invest+ent and &ontri.ute to the 'ro%th of the e&ono+) is the

Page 71: Nat Res Digested

8/10/2019 Nat Res Digested

http://slidepdf.com/reader/full/nat-res-digested 71/173

e uall) essential i+perative of prote&tin' the health, na) the ver) lives of

the people, fro+ the deleterious effe&t of the pollution of the environ+ent

Page 72: Nat Res Digested

8/10/2019 Nat Res Digested

http://slidepdf.com/reader/full/nat-res-digested 72/173

G.R. No. !#80!9 &1l2 3!" !997

E%I$ANIO LALICAN"petitioner& vs. (ON. $ILOMENO A. *ERGARA"% e 4 4, &1 e" RTC B +,c6 5 " %1e to

% 4,ce + C4t2 +, %EO%LE O$ T(E %(ILI%%INES" respondents.I 1e:

Hhether the term lum#er is included in the concept o+ tim#er in orderto constitute an o ense as stated in Sec. (6 o+ :residential Decree ,o. 1)7 he Forestry -e+orm Code o+ the :hilippines8.

$+ct : he petitioners *ere apprehended on the Sitio Cadi3& Barangay

Bacungan :uerto :rincesa +or violating Section (6 o+ :D ,o. 1) or >no*n as he Forestry -e+orm Code o+ the :hilippines. here *ere $& 611 #oard +eet o+

lum#er loaded in t*o 7%8 passenger eeps in di erent si3es and dimensionthat *ere con/scated. "n August '& $''$& all the accused *ere pleaded notguilty to the crime charged.

:etioner alican /led a motion to ;uash the in+ormation /led againstthem contenting that& Section (6 o+ :D 1) does not include lum#er #ecausethe *ording o+ the la* categorically speci+y tim#er to #e collected as toconstitute the violation on the said la*. 5e +urther contends that& the la* isvague #ecause it does speci+y the authority or legal documents re;uired #ye isting +orest la* and regulation.

he prosecution opposed the motion to ;uash on the ground that it isnot the courts to determine the *isdom o+ the la* or to set the policy as rest

#y the legislature. 5e +urther asserts that the *ord tim#er should includelum#er *hich is a product or derivative o+ a tim#er. he position o+ theprosecution could result to the circumvention o+ the la*& +or one couldstealthily cut a tim#er and process it to #ecome a lum#er. "n Septem#er %0&$''$& the lo*er court construed the interpretation o+ the la* against theState thus the motion *as granted.

he prosecution /led a motion +or reconsideration on the orderunderscoring the +act that the accused presented :rivate and im#er :ermit,o. 141$01 dated Fe#ruary $1& $''$ *hich had e pired? that *hile thecerti/cate o+ origin indicated Brgy. Sta. Cru3& the product actually came +romSitio Cadi3& and that the t*o eeps #earing the product *ere not e;uipped*ith certi/cates o+ transport agreement. Added to this *as the +act that& i+ the product *ere indeed lum#er& then the accused could have presented acerti/cate o+ lum#er origin& lum#er sale invoices in case o+ sale& tally sheetsand delivery receipts +or transportation +rom one point to another. hemotion *as approved thus this case.

R1l4, :

Page 73: Nat Res Digested

8/10/2019 Nat Res Digested

http://slidepdf.com/reader/full/nat-res-digested 73/173

,"& he Court ruled that& the *ord lum#er includes tim#er. he primaryreason *hy the la* *as enacted is to secure and ma imi3e the use o+ thenatural resources? the non inclusion o+ lum#er on the la* may give rise +orthe circumvention o+ la*. Section (6 o+ the said la* punishes these actsnamely 7a8 the cutting, gathering, collection, or removal of timber or other

forest products from the places therein mentioned without any authority; or (b) possession of timber or other forest products without the legaldocuments as required under existing forest laws and regulations. Be that asit may& the legislative intent to include possession o+ lum#er in Sec. (6 isclearly gleaned +rom the e pressed reasons +or enacting the la* *hich&under E ecutive "rder ,o. % . o e clude possession o+ Olum#erO +rom theacts penali3ed in Sec. (6 *ould certainly emasculate the la* itsel+. A la*should not #e so construed as to allo* the doing o+ an act *hich is prohi#ited#y la*& nor so interpreted as to a ord an opportunity to de+eat compliance*ith its terms& create an inconsistency& or contravene the plain *ords o+ thela*. A+ter all& the phrase O+orest productsO is #road enough to encompass

lum#ers *hich& to reiterate& is manu+actured tim#er. 5ence& to mentionlum#er in Sec. (6 *ould merely result in tautology.

G.R. No. !58!8 &1,e ! " ##8

SESINANDO MERIDA" petitioner& vs. %EO%LE O$ T(E%(ILI%%INES" respondent.

I 1e:

$. Hhether the trial court ac;uired urisdiction over Criminal Case ,o.%%1 even though it *as #ased on a complaint /led #y ansiongco andnot #y a DE,- +orest ocer

%. Hhether petitioner is lia#le +or violation o+ Section (6 o+ :D 1). N$+ct :

:etitioner *as charged in the - C o+ -om#lon *ith violation o+ Section(6 o+ :D 1) +or Ocutting& gathering& collecting and removing a lone narra tree insidea private land over *hich private complainant "scar ansiongco claimso*nership. Hhen con+ronted during the meeting a#out the +ell narra tree&

petitioner admitted cutting the tree #ut claimed that he did so *ith the permissiono+ one 2icar Cali *ho& according to petitioner& #ought the Mayod :roperty +rom ansiongco in "cto#er $'6 under a pacto de retro sale. It *as later +ound out that heconverted the narra trun> into lum#er.

5e *as +ound guilty #y the rial Court #ut he appealed to the Courto+ Appeals reiterating his de+ense o+ denial. :etitioner also contended that the

Page 74: Nat Res Digested

8/10/2019 Nat Res Digested

http://slidepdf.com/reader/full/nat-res-digested 74/173

trial court did not ac;uire urisdiction over the case #ecause it *as #ased on acomplaint /led #y ansiongco and not #y a +orest o cer as provided under Section 61o+ :D 1). CA a rmed the lo*er court<s ruling& #ut ordered the sei3edlum#er con/scated in the government=s +avor. Also& it sustained the trialcourt=s /nding that petitioner is #ound #y his e tra udicial admissions o+ cutting the narra tree in the Mayod :roperty *ithout any DE,- permit.

R1l4, :

es& he -evised -ules o+ Criminal :rocedure list the cases *hich must#e initiated #y a complaint /led #y speci/ed individuals& non@compliance o+ *hich ousts the trial court o+ urisdiction +rom trying such cases. 5o*ever&these cases concern only de+amation and other crimes against chastity andnot to cases concerning Section (6 o+ :D 1). Further& Section 61o+ :D 1) does not prohi#it an interested person +rom /ling a complaint#e+ore any ;uali/ed o cer +or violation o+ Section (6 o+ :D 1)& asamended.

Moreover& here& it *as not +orest o cers o+ employees o+ the Bureau o+ Forest Development *ho reported to 5ernande3 the tree@cutting in theMayod :roperty #ut ansiongco& a private citi3en *ho claims o*nershipover the Mayod :roperty. hus& 5ernande3 cannot #e +aulted +or notconducting an investigation to determine Oi+ there is prima +acieevidence to support the complaint or report.O At any rate& ansiongco *asnot precluded& either under Section 61 o+ :D 1) or the -evised -ules& +rom /linga complaint #e+ore the :rovincial :rosecutor +or petitioner=s alleged violation o+ Section(6 o+ :D 1).

%. es& :etitioner is guilty o+ the second paragraph o+ section 61& *hich is thecutting& gathering& collecting& or removing o+ tim#er +rom aliena#le or disposa#lepu#lic land& or +rom private land *ithout any authority. he court also saidthat the lum#er or Lprocessed log Lis covered #y the L+orest products term in:D 1)& as the la* does not distinguish #et*een a ra* and processed tim#er.

A.M. No. MT&-93-87 M+ c6 ! " !995AUGUSTUS L. MOMONGAN petitioner& vs. &UDGE RA$AEL B. OMI%ON"respondent.

I 1e:Hhether the respondent Judge erred in releasing the truc> used to

transport an illegal lum#er despite o+ prima +acie evidence +or violation o+ :D1) as amended #y E" % .

Page 75: Nat Res Digested

8/10/2019 Nat Res Digested

http://slidepdf.com/reader/full/nat-res-digested 75/173

$+ct :Augustus Momongan is the -egional Director o+ DE,- in aclo#an City&

*hile the respondent Judge "mipon is the incum#ent Judge o+ MC C o+ 5inunangan Silago& Southern eyte. At around $1!11 o+ ,ovem#er $0& $''%the police o cer o+ 5inunangan Silago& Southern eyte apprehended a truc>

loaded *ith illegally cut lum#er. he truc> *as o*ned #y Basilio Ca#ig drived#y Dionisio Golpe. A+ter the apprehension and con/scation& a preliminaryinvestigation *as done to determine *hether there is a pro#a#le cause toengender the o*ner o+ the truc> and the driver guilty on the violation o+ :D1). Despite o+ the presence o+ prima +acie evidence the respondent Judgeordered the release o+ the truc> apprehended. Mr.Ca#ig *as charged against:D 1) #ut Mr. Golpe the driver *as not included in the complaint.

he -egional Director Momongan /led an instant complaint against the udge alleging that the release order *as a violation o+ :D 1) Sections (6and (6@A respectively& and Administrative "rder ,o. )'.Complainant claimsthat respondent Judge has no authority to order the release o+ the truc>

despite the non@inclusion o+ Mr. Golpe in the complaint. he truc> shouldhave #een turned over to the Community Environment and ,atural-esources " ce o+ San Juan& Southern eyte +or appropriate disposition asthe same +alls under the administrative urisdiction o+ the Department o+ Environment and ,atural -esources " ce.

-espondent Judge e plained that a+ter conducting the preliminaryinvestigation& he +ound that Golpe& the o*ner o+ the truc>& is principallyengaged in the hauling o+ sand and gravel and the delivery o+ hollo* #loc>s&and the loading o+ the tim#er in the car is due to the re;uest o+ his +riendCa#ig. -espondent Judge o#served that Golpe has a lesser participation inthe crime o+ illegal logging. More importantly& the +act that the complaint

charged only Ca#ig& respondent Judge& in the e ercise o+ his sounddiscretion& ordered the release o+ the truc> o*ned #y Golpe.

R1l4, :,o& he court +ound that the respondent order to release the truc>

o*ned and driven #y Mr. Dionisio Golpe legally usti/a#le. According to the-:C& LEvery penalty imposed for the commission of a felony shall carry withit the forfeiture of the proceeds of the crime and the instrument or tools withwhich it was committed. 5o*ever& this cannot #e done i+ such proceeds andinstruments or tools O#e the property o+ a third person not lia#le +or o ense.OIn this case& the truc>& though used to transport the illegally cut lum#er&cannot #e con/scated and +or+eited in the event accused therein #econvicted #ecause the truc> o*nerKdriver& Mr. Dionisio Golpe *as notindicted. 5ence& there *as no usti/cation +or respondent Judge not torelease the truc>.

Complainant is correct in pointing out that the DE,- Secretary or hisduly authori3ed representative has the po*er to con/scate any illegallyo#tained or gathered +orest products and all conveyances used in thecommission o+ the o ense and to dispose o+ the same in accordance *ith

Page 76: Nat Res Digested

8/10/2019 Nat Res Digested

http://slidepdf.com/reader/full/nat-res-digested 76/173

pertinent la*s. he release o+ the truc> did not render nugatory theadministrative authority o+ the DE,- Secretary. Despite the order o+ release&the truc> can #e sei3ed again either #y /ling a motion +or reinvestigation andmotion to include the truc> o*nerKdriver& as co@accused& *hich complainanthas done as mani+ested #e+ore the lo*er court or #y en+orcing Adm. "rder

,o. )'. Section $%.G.R. No. !#!#83 &1l2 3#" !993

&UAN ANTONIO" ANNA ROSARIO +, &OSE AL$ONSO" +ll 1 ,+ eO%OSA" 4,o " +, e e e,te b2 t6e4 + e,t petitioners&vs.T(E (ONORABLE $ULGENCIO S. $ACTORAN" &R." 4, 64 c+ +c4t2 +t6e Sec et+ 2 o; t6e De + t e,t o; E,<4 o, e,t +, N+t1 +lRe o1 ce " +, T(E (ONORABLE ERIBERTO U. ROSARIO" % e 4 4,

&1 e o; t6e RTC" M+=+t4" B +,c6 00" respondents.

I 1eHhether children have the legal standing to /le the caseP

$+ct his case is uni;ue in that it is a class suit #rought #y 00 children&

through their parents& claiming that they #ring the case in the name o+ Ltheirgeneration as *ell as those generations yet un#orn. Aiming to stopde+orestation& it *as /led against the Secretary o+ the Department o+ Environment and ,atural -esources & see>ing to have him cancel all thetim#er license agreements 7 As8 in the country and to cease and desist+rom accepting and approving more tim#er license agreements.

he children invo>ed their right to a #alanced and health+ul ecology and toprotection #y the State in its capacity as parens patriae.

he petitioners claimed that the DE,- Secretary=s re+usal to cancel the As and to stop issuing them *as Ocontrary to the highest la* o+ human>ind@@ the natural la*Qand violative o+ plainti s= right to sel+@preservation and perpetuation.O he case *as dismissed in the lo*er court&invo>ing the la* on non@impairment o+ contracts& so it *as #rought to the

Supreme Court on certiorari.R1l4,

es. he Supreme Court in granting the petition ruled thatthe children had the legal standing to /le the case #ased on the concept o+ Lintergenerational responsi#ility . heir right to a healthy environmentcarried *ith it an o#ligation to preserve that environment +or the succeeding

Page 77: Nat Res Digested

8/10/2019 Nat Res Digested

http://slidepdf.com/reader/full/nat-res-digested 77/173

generations. In this& the Court recogni3ed legal standing to sue on #ehal+ o+ +uture generations. Also& the Court said& the la* on non@impairment o+ contracts must give *ay to the e ercise o+ the police po*er o+ the state inthe interest o+ pu#lic *el+are.

G.R. No. !3! 7# M+ c6 !7" ###%ER$ECTO %ALLADA" petitioner& vs. %EO%LE O$ T(E%(ILI%%INES" respondent.

I 1e:Hhether a separate certi/cates o+ origin is used +or lum#er and tim#er.

$+ct :

Sometime in the latter part o+ $''%& DE,- received a reports that

illegally cut lum#er *ere delivered in the *arehouse o+ 2alencia Golden5arvest Corporation in 2alencia Bu>idnon. DE,- o cers in colla#oration o+ :,: raided the company<s *arehouse and +ound a large stoc>pile o+ lum#erin varying si3es cut #y a chainsa*. As proo+ that the company had ac;uiredthe lum#er #y purchase& petitioner produced t*o receipts issued #y -. .-ivero um#eryard o+ Maramag& Bu>idnon& dated March ( and $ & $''%. heDE,- o cers did not& ho*ever& give credit to the receipt considering that -.. -ivero um#eryard=s permit to operate had long #een suspended. Hhat ismore& the pieces o+ lum#er *ere cut #y chain sa* and thus could not havecome +rom a licensed sa*mill operator.

"n Fe#ruary %4& $''4& petitioner& as general manager& together *ith,oel Sy& as assistant operations manager& and Francisco an>i>o& aspresident o+ the 2alencia Golden 5arvest Corporation& and Isaias 2aldehue3a&*ere charged *ith violation o+ section (6 o+ :.D. ,o. 1)& as amended.During the trial& the accused presented documents that the lum#er arelegally o#tained. his may include the certi/cate o+ origin. 5o*ever& thecourt +ound out that :allada *as guilty o+ the violation o+ :D 1) and the resto+ the accused *ere ac;uitted due to insu ciency o+ evidence. he case *as

Page 78: Nat Res Digested

8/10/2019 Nat Res Digested

http://slidepdf.com/reader/full/nat-res-digested 78/173

appealed to the CA and rendered a decision a rming the decision o+ thelo*er court& thus this case *as elevated.

R1l4, :

es& there should #e a separate Certi/cate o+ origin. he trial courtacted correctly in not giving credence to the Certi/cates o+ im#er "riginpresented #y petitioner since the lum#er held #y the company should #ecovered #y Certi/cates o+ um#er "rigin. For indeed& as BFD Circular ,o. $1@64 states in pertinent parts!

!n order to provide an e"ective mechanism to pinpoint accountability and responsibility for shipment of lumber . . . and to have uniformity indocumenting the origin thereof, the attached #erti$cate of %umber &rigin (#%&) . . . which form's part of this circular 'is hereby adoptedas accountable forms for o cial use by authori*ed + - o cers . . . .

. %umber . . . transported/shipped without the necessary #erti$cate of %umber &rigin (#%&) . . . as herein required shall be considered as proceeding from illegal sources and as such, shall be sub0ect tocon$scation and disposition in accordance with %&! 1232 and + -implementing guidelines.

he irregularities and discrepancies ma>e the documents in *hich theyare +ound not only ;uestiona#le #ut invalid and& thus& usti/ed the trialcourt in giving no credence to the same. he presence o+ such glaringirregularities negates the presumption that the C "s *ere regularlye ecuted #y the DE,- o cials concerned.

G.R. No. L- 077 $eb 1+ 2 !3" !99%EO%LE O$ T(E %(ILI%%INES" petitioner& vs. COURT O$ $IRSTINSTANCE O$ >UE?ON " respondent.

I 1e: Hhether the in+ormation correctly and properly charged an o ense and

*hether the trial court had urisdiction over the case.

$+ct : he private respondents *ere charged *ith the crime o+ ;uali/ed the+t

o+ logs& de/ned and punished under Section (6 o+ :residential Decree ,o. 1)&other*ise >no*n as the -evised Forestry Code o+ the :hilippines.

he in+ormation provided that Godo+redo Arro3al and uis Flores&together *ith %1 other John Does *hose identities are still un>no*n& the

Page 79: Nat Res Digested

8/10/2019 Nat Res Digested

http://slidepdf.com/reader/full/nat-res-digested 79/173

/rst@named accused #eing the administrator o+ the In+anta oggingCorporation& conspired and entered the privately@o*ned land o+ one Felicitacion:u alte& titled in the name o+ her deceased +ather& Macario :rudente& andproceeded to illegally cut& gather& and ta>e& there +rom& *ithout the consento+ the said o*ner and *ithout any authority under a license agreement& (1logs o+ di erent species.

"n March %4& $' & the named accused /led a motion to ;uash thein+ormation on %grounds& to *it! 7$8 that the +acts charged do not constitutean o ense? and& 7%8 that the in+ormation does not con+orm su#stantially tothe prescri#ed +orm. rial court thus dismissed the in+ormation #ased on therespondent<s grounds.

R1l4, : he elements o+ the crime o+ ;uali/ed the+t o+ logs are! $8 hat the

accused cut& gathered& collected or removed tim#er or other +orest products?%8 that the tim#er or other +orest products cut &gathered& collected orremoved #elongs to the government or to any private individual? and 48 thatthe cutting& gathering& collecting or removing *as *ithout authority undera license agreement& leas& license& or permit granted #y the state. he +ailureo+ the in+ormation to allege that the logs ta>en *ere o*ned #y the state isnot +atal. It should #e noted that the logs su# ect o+ the complaint *ere ta>ennot +rom a pu#lic +orest #ut +rom private *oodland registered in the name o+ complainant=s deceased +ather& Macario :rudente. he +act that only thestate can grant a license agreement& license or lease does not ma>e thestate the o*ner o+ all the logs and tim#er products produced inthe :hilippines including those produced in private *oodlands. hus&o*nership is not an essential element o+ the o ense as de/ned in Section (1 o+ :.D. ,o. 1). As to the second issue raised& the regular courts still has

urisdiction. Sec. 61 o+ :D 1)covers % speci/c instances *hen a +orest o cermay commence a prosecution +or the violation o+ the -evised Forestry Code o+ the :hilippines.

he /rst authori3es a +orest o cer or employee o+ the Bureau o+ Forestry to arrest *ithout a *arrant& any person *ho has committed or is

committing& in his presence& any o+ the o enses descri#ed in the decree. he second covers a situation *hen an o ense descri#ed in the decree isnot committed in the presence o+ the +orest o cer or employee and thecommission is #rought to his attention #y a report or a complaint. In #oth cases&ho*ever& the +orest o cer or employee shall investigate the o ender and /lea complaint *ith the appropriate o cial authori3ed #y la* to conduct apreliminary investigation and /le the necessary in+ormations in court.

Page 80: Nat Res Digested

8/10/2019 Nat Res Digested

http://slidepdf.com/reader/full/nat-res-digested 80/173

n+ortunately& the instant case does not +all under any o+ the situations covered#y Section 61 o+ :.D. 1). he alleged o ense *as committed not in thepresence o+ a +orest o cer and neither *as the alleged commission reportedto any +orest o cer. he o ense *as committed in a private land and thecomplaint *as #rought #y a private o ended party to the /scal. As such&the "SG *as correct in insisting that :.D. 1) did not repeal Section $(6o+ the Administrative Code giving authority to the /scal to conductinvestigation into the crime o+ demeanour and have the necessary in+ormationor complaint prepared or made against person charged *ith the commission o+ the crime. In short& Section 61 does not grant e clusive authority to the +oresto cers& #ut only special authority to rein+orce the e ercise o+ such #y thoseupon *hom vested #y the general la*.

G.R. No. !30! Octobe " ###

%EO%LE O$ T(E %(ILI%%INES" plainti"4appellee, vs. AL$ONSO DATORet.+l " Acc1 e - A el+,t

I 1e: Hhether the penalty imposed to elan the accused is correct in

violation o+ :D 1)

$+ct :

:ator eala and his co accused Al+onso Dator and Benito Genol *erecharged *ith the crime o+ violation o+ Section (6 o+ :residential Decree ,o.1)& other*ise >no*n as the -evised Forestry Code. he accused *hile

transporting pieces o+ lum#er #ound to Maasin Souther eyte& they *ereapprehended #y the police o cer and sei3ed pieces o+ lum#er. As a resultS:"$ Bacala issued a sei3ure receipt covering the /+ty@one 7)$8 pieces o+ con/scated Dita and Antipolo lum#er and one 7$8 unit o+ Isu3u cargo truc>*ith :late ,o. 5AF (%6. he con/scated pieces o+ lum#er and the cargo truc>*ere turned over to S:"4 Daniel asala& :,: :roperty Custodian o+ Maasin&Southern eyte *ho& in turn& o cially trans+erred custody o+ the same to theCE,-"& Maasin& Southern eyte. he accused elan alleged that the pieces o+ lum#er *ere cut +rom the trac> o+ land #elonging to his mother in San Jose&Maasin& Southern eyte *hich he intended to use in the renovation o+ hishouse in Barangay A#gao o+ the same municipality. 5e +urther contends thathe secured ver#al permission to Boy eonor an o cer@in @charge o+ theDE,-.

he lo*er courts +ound out that the accused is guilty in violation o+ :D1) sentencing the accused to su er the indivisi#le penalty o+ -EC SI",

Page 81: Nat Res Digested

8/10/2019 Nat Res Digested

http://slidepdf.com/reader/full/nat-res-digested 81/173

:E-:E A& *ith the accessory penalties provided #y la*& *hich is t*o 7%8degrees higher than :-ISI", MA"- ma imum& the authori3ed penaltysimilar to 9uali/ed he+t& and to pay the costs. hus& this case *as elevatedto the court.

R1l4, :,o& In the case at #ench& the con/scated /+ty@one 7)$8 pieces o+

assorted Dita and Antipolo lum#er *ere classi/ed #y the CE,-" o cials asso+t& and there+ore not premium ;uality lum#er. It may #e noted that the saidpieces o+ lum#er *ere cut #y the appellant& a mere anitor in a pu#lichospital& +rom the land o*ned #y his mother& not +or commercial purposes#ut to #e utili3ed in the renovation o+ his house. It does not appear thatappellant elen had #een convicted nor *as he an accused in any otherpending criminal case involving violation o+ any o+ the provisions o+ the-evised Forestry Code 7:.D. ,o. 1)& as amended8. In vie* o+ the attendant

circumstances o+ this case& and in the interest o+ ustice& the #asis +or thepenalty to #e imposed on the appellant should #e the minimum amountunder Article 41' paragraph 7(8 o+ the -evised :enal Code *hich carries thepenalty o+ arresto mayor in its minimum and medium periods +or simplethe+t.

Considering that the crime o+ violation o+ Section (6 o+ :residentialDecree ,o. 1)& as amended& is punished as ;uali/ed the+t under Article 4$1o+ the -evised :enal Code& pursuant to the said decree& the imposa#lepenalty on the appellant shall #e increased #y t*o degrees& that is& +romarresto mayor in its minimum and medium periods to prision mayor in its

minimum and medium periods. Applying the Indeterminate Sentencea*& the penalty to #e imposed on the appellant should #e si 7(8 monthsand one 7$8 day o+ prision correccional to si 7(8 years and one 7$8 day o+ prision mayor.

G.R. No. ! #305 Dece be !7" !990

%EO%LE O$ T(E %(ILI%%INES" plainti @appellee& vs. ILSON B.>UE" accused@appellant

I 1e:Hhether the appellant<s activities consist an o ense

$+ct ::rovincial as> Force got *ind that a that a ten@*heeler truc> #earing

plate num#er :AD@)06 loaded *ith illegally cut lum#er *ill pass throughIlocos ,orte. Acting on said in+ormation& mem#ers o+ the : F *enton patrol several times *ithin the vicinity o+ Genera l Segundo Avenuein aoag City. "n March 6& $''0& S:"$ Corpu3& together *ith S:"$ Raldy

Page 82: Nat Res Digested

8/10/2019 Nat Res Digested

http://slidepdf.com/reader/full/nat-res-digested 82/173

Asuncion and S:"$ Elmer :atoc *ent on patrol around the area. Ata#out$!11 in the morning& they posted themselves at the corner o+ GeneralSegundo Avenue and -i3al Street. hirty minutes later& they sa* a ten@*heeler truc> *ith plate num#er :AD@)06 pass #y. hey +ollo*ed the truc>and apprehended it at the Marcos Bridge."n June %4& $''0& accused@appellant *as charged #e+ore the -egional rial Court o+ aoag *ith violationo+ Section (6 o+ :.D. 1)as amended #y E.". % . he In+ormation allegedthat& on or a#out the 6 th day o+ March& $''0& in the City o+ aoag& :hilippines&and *ithin the urisdiction o+ this 5onora#le Court& the a#ove@namedaccused& #eing then the o*ner o+ an I7s8u3u en Hheeler ruc> #earing :late,o. :AD@)06& *ith intent o+ gain& did then and there *ill+ully& unla*+ully and+eloniously have in possession& control and custody %)6 pieces o+ varioussi3es o+ Forest :roducts Chain sa* lum#er 7Species o+ anguile8 *ith a totalvolume o+ 4& %'.4 #d. +t. or e;uivalent to 6. ' cu#ic meters valued in the

total amount o+ :'4&%4%.)1 at:%).11K#d. +t.& necessary permit& license orauthority to do so +rom the proper authorities Accused@appellant denied thecharge against him. 5e claimed that he ac;uired the %)6 pieces o+ tanguilelum#er +rom a legal source. During the trial& he presented the private landtim#er permits 7: :8 issued #y the Department o+ Environment and ,atural-esources 7DE,-8 to Enrica Cayosa and Elpidio Sa#al he : : authori3es itsholder to cut& gather and dispose tim#er +rom the +orest area covered #y thepermit. 5e alleged that the tanguile lum#er came +rom the +orest areacovered #y the : :<s o+ Cayosa and Sa#al and that they *ere given to him#y Cayosa and Sa#al as payment +or his hauling services

R1l4, : es& :ossession o+ the lum#er *ithout the necessary permit is a

violation o+ the -FC. Hhen the police apprehended 9ue& he +ailed to presentdocumentary evidence to prove that he has the permit to possess andtransport the lum#er. All he had *as the permit +or the coconut sla#s. 5eeven concealed the lum#er so as to avoid it +rom #eing seen upon /rstinspection o+ the load. nder the circumstances& there is no dou#t that theaccused *as a*are that he needed documents to possess and transport thelum#er& #ut could not secure one and there+ore& concealed such #y placing itin such a manner that it could not #e seen #y merely loo>ing at the cargo.

here are % *ays o+ violating Sec. (6 o+ the -evised Forestry Code!a. #y cutting& gathering andKor collecting tim#er or other +orest

products *ithout licence and#. #y possessing tim#er or other +orest products *ithout re;uired legal

documents.

Page 83: Nat Res Digested

8/10/2019 Nat Res Digested

http://slidepdf.com/reader/full/nat-res-digested 83/173

In the /rst o ense& one can raise as a de+ense the legality o+ said acts.5o*ever& in the second o ense& mere possession *ithoutproper documentation consummates the crime.

G.R. No. !0!798 Octobe #" ##

%ICO% RESOURCES" INC." petitioner& vs. (ON. AUGUSTUS L. CALO"% e 4 4, &1 e" respondent

I 1e@

Hhether petitioner has the right to retain the sei3ed con/scatedproducts #y the virtue o+ M"A regarding the :rocedural Guidelines in theConduct o+ 2eri/cation o+ :rivate ree :lantation.

$+ct :

:etitioner :IC": -esources& Inc. 7:IC":8 o*ns and operates a multi@#illion peso pulp and paper manu+acturing +acility in Bislig City& Agusan del,orte. It holds government@issued :ulp*ood and im#er icense Agreement7: A8 ,o. 0 and Integrated Forest Management Agreement 7IFMA8 ,o. 4)*hich gave petitioner the e clusive right to co@manage and develop *ith theState almost $41&111 hectares o+ +orest land *ithin the Agusan@Davao@Surigao Forest -eserve.

he Department o+ Environment and ,atural -esources 7DE,-8&through its o cers& rendered three Memoranda& dated August %%& $'' &Fe#ruary $(& %11$ and April (& %11$ designating the petitioner as DE,-depository and custodian +or apprehended +orest products and conveyances*ithin its concession. "n May %)& %11$& the " ce o+ the CE,-"@Bislig andpetitioner entered into a Memorandum o+ Agreement 7M"A8 containingO:rocedural Guidelines in the Conduct o+ 2eri/cation o+ :rivate ree:lantation.O he M"A provided& among others& that /eldvalidationKveri/cation o+ applications +or Certi/cates o+ :rivate ree"*nership 7C :"s8 shall #e conducted ointly #y the DE,-& the localgovernment unit concerned& and petitioner. :ursuant to these Memoranda&petitioner<s security personnel *ere deputi3ed as DE,- o cers to apprehendand sei3e the tools& e;uipment and conveyance used in the commission o+ illegal logging and the +orest products removed and possessed #y theo enders.

In the course o+ the en+orcement o+ the a+oresaid Memoranda&petitioner :IC":& through its security personnel& had on numerous occasionsapprehended *ithin its concession and tree plantation area. hese illegallycut +orest products and conveyances *ere >ept in :IC":<s impounding area.

Page 84: Nat Res Digested

8/10/2019 Nat Res Digested

http://slidepdf.com/reader/full/nat-res-digested 84/173

A class suit *as initiated among the mem#ers o+ FAB as>ing +orpreliminary mandatory In unction. hey +urther as>ed +or the declaration o+ the memoranda null and void and sought to restrain the DE,- and those*ho are participants +rom en+orcing the said memoranda. he - C orderedElias -. Seraspio& Jr. to recall& *ithdra* and a#rogate the en+orcement o+ the

assailed Memorandum dated Fe#ruary $(& %11$ and to re+rain and desist+rom implementation. :etitioner *as also ordered to release thecon/scated falcata logs and vehicles to the o*ners thereo+& or to the CE,-"@Bislig or the " ce o+ the Government :rosecution@Surigao del Sur& *here theadministrative and criminal proceedings *ere ongoing.

R1l4, :

:etitioner had no right or interest to protect in the con/scated +orestproducts and conveyances. :etitioner<s compound *as used only as adepository +or the con/scated logs and conveyances #y virtue o+ the

Memorandum. Hhile it claimed that some o+ the con/scated +orest productsmay have come +rom its concession area& petitioner admitted that theo*nership o+ the con/scated products *as still to #e determined in the casespending either at the CE,-"@Bislig or at the " ce o+ the Government:rosecution@Surigao del Sur. 5ence& petitioner<s interest in the con/scated+orest products *as merely contingent and cannot #e material ascontemplated under Section %& -ule 4 o+ the -evised -ules o+ Civil :rocedure.:etitioner contends that private respondents< intrusion *as in violation o+ petitioner<s : A ,o. 0 and IFMA ,o. 4). hese license agreements gavepetitioner the e clusive right to co@manage and develop +orest lands& andrecogni3ed petitioner as o*ner o+ the trees and other products in the

concession area. In /ling this petition& petitioner is merely de+ending itssu#sisting proprietary interest pursuant to these license agreements.

It is clear that petitioner has no material interest to protect in thecon/scated +orest products and conveyances. It has no su#sisting proprietaryinterest& as #orne out #y its licensing agreements& *hich need to #eprotected #y annulling the *rit o+ in unction issued #y the trial court.:etitioner also cannot claim the right to retain custody o+ the apprehendedlogs and conveyances #y virtue o+ its #eing designated a depository o+ theDE,- pursuant to the assailed Memoranda. As such depository& petitionermerely holds the con/scated products and conveyances in custody +or theDE,- *hile the administrative or criminal proceedings regarding saidproducts are pending.

Page 85: Nat Res Digested

8/10/2019 Nat Res Digested

http://slidepdf.com/reader/full/nat-res-digested 85/173

GR NO. !5 989. Se te be " ##

Rol +," & . petitioner <. (o," M+ o,+ et.+l. respondents

I 1e:

a. Hhether a person *ho cuts trees +or his o*n use *ithin his property*ithout the necessary permit +rom the DE,- and *ithout transporting thesame outside said property& #e criminally charged +or violating :D 1)P

#. Hhether the o*ner o+ a private property is administratively lia#leunder Section $0 o+ DE,- Administrative "rder ,o. %111@%$ despite the +actthat he did not transport the logs out o+ his property and used them +or hiso*n agricultural purposes.

$+ct : "n August '& %11$& petitioner applied +or a :rivate and im#er :ermit

7: :8 +rom the Department o+ Environment and ,atural -esources +or him tocut some trees +or a proposed road and poultry +arm in his property. 5e alsopaid all the +ees re;uired #y the various government agencies. Hhile *aiting+or the permit to #e issued& petitioner *as allegedly in+ormed #y some

Page 86: Nat Res Digested

8/10/2019 Nat Res Digested

http://slidepdf.com/reader/full/nat-res-digested 86/173

employees +rom the Department o+ Environment and ,atural -esources7DE,-8 that he could proceed *ith the cutting o+ trees even though hisapplication *as still a*aiting approval. Conse;uently& petitioner proceeded*ith the cutting o+ trees and #ulldo3ing o+ the road*ay. 5e used the cut logsas materials to #uild his chic>en cages. A#out three *ee>s later&representatives o+ the Community Environment and ,atural -esources " ce7CE,-"8 o+ the Department o+ Environment and ,atural -esources andpersonnel +rom the Intelligence Service& Armed Forces o+ the :hilippines7ISAF:8 o+ aclo#an City raided petitioner<s place& allegedly *ithout a search*arrant. An inventory o+ the cut trees *as conducted there *ere 6 % pieceso+ sa*n lum#erK itches 76&)1( #oard +eet8 and three +elled tim#er logs *ith atotal mar>et value o+ :%4)&0)0.(6 at :% .11 per #oard +oot.

R1l4, :a. es& nder Section (6& :D 1) as amended #y E.". % & it is clear

that the violators o+ the said la* are not declared as #eing guilty o+ ;uali/edthe+t. As to the assertion that his penalty +or cutting trees in his o*n landshould not #e e;uated *ith that +or ;uali/ed the+t& su ce it to say that the

udiciary is never concerned *ith the *isdom o+ the la*. Hhether or not thelegislature *as correct in imposing on violators o+ :D 1) a penalty e;ual tothat imposa#le on those guilty o+ ;uali/ed the+t is a ;uestion #eyond thepo*er o+ the Court to resolve. It is a settled rule that the +undamental duty o+ the Court is to apply the la* regardless o+ *ho may #e a ected& even i+ thela* is harsh @ dura lex sed lex

Section $0 o+ Administrative "rder ,o. %111@%$& the L-evisedGuidelines in the Issuance o+ :rivate and im#er :ermitKSpecial :rivate and

im#er :ermit& provides!SEC. $0. :enal :rovisions. @ Any logKtim#er or /nished@*ood

products covered #y these regulations *hich are transported*ithout the prescri#ed documents shall #e considered illegaland& there+ore& su# ect to con/scation in +avor o+ the governmentand shall #e disposed in accordance *ith la*s& rules andregulations governing the matter.

#. ,o& he rule is clear. he a+orementioned administrative orderconsiders the mere act of transporting any *ood product or tim#er without the prescribed documents as an o ense *hich is su# ect to the penaltiesprovided +or #y la*.

Page 87: Nat Res Digested

8/10/2019 Nat Res Digested

http://slidepdf.com/reader/full/nat-res-digested 87/173

G.R. No. !8 #98 No<e be 5" ##8

AMADO TAO%A" petitioner& vs. %EO%LE O$ T(E%(ILI%%INES" respondent.

I 1e:

Hhether the penalty imposed against the petitioner is correct inviolation o+ :D 1)

$+ct :

"n April %& $''(& the Community Environment and ,atural-esources " ce o+ 2irac& Catanduanes sei3ed a truc> loaded *ith illegally@cut lum#er and arrested its driver& :lacido Cuison. he lum#er *as covered

*ith #undles o+ a#aca /#er to prevent detection. "n investigation& Cuisonpointed to petitioner Amado aopa and a certain -u/no "galesco as theo*ners o+ the sei3ed lum#er aopa& "galesco and Cuison pleaded not guiltyon arraignment. A+ter trial on the merits& the -C +ound them guilty ascharged #eyond reasona#le dou#t. "nly aopa and Cuison appealed the - Cdecision to the Court o+ Appeals 7CA8. Cuison *as ac;uitted #ut aopa=sconviction *as a rmed. he dispositive portion o+ the CA decision read!

WHEREFORE , the -ecision appealed from is REVERSED withrespect to accused4appellant #uison, who is ACQUITTED of the crimecharged on reasonable doubt, and MODIFIED with respect to accused4

appellants 5mado Taopa and 6u$no &galesco by reducing the penaltyimposed on them to four (7) years, nine (8) months and eleven

(11) days of prision correccional, as minimum, to ten (12) yearsof prision mayor, as maximum

-uling!

,"& Section (6 o+ :D 1)& as amended& re+ers to Articles 41' and 4$1o+ the -evised :enal Code 7-:C8 +or the penalties to #e imposed on violators.2iolation o+ Section (6 o+ :D 1)& as amended& is punished as ;uali/ed the+t.

he la* treats cutting& gathering& collecting and possessing tim#er or other+orest products *ithout license as an o ense as grave as and e;uivalent tothe +elony o+ ;uali/ed the+t.

he actual mar>et value o+ the $$4 pieces o+ sei3ed lum#er*as :( &(41. Follo*ing Article 4$1 in relation to Article 41'& the imposa#lepenalty should #e reclusion temporal in its medium and ma imum periods ora period ranging +rom $0 years& eight months and one day to %1 years plusan additional period o+ +our years +or the e cess o+ :0 &(41.

Page 88: Nat Res Digested

8/10/2019 Nat Res Digested

http://slidepdf.com/reader/full/nat-res-digested 88/173

he minimum term o+ the indeterminate sentence imposa#le on aopashall #e the penalty ne t lo*er to that prescri#ed in the -:C. In this case& theminimum term shall #e any*here #et*een $1 years and one day to $0 yearsand eight months or prision mayor in its ma imum period to reclusiontemporal in its minimum period.

he ma imum term shall #e the sum o+ the additional +our years andthe medium period o+reclusion temporal in its medium and ma imumperiods or $( years& /ve months and $$ days to $6 years& t*o months and%$ days o+ reclusion temporal . he ma imum term there+ore may #eany*here #et*een $( years& /ve months and $$ days o+ reclusiontemporal to %% years& t*o months and %$ days o+ reclusion perpetua .

G.R. No. !75 89 A1 1 t 3!" #!!

CRISOSTOMO *ILLARIN et.+l " :etitioners& . %EO%LE O$ T(E

%(ILI%%INES" -espondent.I 11e:

Hhether mere possession o+ tim#er *ithout the legal documentsre;uired under +orest la*s and regulations ma>es one automatically lia#leeven criminal intent in violation o+ Section (6& :residential Decree 7:.D.8 ,o.1)& as amended.

$+ct :

In a Criminal Complaint

/led #e+ore the Municipal rial Court in Cities&Branch 0& Cagayan de "ro City #y Marcelino B. :io;uinto 7:io;uinto8& Chie+ o+ the Forest :rotection and a* En+orcement nit under the Stri>e Force

eam o+ Department o+ Environment and ,atural -esources 7DE,-8&petitioner Aniano atayada 7 atayada8 and three others namely& BarangayCaptain Camilo Sudaria 7Sudaria8 o+ agpangi& Cagayan de "ro City& MarlonBaillo 7Baillo8 and Cipriano Boyatac 7Boyatac8& *ere charged *ith violation o+ Section (6& :.D. ,o. 1) as amended #y E ecutive "rder ,o. % . herespondents *ere guilty o+ gathering and possessing si ty@three 7(48 piecesitches o+ varying si3es #elonging to the Apitong specie *ith a total volumeo+ Four housand hree 5undred *enty Si 70&4%(8 #oard +eet valuedat :$16&$)1.11& *ithout any authority and supporting documents as re;uiredunder e isting +orest la*s and regulation to the damage and pre udice o+ thegovernment.

R1l4, :

es& As a special la*& the nature o+ the o ense is malumprohi#itum and as such& criminal intent is not an essential element. here is

Page 89: Nat Res Digested

8/10/2019 Nat Res Digested

http://slidepdf.com/reader/full/nat-res-digested 89/173

no dispute that petitioners *ere in constructive possession o+ the tim#er*ithout the re;uisite legal documents. 2illarin and atayada *ere personallyinvolved in its procurement& delivery and storage *ithout any license orpermit issued #y any competent authority. Given these and considering thatthe o ense is malum prohi#itum& petitioners< contention that the possession

o+ the illegally cut tim#er *as not +or personal gain #ut +or the repair o+ said#ridge is& there+ore& inconse;uential.

CA-G.R. S%. No. 8#9 7. $eb 1+ 2 " ##5

LT. RODELLO B. LARAYA" %N" et.+l." petitioners, vs. (ON. %ER$ECTO E.%E" RTC o; %+l+ +," respondent.

I 1e:Hhether the -espondent Judge acted in e cess o+ his Jurisdiction or

*ith Grave A#use "+ Discretion *hen he allo*ed the -e@"pening "+ :re@ rialFor :urposes "+ :lea@Bargaining *ithout he consent o+ the Complainantsand Contrary to Section %& -ule $$( "+ he -evised -ules "n Criminal:rocedure& As Amended.

$+ct :

It *as on $% Septem#er %11%& thirty eight 7468 Chinese nationals on#oard their /shing vessels *ere caught *ithin the Malampaya ,atural Gas:lat+orm :ro ect E clusive Rone& in El ,ido& :ala*an in the act o+ illegal/shing. 5undreds o+ >ilos o+ Groupers 7locally >no*n& as %apu4%apu 8& Hrasse7locally >no*n& as 9ameng 8 and Snappers 7locally >no*n& as Maya@Maya8*ere +ound in the said /shing vessels +ound *ithin the vessels& amongothers& *ere po*ders and pillets& suspected to #e no ious su#stances orcomponent ingredients o+ e plosives. A #ottle o+ home@made dynamite *as

Page 90: Nat Res Digested

8/10/2019 Nat Res Digested

http://slidepdf.com/reader/full/nat-res-digested 90/173

also retrieved. A criminal complaint *as /led against the accused@privaterespondents +or the violation o+ -epu#lic Act ,o. 6))1& other*ise >no*n asthe :hilippine Fisheries Code o+ $''6& speci/cally& Section 6 7:oaching8 andSection 66 7Fishing through E plosives& ,o ious or :oisonous Su#stances8 o+ the said Act.

Su#se;uently& criminal in+ormations +or violation o+ -epu#lic Act ,o.6))1 *ere /led against all 46 Chinese nationals *ith the -egional rial Courto+ :ala*an. All o+ the accused@private respondents therea+ter pleaded notguilty to the charges on 1 May %114.

During the pre@trial o+ the criminal cases& counsel +or the accused@private respondents mani+ested their intention to enter into plea #argaining#y entering a plea o+ guilty to a lesser o ense under paragraph % o+ Section66 o+ -epu#lic Act 6))1. 5o*ever& the then prosecuting o cer o+ the cases&:rovincial :rosecutor Alen -oss B. -odrigue3 did not accede

A+ter prosecution presented its /rst *itness and initial cross@e amination *as conducted #y the counsel +or the de+ense& or a+ter only t*ohearing dates& ho*ever& :rovincial :rosecutor Alen -oss B. -odrigue3mani+ested his intention to inhi#it +rom +urther prosecuting the criminalcases. Continuation o+ the trial *as& hence& rescheduled the +ollo*ing day.

"n $( July %114& *ith the appearance o+ a ne* prosecutor& counsel +orthe de+ense as>ed the court to re@open the pre@trial stage o+ the criminal

cases and reiterated the accused@private respondent<s earlier intention o+ availing plea #argaining. his time& the :rovincial :rosecutor<s " ce&represented #y :rosecutor "legario Cayetano& Jr.& did not o# ect. "n thecontrary& :rosecutor Cayetano mani+ested that the government *asamena#le to re@open pre@trial +or the purpose o+ plea@#argaining.Conse;uently& the trial court ordered the re@opening o+ the pre@trial.

During the re@opened pre@trial stage& all the accused@private respondentsthrough their counsel plead guilty to violation o+ :aragraph %& Section 66 o+ -epu#lic Act 6))1. he pu#lic prosecutor interposed no o# ection *ith the

change o+ plea and in+ormed the court that the prosecutor<s con+ormity *iththe plea #argaining *as in consonance *ith the directive o+ the Chie+ State:rosecutor

R1l4, :

NO" Section $& -ule () o+ the -evised -ules o+ Court e presslyprovides!

Page 91: Nat Res Digested

8/10/2019 Nat Res Digested

http://slidepdf.com/reader/full/nat-res-digested 91/173

: E#T!&< 1. =etition for #ertiorari. > ?hen any tribunal, board or o cer exercising 0udicial and quasi40udicial functions has acted without or in excessof its 0urisdiction, or with grave abuse of discretion amounting to lac@ or excess of 0urisdiction, and there is no appeal, or any plain, speedy, andadequate remedy in the ordinary course of law, a person aggrieved thereby may $le a veri$ed petition in the proper court, alleging the facts withcertainty and praying that 0udgment be rendered annulling or modifying the

proceedings of such tribunal, board or o cer, and granting such incidentalreliefs as law and 0ustice may require. AAA

AAA AAAB

5ence& +or the herein petitioners to lodge the instant action& they must/rst #e :a person aggrievedB, other*ise& they *ould #e *ithout legalstanding to pursue this legal recourse. hat having #een said& this Courtrules that the herein petitioners are not persons aggrieved #y the assaileddecision o+ the trial court in the su# ect criminal cases. It is elementary incriminal la* that a crime is an o ense against the State& and is henceprosecuted in the name o+ the :eople o+ the :hilippines. For this reason&Section ) o+ -ule $$1 o+ the -evised -ules o+ Criminal :rocedure providesthat Lall criminal actions commenced #y a complaint or in+ormation shall #eprosecuted under the direction and control o+ the prosecutor. Furthermore&Section $ o+ :.D. $% )& provides the e clusive domain o+ the prosecutory armo+ the government as ho* #est to deal *ith the prosecution o+ criminalcases. 5ence& any grievance in course thereo+ a ecting the interest o+ the

State must proceed only +rom such an arm o+ the government.

Page 92: Nat Res Digested

8/10/2019 Nat Res Digested

http://slidepdf.com/reader/full/nat-res-digested 92/173

A.M. No. MT&- #3- ! 87. Dece be !" ##3

SANGGUNIANG BAYAN O$ GUINDULMAN" BO(OL & petitioner " vs . &UDGE

MANUEL A. DE CASTRO" Act4, % e 4 4, &1 e" MCTC" G14, 1l +,-D1e o" Bo6ol &respondent .

I 1e:

Hhether the respondent Judge committed gross ignorance o+ the la*+or not imposing the proper penalty provided +or in Sec. '1& -.A. ,o. 6))1&other*ise >no*n as he :hilippine Fisheries Code o+ $''6.

Facts!It *as on May $ & %11% *hen la*men apprehended a #oat captain

and eight cre* mem#ers o+ the /shing #oat BKB Junida@J *ho *ere /shing*ithin the vicinity o+ the municipal *aters and /sh sanctuary o+ Basdio&Guindulman& Bohol *ith a ring net 7>no*n locally as Llicom 8. Charges +orviolation o+ Sections 6(& '1 and '( o+ -epu#lic Act ,o. 6))1 *ereimmediately /led *ith the MC C& Guindulman@Duero& Bohol& presided over inan acting capacity #y Judge Manuel A. de Castro. In the morning o+ the veryne t day& a Saturday& t*o o+ the accused& namely! ,arciso J. Jusay& Jr. 7#oato*ner8 and -olando . Amistoso 7#oat captain8 *ere released +rom detentionupon order o+ respondent. It appears that respondent held a court sessionon May $6& %11%& despite the +act that it *as a Saturday& and proceeded toarraign #oth accused *ho pleaded guilty. he respondent udge e aggeratedthe speedy disposition o+ the case commanding the accused to pay /vethousand pesos and +urther commanded to return the alleged impounded#oats to the accused. Surprised #y such turn o+ events& the SangguniangBayan passed a resolution +or the +urther investigation on the action o+ respondent udge a#out the crime charged against the accused and a copy

thereo+ *as received #y the " ce o+ the Chie+ Justice& Supreme Court on June %)& %11%.

R1l4, :

es& the respondent Judge committed a grave ignorance o+ the la* andviolation o+ circulars issued #y the court *hen he tried a case in Saturday.

he respondent udge impudently misused his authority to impose the

Page 93: Nat Res Digested

8/10/2019 Nat Res Digested

http://slidepdf.com/reader/full/nat-res-digested 93/173

penalty under the la* *hich it cannot #e countenanced. I+ udges *antonlymisuse the po*ers vested in them #y la*& there *ill not only #e con+usion inthe administration o+ ustice #ut even also oppressive disregard o+ the #asicre;uirements o+ due process. he o#servance o+ the la*& *hich he is #oundto >no* is re;uired o+ every udge. Hhen the la* is su ciently #asic& a

udge o*es it to his o ce to simply apply it? anything less than that *ould #econstitutive o+ gross ignorance o+ the la*. A udge should #e theem#odiment o+ competence& integrity and independence. It is a pressingresponsi#ility o+ udges to >eep a#reast *ith the la* and the changes therein+or ignorance o+ the la*& *hich everyone is #ound to >no*& e cuses no one&not even udges. Indeed& it has #een said that @@ *hen the ine ciencysprings +rom a +ailure to consider so #asic and elemental a rule& a la* or aprinciple in the discharge o+ his duties& a udge is either too incompetent andundeserving o+ the position and the title he holds or is too vicious that the

oversight or omission *as deli#erately done in #ad +aith and in grave a#useo+ udicial authority.

A.M. No. MT&-# -! 3#. Se te be 8" ##3

S%ECIAL %ROSECUTOR ROMEO B. SENSON" complainant , vs . &UDGE(ERIBERTO M. %ANGILINAN"

MTCC" %UERTO %RINCESA CITY"respondent .

I 1e:

Hhether the decision o+ the respondent udge is correct in approvingthe rgent motion +or custody o+ Fishing ,et.

$+ct :

"n $0 March %111& several persons *ere apprehended +or violation o+ Section 6( o+ -epu#lic Act ,o. 6))1& also >no*n as L he :hilippine FisheriesCode o+ $''6 #y mem#ers o+ the :hilippine ,ational :olice. he itemssei3ed +rom those arrested included 7a8 $ unit /sh net& 7#8 4( units lights7411 *atts8& 7c8 $ unit light 7)11 *atts8& 7d8 $ unit #uoy& 7e8 containers& 7+8 plastic container #o es& 7g8 0 styropore #o es& and 7h8 $1 #o es o+ /sh. "nthe same day& Criminal Case ,o.$)1$' against them *as /led. hree dayslater& Danilo Alayon and ,orma 2illarosa& asserting to #e the co@o*ners o+ the MKB Ting Fisher that *as used in the illegal /shing activity& /led anL rgent Motion +or Custody o+ Fishing ,et& alleging that the /sh net *hichcosts Lno less than :(11&111.11 *as le+t unattended at the #each e posed

Page 94: Nat Res Digested

8/10/2019 Nat Res Digested

http://slidepdf.com/reader/full/nat-res-digested 94/173

to the elements and movements o+ the sea *hich could cause its earlydeterioration and ultimate loss. -espondent Judge& despite the vigorouso# ection o+ the pu#lic prosecutor& granted the motion in part& to the+ollo*ing e ect @

:To obviate their possible loss, destruction and/or deterioration, pendingresolution of the above4captioned case, the apprehending o cers or whoever has the custody, are ordered to cause the immediate turnover of the following items to movants who underta@e to produce the samewhenever needed in court, as they can only be properly con$scated in favor of the government upon conviction of the accused .

he respondent contended that -epu#lic Act ,o. 6))1& the la* under*hich the accused *ere charged *ith having transgressed& did not provide+or the sei3ure o+ the /shing paraphernalia pending trial and that theprosecution still could prove the guilt o+ the accused #eyond reasona#le

dou#t even *ithout the evidence #eing presented since it had su cient*itnesses +or the purpose.

R1l4, :

,o& the sei3ure o+ the /shing paraphernalia has #een made as #eing anincident to a la*+ul arrest. -ule $% & Section $%& o+ the -ules o+ Courtprovides!

LSEC. $%. Search incident to la*+ul arrest.@ A person la*+ully arrested may#e searched +or dangerous *eapons or anything *hich may #e used as proo+

o+ the commission o+ an o ense& *ithout a search *arrant.

G.R. No. !3 5! Dece be !7" !999

CONGRESSMAN ENRI>UE T. GARCIA" petitioner& vs. (ON. RENATO C.CORONA" 4, 64 c+ +c4t2 + t6e E ec1t4<eSec et+ 2" (ON. $RANCISCO *IRAY" 4, 64 c+ +c4t2 +

t6e Sec et+ 2 o; E,e 2" CALTE) %(ILI%%INES INC."%ILI%INAS S(ELL %ETROLEUM COR%. +,%ETRON COR%."respondents.

I 1e:

Hhether the e clusion o+ Section $' 7setting the time o+ +ullderegulation o+ oil la*8 on -A 60 ' made #y the congress ma>es the la*unconstitutional.

Page 95: Nat Res Digested

8/10/2019 Nat Res Digested

http://slidepdf.com/reader/full/nat-res-digested 95/173

$+ct :

,ovem#er )& $'' & this Court in Tatad v . ecretary of the -epartment of Energy and %agman &et al .&v . Con .6uben Torres &et al .& declared -epu#licAct ,o. 6$61& entitled OAn Act Deregulating the Do*nstream "il Industry and

For "ther :urposesO& unconstitutional& and its implementing E ecutive "rder,o. 4'% void. -.A. 6$61 *as struc> do*n as invalid #ecause three >eyprovisions intended to promote +ree competition *ere sho*n to achieve theopposite result. More speci/cally& this Court ruled that its provisions on tari di erential& stoc>ing o+ inventories& and predatory pricing inhi#it +aircompetition& encourage monopolistic po*er& and inter+ere *ith the +reeinteraction o+ the mar>et +orces.

As a result o+ the atad decision& Congress enacted -epu#lic Act ,o.60 '& a ne* deregulation la* *ithout the o ending provisions o+ the earlierla*. :etitioner Enri;ue . Garcia& a mem#er o+ Congress& has #rought the

petition see>ing to declare Section $' thereo+& *hich sets the time o+ +ullderegulation& unconstitutional. A+ter +ailing in his attempts to have Congressincorporate in the la* the economic theory he espouses& petitioner as>s thecourt& in the name o+ upholding the Constitution& to undo a violation *hich heclaims Congress has committed.

R1l4, :

,o& It #ears stressing that -.A. 6$61 *as declared invalid not #ecause

deregulation is unconstitutional. he la* *as struc> do*n #ecause& ascra+ted& three >ey provisions plainly encouraged the continued e istence i+ not the proli+eration o+ the constitutionally proscri#ed evils o+ monopoly andrestraint o+ trade. It is not the +unction o+ the Court to sa+eguard themem#ers o+ the congress on *hat la* they *ill enact #ecause this is thee act tenet on the encroachment o+ po*er as stipulated on the principle o+ separation o+ po*er. he court as a *hole& respects the la*s legislate #y thecongress. he validity o+ such la*s& may only #e impugned i+ it violates theConstitution. In the case at #ar& the Constitution perse *as not violated norridiculed.

G.R. No . ! #805-7! Dece be 7" !995

LAGUNA LA/E DE*ELO%MENT AUT(ORITY" petitioner& vs. COURT O$A%%EALS@ (ON. &UDGE

Page 96: Nat Res Digested

8/10/2019 Nat Res Digested

http://slidepdf.com/reader/full/nat-res-digested 96/173

(ERCULANO TEC(" %RESIDING &UDGE" BRANC( 7#" REGIONALTRIAL COURT O$ BINANGONAN

RI?AL@ $LEET DE*ELO%MENT" INC. +,CARLITO ARROYO@ T(E MUNICI%ALITY O$

BINANGONAN +, o MAYOR ISIDRO B. %ACIS" respondents.

I 1e:

Hhether the ocal Government Code Code o+ $''$ repealed theCharter o+ aguna a>e Developmental Authority 7-A ,". 06)18 in theissuance o+ /sh pen permits and other related activity involving aguna deBay.

Facts!

his is a petition made #y aguna a>e Development Authority todeclare an e clusive po*er on the regulation o+ issuing a /sh open permitsover the #usinessmen engage in the aguna de #ay. he po*er to issue apermit *as then trans+erred to the o ce o+ the mayor on the di erentmunicipalities o+ aguna thus ma>ing the aguna de #ay cro*ded andunhealthy +or living o+ natural resources and danger to the livelihood amongthe +ol>s o+ aguna.

R1l4, :

,o& the court holds that the provisions o+ -epu#lic Act ,o. $(1 do notnecessarily repeal the la*s creating the aguna a>e Development Authorityand granting the latter *ater rights authority over aguna de Bay and thela>e region. he ocal Government Code o+ $''$ does not contain anye press provision *hich categorically e pressly repeal the charter o+ theAuthority. It has to #e conceded that there *as no intent on the part o+ thelegislature to repeal -epu#lic Act ,o. 06)1 and its amendments.

It has to #e conceded that the charter o+ the aguna a>eDevelopment Authority constitutes a special la*. -epu#lic Act ,o. $(1& theocal Government Code o+ $''$& is a general la*. It is #asic in statutoryconstruction that the enactment o+ a later legislation *hich is a general la*

cannot #e construed to have repealed a special la*. It is a *ell@settled rule inthis urisdiction that Oa special statute& provided +or a particular case or classo+ cases& is not repealed #y a su#se;uent statute& general in its terms&provisions and application& unless the intent to repeal or alter is mani+est&although the terms o+ the general la* are #road enough to include the casesem#raced in the special la*.O

Page 97: Nat Res Digested

8/10/2019 Nat Res Digested

http://slidepdf.com/reader/full/nat-res-digested 97/173

Hhere there is a con ict #et*een a general la* and a special statute& thespecial statute should prevail since it evinces the legislative intent moreclearly than the general statute. he special la* is to #e ta>en as ane ception to the general la* in the a#sence o+ special circumstances +orcinga contrary conclusion. A special la* cannot #e repealed& amended or altered

#y a su#se;uent general la* #y mere implication. hus& it has to #econcluded that the charter o+ the Authority should prevail over the ocalGovernment Code o+ $''$.

G.R. No. L-08 7 $eb 1+ 2 !!" !980

NUCLEAR $REE %(ILI%%INE COALITION" ET AL."petitioners& vs.NATIONAL %O ER COR%ORATION" ET AL." respondents.G.R. No. 7#03 $eb 1+ 2 !!" !980

LOREN?O M. TA ADA" ET AL." et4t4o,e " vs. %(ILI%%INE ATOMICENERGY COMMISSION" ET AL." respondents .

I 1e:

Hhether the udgement o+ :AEC on the nuclear po*er plant sa+e.

$+ct :a. G.-. ,o. 1(4%& petitioners ;uestion the competence o+ respondent

:AEC Commissioners to pass udgment on the sa+ety o+ the :hilippine,uclear :o*er :lant@$ :,::@$ in :AEC icensing :roceedings ,o. $@*ithout ho*ever see>ing their ouster +rom o ce& although OprovencompetenceO is one o+ the ;uali/cations prescri#ed #y la* +or :AECCommissioners. :etitioners also assail the validity of the motion(application) /led #y the ,ational :o*er Corporation 7,:C8 +or the conversiono+ its construction permit into an operating license +or :,::@$ on theprincipal ground that it contained no in+ormation regarding the /nancial;uali/cations o+ ,:C& its source o+ nuclear +uel& and insurance coverage +ornuclear damage.

#.. In G.-. ,o. (60 0& acting on the motion /led therein dated June 6&$'6) to order :AEC to reconsider its orders o+ May 4$ and June )& $'6)& theurgent motion +or mandatory in unction andKor restraining order datedAugust 4& $'6)& the second urgent motion +or mandatory in unction datedAugust $%& $'6)& and the various pleadings and other documents su#mitted

Page 98: Nat Res Digested

8/10/2019 Nat Res Digested

http://slidepdf.com/reader/full/nat-res-digested 98/173

#y the parties relative thereto& and considering the paramount need o+ areasona#le assurance that the operation o+ :,::@$ *ill not pose an undueris> to the health and sa+ety o+ the people& *hich dictates that the conduct o+ the in;uiry into the sa+ety aspects o+ :,::@$ #e characteri3ed #y su cientlatitude& the #etter to achieve the end in vie*& un+ettered #y technical rules

o+ evidence 7-epu#lic Act )%1 & section 408& and in >eeping *ith there;uirements o+ due process in administrative proceedings.

R1l4, :

+. he Court there+ore resolved to -ES -AI, respondent :AECCommissioners +rom +urther acting in :AEC icensing :roceedings ,o. [email protected] said decision is due to the pamphlets that :AEC had circulated.

5aving thus pre udged the sa+ety o+ the :,::@$ respondent :AECCommissioners *ould #e acting *ith grave a#use o+ discretion amounting tolac> o+ urisdiction *ere they to sit in udgment upon the sa+ety o+ the plant&

a#sent the re;uisite o# ectivity that must characteri3e such an importantin;uiry.

#. he respondent :AEC 7once reconstituted8 to re@open the hearing on:,::@$ so as to give petitioners su cient time to complete their cross@e amination o+ the e pert *itnesses on ;uality assurance& to cross@e aminethe *itnesses that petitioners have +ailed to cross@e amine on and a+terAugust '& $'6)& and to complete the presentation o+ their evidence& +or*hich purpose& respondent :AEC shall issue the necessary su#poena andsu#poena duces tecum to compel the attendance o+ relevant *itnessesandKor the production o+ relevant documents. For the said purposes& the

:AEC may prescri#e a time schedule *hich shall reasona#ly assure theparties su cient latitude to ade;uately present their case consistently *iththe re;uirements o+ dispatch. lt is understood that the :AEC may give ,:Cthe opportunity to correct or supply de/ciencies in this application orevidence in support thereo+.

Page 99: Nat Res Digested

8/10/2019 Nat Res Digested

http://slidepdf.com/reader/full/nat-res-digested 99/173

A*ucena Salalima vs# 3mployees Compensation Comm# and Soc# Sec# System1#R# "o#$'=H(H>

8acts6

Petitioner-s husband Guancho Saldima ,as employed for t,enty nine years as a route helper andsalesman for the Meycauayan Plant of Coca Cola 2ottlers Philippines %nc# during the annualcompany medical e/amination) Guancho ,as diagnosed ,ith pulmonary tuberculosis# :aterfound him to have cancer of the lungs and died after fe, months# A*ucena) the ,ife of Guanchois no, claiming for the benefits of her husband from the company and the SSS# RTC dismissedthe case# CA affirmed and this petition ,as therefore filed#

%ssue6

7hether the petitioner is entitled to benefits provided by P#D# H H

9eld6

SC ans,ered in the affirmative# 2ecause the facts of the case sho,ed that the cause of Guancho-sdeath ,as his mere ,or< and that his medical history states that his stay at Coca cola is acontributory to his sic<ness# SSS ,as ordered to pay the claimant and the RTC-s and CA-sdecisions ,ere reversed and set aside#

Social Gustice Society) et# al# vs# 9onorable Gose Atien*a) Gr#1#R# "o# 'EH>E

8acts6

Chevron is engaged in the business of importing) distributing and mar<eting of petroleum products in the Philippines ,hile Shell and Petron are engaged in the business of manufacturing)refining and li<e,ise importing and mar<eting of petroleum products# Petitioners sought tocompel Mayor Tien*a to enforce 4rdinance "o# @> ,hich ,as enacted by SangguniangPanlungsod of Manila and became effective upon approval by Mayor Atien*a# This ordinancereclassifies the area described from industrial to commercial and directed the o,ners to ceaseand desist from operating their business ,ithin H months# Among the business is the PandacanTerminal of the 4il companies# 4il companies intervened in the issue attac<ing the validity of theordinance#

%ssue6

7hether the ordinance approved by respondent is valid or not

9eld6

Page 100: Nat Res Digested

8/10/2019 Nat Res Digested

http://slidepdf.com/reader/full/nat-res-digested 100/173

&alid# 2ecause the tremendous event happened near the area ,hich many ,ere put into danger)the Manila Municipal 4ffice shall do its ministerial duty to protect all property and health ofthose people ,ho lived in the vicinity and nearby cities# The court ordered the transfer ofPandacan Terminal ,ithin a non e/tendible period of ?> days# The life of the people shall be theutmost priority of the government in terms of its security) though the business ,ill lose billions

of money) the municipality cannot sacrifice its people#

8ilinvest Credit Corporation vs# %AC and "estor Sunga) Gr#1#R# "o# HE?(E

8acts6

A case filed by "estor Sunga Gr#) businessman and o,ner of the "2S Machineries and the "AP$ "AP Transit# 9e purchased a minibus Ma*da from Motorcester ,ith an agreement to pay the balance in monthly basis# :ater) "estor failed to pay his obligations to the company ,hich causethe confiscation of the minibus by the officers of 8ilinvest Corp# The minibus ,as mortgaged to

8ilinvest Corp# Sunga cleared his obligations to 8ilinvest ,hich the court rendered decisionsgranting Sunga moral) actual damages) litigation e/penses and AttorneyIs fees# 8ilinvest filedmotion to revie, the decision of the court#

%ssue6

7hether the a,ard of damages to Sunga is valid

9eld6

Jes# %t is valid but it is unconscionable) therefore the SC reduced the amount granted to Sunga

since the facts sho, that the latter had not suffered much and that it is his obligation to pay theminibus as it ,as stipulated bet,een him and 8ilinvest Corp# Moral and actual damages ,eregranted but litigation e/penses ,as eliminated for it has no price for litigation#

:aguna :a<e Development Authority vs# Court of Appeals) et# al#1#R# "os# ' >@HE$ '

8acts6

A case filed by authority against all those ,ho ,ere given permit by Municipal mayors Pacis)Papa and Gala$Gala to include them as releasers of permits and other respondents for violating the

provisions ,ho has the .urisdiction to give permits# 2ut the issue not only lies on the granting of permits but the error on deciding that authority is not a 0uasi$.udicial agency that R#A# =@E> ,asamended by P#D# @'( and 3#4# ? s# of '?@( and the :15 has the po,er to issue permits#

%ssue6

7hether ::DA has the po,er as a regulatory and 0uasi$.udicial body

Page 101: Nat Res Digested

8/10/2019 Nat Res Digested

http://slidepdf.com/reader/full/nat-res-digested 101/173

9eld6

Jes# ::DA has e/press po,ers as regulatory and 0uasi$.udicial body to cease and desist ordersand on matters affecting the construction of illegal fish pens) fish cages and other a0ua$culturestructures in :aguna De 2ay# %t is not co$e0ual to RTC but only to its e/tent of po,er provided

by la,# All permits issued ,ere declared null and void and all structures on the said area shall bedemolished because of the void permits granted to o,ners and operators#

Agapito Magban,a) et# al# vs# %AC) et# al#1#R# no# HH@ >$

8acts6

A case ,here all plaintiffs ,ho ,ere tenants of the defendants complained the diversion of thefree flo, of ,ater from their farm lots ,hich caused portions of their landholdings to dry up totheir great damage and pre.udice and they ,ere as<ed to vacate the areas for they could not plant

any longer for lac< of ,ater#

%ssue6

7hether plaintiffs are entitled to damages

9eld6

Jes# 2ecause the closing of ,ater flo, to the petitioners- farm lots caused damage and pre.udicial to them in their harvest# %t has no sho,ing in the facts that petitioners ,ere negligent but instead the respondents- bad faith ,hich caused pre.udice to the former# 5nder the la,) the

lando,ner has the obligation to <eep tenant in the peaceful and continuous cultivation of hislandholding# A disturbance of possession such as the act complained of is violative of the la,#Therefore) the court granted moral and e/emplary damages and Attorney-s fees to plaintiffs#

3rnesto Rodrigue*) et# al# vs# %AC and Daytona Construction and Development Corporation1#R# no# =@'H

8acts6

Plaintiffs filed an action for abatement of a public nuisance ,ith damages against defendant#After four e/tensions of time to file an ans,er by defendant) Daytona moved to dismiss the case

for lac< of .urisdiction and cause of action# Motions denied) defendant ,as declared in defaultand authori*ed plaintiffs to present evidence e/ parte# The facts of the case at bar sho,s thatthere is other nearby residents ,ho ,ere pre.udiced by the Daytona Corp# for the running of its

business# %t affected their health and property# 2ut in this case) the delay of the respondents toans,er id 0uestionable and contrary to la,#

%ssues6

Page 102: Nat Res Digested

8/10/2019 Nat Res Digested

http://slidepdf.com/reader/full/nat-res-digested 102/173

Page 103: Nat Res Digested

8/10/2019 Nat Res Digested

http://slidepdf.com/reader/full/nat-res-digested 103/173

Ramire* of a P ) >>> bond) Gudge Roman Cansino ordered the court sheriff to ta<e possession ofthe @E tubs of fish for five days and release it to Ramire*# Commander 4campo filed a petition toreturn the fish for the same fish ,ere caught in T4"J :3F % boat# 9ere) the fish ,ere caughtthrough the use of dynamite) ,hich is a violation and crime under R#A# = @# Action for

prohibition and in.unction ,ere filed by 4campo against the Gudge and the sheriff#

%ssues6

7hether respondent Gudge erred in posting a bond in dissolving the ,arrant of sei*ure7hether Ramire* violated R#A# = @

9eld6

Jes# RTC Gudge Cansino erred in dissolving the ,arrant of sei*ure because the petitioner is theR#P and the same is e/empt from the obligation to post such a bond#Jes# Ramire* violated R#A# = @ because under this la,) it is violative to use dynamite in fishing

,hich the 2ureau of 8isheries strictly observes the implementation of the said la,#

Sea :ion 8ishing Corporation vs# People of the Philippines1#R# no# ' H @

8acts6

This is a petition for revie, on Certiorari assailing the decision of CA in denying the Motion forReconsideration and petition for Certiorari and Mandamus# 9ere) ' Chinese fishermen ,erecaught poaching off Mangsee %sland in Pala,an# The 2arangay officials and team of PhilippineMarines found 8+& Sea :ion ,ith five boats ,ith fishing nets spread over the ,ater# The court

filed various cases against the Chinese fishermen to ,it6 &iolation of section ? of R#A#@EE>@[@]; &iolation of section ?>?[?] of R#A# @EE> and violation of Section a! and f! '> ['>]of R#A# ?'=' '' ''> and section @ ' ' ! of R#A# @EE>) 8+& sea :ion filed an 5rgent Motionfor Release of evidence alleging that it o,ns the vessel# The court released the vessel but laterthe court found out that the evidence of o,nership of Sea :ion ,as not supported by documents#Therefore) government forfeited the vessel#

%ssue6

7hether the forfeiture of 8+& Sea :ion in favor of the government ,as proper

9eld6

Jes# The government ,as correct ,hen it forfeited 8+& Sea :ion since its motion ,as only filedafter the .udgment has been rendered and it failed to see< all remedies given the sufficient time todo so# The lo,er court had .urisdiction over the case and the petitioner ,as not denied of due

process and gets it failed to comply ,ith the other re0uirements provided in the la,#

Page 104: Nat Res Digested

8/10/2019 Nat Res Digested

http://slidepdf.com/reader/full/nat-res-digested 104/173

:egaspi &S# Civil Service Commission1#R# no# ''?

8acts6

Civil Service Commission denied &alentin :egaspi-s petitioner! re0uest for information on thecivil service eligibilities of people employed as sanitarians) Gulian Sibonghanoy and MarianoAgas) in the 9ealth Department in Cebu# Petitioner claims that his right to information isguaranteed by the Constitution prays for the issuance of the e/traordinary ,rit of mandamus tocompel the respondent Commission to disclose said information# The Solicitor 1eneralchallenges the petitioner-s standing to sue upon the ground that the latter does not possess anylegal right to be informed of the civil services eligibilities of the government employeesconcerned# Sol1en further argues that there is no ministerial duty on the part of the Commissionto furnish the petitioner ,ith the information he see<s#

%ssue6

74" the petitioner has legal to access government records to validate the civil serviceeligibilities of the 9ealth Department employees#

9eld6

Civil Service Commission is ordered to open its register of eligible for the position of sanitarian)and to confirm or deny) the civil service eligibility of Gulian Sibonghanoy and Mariano Agas) forsaid position in the 9ealth Department of Cebu City) as re0uested by the petitioner &alentin :#:egaspi#

Aldovino vs Alunan1#R no# '> (

8acts6

The petitioners herein ,ere affected by reorgani*ing of Ministry of Tourism as provided inSection ? of 3/ecutive 4rder "o# ' > ,hich too< effect on Ganuary (>) '?@ # These 34

provides that incumbents ,hose positions are not included in the ne, position structure andstaffing pattern or ,ho are not reappointed are deemed separated from the service# Pursuant tothis) the Department of Tourism issued various office orders and memoranda declaring all

positions thereat vacant# To that effect) it leads to the separation of many of its employees

including the petitioners# The court had previously decided similar cases of Mandani) Abrogarand Arnaldo# The petitioners and intervenors claimed that they should not be deprived of theirlife granted to their former co$employees plead for reinstatement ,ithout the loss of seniorityrights# 8urthermore) they claimed for bac< salaries ,ill be computed under the ne, staffing

pattern from dates of their invalid termination at rates not lo,er than their former salaries# Thecourt aims to determine ,hether the separation of herein petitioners and intervenors from service,as pursuant to office orders and memoranda declared void in Mandani case) thus reinstatingand paying them ,ith their bac< ,ages#

Page 105: Nat Res Digested

8/10/2019 Nat Res Digested

http://slidepdf.com/reader/full/nat-res-digested 105/173

%ssues6

7hether or not the petitioners and intervenors must be reinstated and paid of their bac< ,ages#

9eld6

The Supreme Court ruled that herein petitioners are reinstated immediately to their former positions ,ithout loss of seniority rights and ,ith bac< salaries computed under ne, staffing pattern from the dates of their invalid dismissal at rates not lo,er than their former salaries butnot to e/ceed a period of E years ,ith several provisions# 9aving found out that the 3/ecutive4rder is unconstitutional) thus dismissal of the employees is also unconstitutional# The courtsdeclared its total nullity# An unconstitutional act is not a la,) it confers no rights) imposes noduties and affords no protection# %n legal contemplation) it is inoperative as if it had not been

passed#

Page 106: Nat Res Digested

8/10/2019 Nat Res Digested

http://slidepdf.com/reader/full/nat-res-digested 106/173

G.-. ,o. @%16 ) April 41& $'%4

2ICE, E ABA"AG& E A vs. 5E DI-EC "- "F A,DS& E A .

Facts!

In the year $660 a num#er o+ OBagosO or Igorots or non@Christians&num#ering at that time a#out thirty& *ere invited #ythe go#ernadorcillo and principalia o+ the then to*n o+ Alava& no* themunicipality o+ Sison& o+ the :rovince o+ :angasinan.

he OBagosO entered upon said land& too> possession o+ it and havecontinued to live upon the same and have cultivated it since that date. "nFe#ruary %6& $'$'& the LBagos /led an action& claiming the land *asregistered to them under the orrens system #ut *as later dismissed on thegrounds o+ lac> o+ evidence that they are entitled to the land.

Issue!

HK, the court is correct in dismissing the case on the grounds o+ lac> o+evidence.

5eld!

,o& the court is not correct in dismissing the case on the grounds o+ lac> o+evidence.

I+ *e *ere to loo> into the -oyal Decrees o+ Spain& as the attorney +or theappellants has done& *e *ill /nd that Spain did not assume to convert all thenative inha#itants o+ the :hilippines into trespassers o+ the land *hich theyoccupy& or even into tenants at *ill. In the -oyal Cedula o+ "cto#er $)&$ )0& *e /nd the +ollo*ing! OHhere such possessors shall not #e a#le toproduce title deeds& it shall #e su cient i+ they shall sho* that ancientpossession as a valid title #y prescription.O He may add that everypresumption o+ o*nership under the pu#lic land la*s o+ the :hilippine Islandsis in +avor o+ one actually occupying the land +or many years& and against theGovernment *hich see>s to deprive him o+ it& +or +ailure to comply *ithprovisions o+ su#se;uently enacted registration land act.

In addition to the +acts a#ove stated& the record sho*s that at the time o+ thedelivery o+ said parcel o+ land to the petitioners& it *as unoccupied andunimproved pu#lic land? that since their entry upon the possession o+ theland in the year $660& they and their ancestors have #een in the open&

Page 107: Nat Res Digested

8/10/2019 Nat Res Digested

http://slidepdf.com/reader/full/nat-res-digested 107/173

continuous& e clusive& and notorious possession and occupation o+ the same&#elieving in good +aith that they *ere the o*ners.

Hhere+ore& the court reversed its decision& thus the petitioners *herepermitted to present *hatever evidence they may have.

G.-. ,o. @0%4$ April $& $'16

CAS E B-" 5E-S& H" FE A,D S",S vs. G IE--ER 5E-MA,"S

Facts!

here *as a ver#al contract #et*een Tnight and Don eopoldo Criado #y*hich the de+endants agreed to sell the plainti s )11 #ales o+ Lgood currentManila hemp at :%0 a picul. hat& *hile some mention o+ mar>s *as made&Tnight said he could not accept those mar>s unless they turned out to #e upto the ;uality o+ Ogood current Manila.O 5o*ever& Don eopoldo Criado

testi/ed that the phrase Ogood currentO *as never mentioned in theconversation and that he stated to Mr. Tnight that the de+endants had ))0#ales o+ the +our mar>s mentioned& and that o+ those he sold )11 #ales.

Tnight<s testimony *as supported #y the testimony o+ 5iggin#otham *ho*as his assistant in the o ce and *ho heard the conversation& and *hostates that there *as a positive agreement on the part o+ Don eopoldoCriado to +urnish )11 #ales o+ Ogood current Manila.O

"n the )11 #ales& delivered& the plainti s accepted and paid +or %$1 #alesand no ;uestion as to these #ales on made in the case. hey re+used toaccept %'' #ales on the ground that it *as not good current Manila hemp&called upon the de+endants to +urnish %'' #ales o+ that ;uality and noti/edthem that& on +ailure to do so& they *ould #uy the same in the mar>et andcharge the increased cost o+ the de+endants. he de+endants re+used tosu#stitute other #ales and the plainti s #ought %'' #ales o+ good currentManila hemp at :%6.)1 a picul& :0.)1 more per picul than the price at *hich

Page 108: Nat Res Digested

8/10/2019 Nat Res Digested

http://slidepdf.com/reader/full/nat-res-digested 108/173

the de+endants had agreed to +urnish them that ;uality o+ hemp. he loss tothe plainti s *as there+ore :%&('$ and to recover that sum this action *as#rought.

hus the court ruled in +avor o+ the plainti s.

Issue!

HK, the ruling o+ the court is correct #asing +rom the evidence presented.

5eld!

es& the ruling o+ the court is correct.

Aside +rom Tnight and 5iggin#otham<s testimonies& the other evidence in*hich the court #ased its ruling is the letter received #y the plainti on

Septem#er $4& $'1( +rom :ortland Cordage Company& o+ "regon *hichstates that they are as>ing the plainti to o er them )11 #ales o+ goodcurrent eyte hemp& #ut the plainti s cannot +urnish )11 good current eytehemp& instead they o ered )11 #ales o+ good current Manila hemp. "n thesame day& #e+ore sending the letter& Tnight sa* Don eopoldo Criado *hotold him that they could +urnish )11 #ales o+ good current Manila Bales& thustheir ver#al contract. Don eopoldo Criado denies that he had theconversation *ith Tnight on that day& #ut the +act that the telegrams *heresent and received *as undenia#le.

G.-. ,o. @%0 '( June %6& $'(6

Director o+ Forestry vs. MuUo3

Facts!:iadeco claims to #e the o*ner o+ Some %&111 hectares o+ land 4 located inthe municipalities o+ Angat& ,or3agaray and San Jose del Monte& province o+Bulacan& and in Antipolo and Montal#an& province o+ -i3al. :iadeco=sevidence o+ o*nership consists o+ itulo de :ropiedad and a deed o+ a#solutesale.

Page 109: Nat Res Digested

8/10/2019 Nat Res Digested

http://slidepdf.com/reader/full/nat-res-digested 109/173

he controversy #egan *hen the Acting Director o+ Forestry Apolonio F.-ivera re;uired :iadeco to surrender the original certi/cate to him. Ground+or this cancellation *as that :iadeco had violated +orestry rules andregulations +or cutting trees *ithin the Angat and Mari>ina Hatershed-eservations& e pressly e cluded +rom the said certi/cate.

:iadeco /led a petition +or certiorari and prohi#ition *ith preliminaryin unction against the Director o+ Forestry& Forest Station Harden Mar;ue3and ,a*asa& essentially upon the averment that their acts hereto+orenarrated *ere Oall precipitate& ar#itrary& *himsical and capricious.O hepreliminary in unction *as granted and then he moved to declare the+orestry o cials in de+ault +or +ailure to ans*er its petition on time. he+orestry o cials as>ed the court to dismiss the petition upon the avermentsthat said court had no urisdiction over their persons or the su# ect matter o+the petition& and that administrative remedies have not yet #een e hausted#y :iadeco. "n the same date& too& #ut in a separate motion& said +orestryo cial as>ed +or a reconsideration o+ the lo*er court=s order grantingpreliminary in unction& #ottomed upon their charge that the illegal cutting o+trees #y :iadeco inside the Angat and Mari>ina Hatershed -eservations Q*hich are the main source o+ *ater supply o+ the City o+ Manila and itssurrounding to*ns and cities Q poses a grave danger o+ causing them to dryup to the pre udice and irrepara#le in ury o+ the inha#itants thereo+. he+orestry o cials *ere declared in de+ault.

:iadeco entered into an amica#le settlement *ith ,a*asa *here#y :iadeco=scase against ,a*asa *as *ithdra*n& the right o+ *ay granted #y ,a*asa to:iadeco remaining revo>ed and cancelled? and ,a*asa=s counterclaimagainst :iadeco *as also *ithdra*n in consideration o+ :$&()$.)' paid #y:iadeco to ,a*asa& representing the +ormer=s lia#ilities to the latter. hecourt approved o+ the amica#le settlement.

:iadeco applied +or the rene*al o+ its Certi/cate o+ :rivate Hoodland-egistration #ut *as denied #y Assistant Director o+ Forestry J. . tleg #ut:iadeco continued logging operations. It *as a#out this time that illegal

logging *as denounced #y some mem#ers o+ Congress there#y attractingnational attention. he Secretary o+ ,ational De+ense directed the Chie+ o+Sta o+ the Armed Forces to implement the re;uest. And& the Chie+ o+ Stadispatched a tas> +orce o+ the army into the Angat area& *hich impoundedand sei3ed all logs cut #y :iadeco and other loggers *hich *ere purportedlyconducting illegal operations and they made a private ;uarters on a portiono+ :iadeco<s land and prevented continuation o+ logging operations& +rom

Page 110: Nat Res Digested

8/10/2019 Nat Res Digested

http://slidepdf.com/reader/full/nat-res-digested 110/173

cutting and gathering o+ tim#er and other +orest products and en oyment o+said property. 5ence& :iadeco /led a complaint #ut *as denied.

Issue!

HK, :iadeco=s title is registra#le *ith the Bureau o+ Forestry

5eld!

,o. :iadeco<s title is not registara#le *ith the Breau o+ Forestry.

he pertinent statutory provision is Section $6%' o+ the -evisedAdministrative Code& vi3!

SEC. $6%'. -egistration o+ title to private +orest land. Q Every private o*nero+ land containing tim#er& /re*ood and other minor +orest products shallregister his title to the same *ith the Director o+ Forestry. A list o+ sucho*ners& *ith a statement o+ the #oundaries o+ their property& shall #e+urnished #y said Director to the Collector o+ Internal -evenue& and the sameshall #e supplemented +rom time to time as occasion may re;uire.

pon application o+ the Director o+ Forestry the /scal o+ the province in *hichany such land lies shall render assistance in the e amination o+ the titlethereo+ *ith a vie* to its registration in the Bureau o+ Forestry.

Ampliatory thereo+ is Section & Forestry Administrative "rder $%@$ o+ July $&$'0$& as amended #y Forestry Administrative "rder $%@%& *hich too> e ecton January $& $'(4. It reads!

. itles that may #e registered. Q "nly the +ollo*ing titles covering landscontaining tim#er& /re*ood and other minor +orest products may #eregistered under and pursuant to Section $6%' o+ the -evised AdministrativeCode?

7a8 Administrative titles granted #y the present Government& such ashomestead patent& +ree patent& and sales patent? and

Page 111: Nat Res Digested

8/10/2019 Nat Res Digested

http://slidepdf.com/reader/full/nat-res-digested 111/173

7#8 Judicial titles& such as orrens itle o#tained under the and -egistrationAct 7Act 0'(& as amended8 or under the Cadastral Act 7Act ,o. %%)'& asamended8.

he amendment o+ Forestry Administrative "rder $%@$ #y Forestry

Administrative "rder $%@% consisted in theomission o+ one paragraph&paragraph 7c8& *hich particulari3ed as one o+ the titles registra#le pursuantto Section $6%' o+ the -evised Administrative Code& O t itles granted #y theSpanish sovereignty in the islands and duly recogni3ed as valid titles underthe e isting la*s.O

In the case at #ar ho*ever& :iadeco<s title *as issued during the Spanishregime. And it is state in Section $6%'& does not descri#e *ith particularitytitles that may #e registered *ith the Bureau o+ Forestry. Spanish titles are;uite dissimilar to administrative and udicial titles under the present system.Although evidences o+ o*nership& these Spanish titles may #e lost thruprescription. hey are& there+ore& neither inde+easi#le nor imprescripti#le. Itshould not have #een allo*ed registration in the /rst place. "#viously&registration thereo+ can never #e rene*ed.

G.-. ,o. @%$6$0 July $)& $' )

5E DI-EC "- "F A,DS vs. ME ECI" ABA,RAD"& E A

Facts!

he Director o+ Forestry /led a petition to revie* a udgment in a land

registration proceeding& no decree having #een issued as yet& arose +rom its+ailure to accord him the opportunity to present his evidence to sho* thatthe land in controversy is part o+ a communal +orestand is thus non@disposa#le.

More speci/cally& *hat *as sought #y appellant pu#lic o cial in his amendedpetition +or revie* *as the reconsideration o+ a previous decision& reached

Page 112: Nat Res Digested

8/10/2019 Nat Res Digested

http://slidepdf.com/reader/full/nat-res-digested 112/173

Page 113: Nat Res Digested

8/10/2019 Nat Res Digested

http://slidepdf.com/reader/full/nat-res-digested 113/173

In the matter "3aeta -omulo 7July 41 $' '8

Facts!

:etitioners contend that the continued use o+ the name o+ a deceased or+ormer partner *hen permissi#le #y local custom& is not unethical #ut careshould #e ta>en that no imposition or deception is practiced through this use.

hey also contend that no local custom prohi#its the continued use o+ adeceased partner<s name in a pro+essional /rm<s name? there is no custom orusage in the :hilippines& or at least in the Greater Manila Area& *hichrecogni3es that the name o+ a la* /rm necessarily identi/es the individualmem#ers o+ the /rm.

*o separate :etitions *ere /led #e+ore this Court!

$8 #y the surviving partners o+ Atty. Ale ander Sycip& *ho died on May )&$' )& and

%8 #y the surviving partners o+ Atty. 5erminio "3aeta& *ho died on Fe#ruary$0& $' (& praying that they #e allo*ed to continue using& in the names o+their /rms& the names o+ partners *ho had passed a*ay. In the Court=s-esolution o+ Septem#er %& $' (& #oth :etitions *ere ordered consolidated.

Issue!

HK, the surviving partners may #e allo*ed #y the court to retain the nameo+ the partners *ho already passed a*ay in the name o+ the /rm

5eld!

,o. he surviving partners are not allo*ed to retain the name o+ thedeceased partner in the name o+ the /rm.

he Court #elieves that& in vie* o+ the personal and con/dential nature o+the relations #et*een attorney and client& and the high standards demanded

in the canons o+ pro+essional ethics& no practice should #e allo*ed *hicheven in a remote degree could give rise to the possi#ility o+ deception. Saidattorneys are accordingly advised to drop the names o+ the deceasedpartners +rom their /rm name.

Page 114: Nat Res Digested

8/10/2019 Nat Res Digested

http://slidepdf.com/reader/full/nat-res-digested 114/173

G.-. ,o. @4 '4 Fe#ruary $'& $'16

CI-I " MA:A vs. 5E I,S A- G"2E-,ME,

Facts!

he petitioner sought to have registered a tract o+ land o+ a#out $( hectaresin e tent& situated in the #arrio o+ San Antonio& in the district o+ Mandurriao&in the municipality o+ Iloilo. he petitioner presented evidence *hich appearsthat the land in ;uestion is lo*land& and has #een uninterruptedly& +or morethan t*enty years& in the possession o+ the petitioner and his ancestors aso*ners and the same has #een used during the said period& and up to thepresent& as /sh ponds& nipa lands& and salt deposits. he *itnesses declare

that the land is +ar +rom the sea& the to*n o+ Molo #eing #et*een the sea andthe said land. Judgment *as rendered in +avor o+ the petitioner and theGovernment has appealed.

Issue!

HK , the said property is an agricultural land.

5eld!

he ;uestion as to *hether the lands there involved *ere or *ere not

agricultural lands *ithin the meaning o+ the sections *as neither discussednor decided.

he decision o+ that court *as #ased upon Act ,o. '%( section )0& paragraph( *hich +ollo*s!

All persons *ho #y themselves or their predecessors in interest have #een inthe open& continuous e clusive& and notorious possession and occupation o+

Page 115: Nat Res Digested

8/10/2019 Nat Res Digested

http://slidepdf.com/reader/full/nat-res-digested 115/173

agricultural pu#lic lands& as de/ned #y said act o+ Congress o+ July /rst&nineteen hundred and t*o& under a #ona /de claim o+ o*nership e cept asagainst the Government& +or a period o+ ten years ne t preceding the ta>inge ect o+ this act& e cept *hen prevented #y *ar& or +orce ma eure& shall #econclusively presumed to have per+ormed all the conditions essential to aGovernment grant and to have received the same& and shall #e entitled to acerti/cate o+ title to such land under the provisions o+ this chapter.

he main phrase Lagricultural lands as de/ned #y said act o+ Congress o+ July $& is +ound not only in section )0 a#ove ;uoted #ut in other parts o+ Act,o. '%(& and it seems that the same construction must #e given to thephrase *herever it occurs in any part o+ that la*. Moreover& i+ it should #esaid that there is no de/nition in the act o+ Congress o+ the phraseOagricultural land&O *e do not see ho* any e ect could #e given to theprovisions o+ Act ,o. '$(& to *hich *e have re+erred. I+ the phrase is notde/ned in the act o+ Congress& then the lands upon *hich homesteads can#e granted cannot #e determined.

G.-. ,o. @4 $0 January %(& $'1'

ISABE " M", A," MA-CIA vs. 5E I,S A- G"2E-,ME, & E A .

Facts!

Isa#elo Montano presents a petition to the Court o+ and -egistration +or theinscription o+ a piece o+ land in the #arrio o+ i#is& municipality o+ Caloocan&used as a /shery. his petition *as opposed #y the Solicitor@General in #ehal+ o+ the Director o+ ands on the ground that the land in ;uestion #elonged tothe Government o+ the nited States& and the latter& that it *as the a#soluteo*ner o+ all the dry land along the eastern #oundary o+ the said /shery. heCourt o+ and -egistration in its decision o+ Decem#er $& $'1(& dismissed thesaid oppositions *ithout costs in +avor o+ Isa#elo Montano y Marcial.

Issue!

Page 116: Nat Res Digested

8/10/2019 Nat Res Digested

http://slidepdf.com/reader/full/nat-res-digested 116/173

HK, the property in ;uestion is an agricultural land.

5eld!

he property is an agricultural land

he section ;uoted is section )0& paragraph (& Act ,o. '%(& in *hich thephrase used is Oagricultural pu#lic lands.O

hroughout the opinion the phrase Opu#lic landsO is repeatedly ande clusively used. he entire discussion *as directed to the ;uestion as to*hether the property there in ;uestion #eing Opu#lic land&O it could #econsidered as agricultural pu#lic land and the conclusion reached is stated atpage $6%& as +ollo*s!

In other *ords& that the phrase Oagricultural land&O as used in Act ,o. '%(&

means those pu#lic lands ac;uired +rom Spain *hich are not tim#er ormineral lands.

In that case the land in ;uestion *as a long distance +rom the sea. In +act&the entire to*n o+ Molo *as #et*een it and the *ater. It could in no sense #ecalled tidal land. here+ore& the opinion *as devoted to a consideration o+ not*hat *ere Opu#lic landsO #ut *hether this particular tract *as or *as notagricultural pu#lic land. he ;uestion *hat the phrase Opu#lic landsO meantneither considered nor decided in that opinion& +or its resolution *as notnecessary. In the concurring opinion& ho*ever& that ;uestion *as discussedand it *as stated that the phrase Opu#lic landsO used in Act ,o. '%( must #einterpreted according to the American understanding o+ the *ords employedand the meaning o+ the terms as de/nitely / ed #y the decrees o+ the nitedStates Supreme Court.

G.-. ,o. @))'(1 ,ovem#er %0& $'66

Page 117: Nat Res Digested

8/10/2019 Nat Res Digested

http://slidepdf.com/reader/full/nat-res-digested 117/173

A" TEE& SRE S""T HA5& SRE AI C5"& and S C5 , E, vs.AIDA S @G",RA ES& MA, E S & E-ESI A S @BE-,ABE& -"D" F" S & and5","-AB E C" - "F A::EA S

Facts!

Sy Tiat& a Chinese national& died on January $ & $' leaving #ehindproperties here in the :hilippines.

herea+ter& Aida Sy@Gon3ales et al /led a petition +or the grant o+ letters o+administration alleging that they are the children o+ the deceased *ithAsuncion Gillego. he petition *as opposed #y ao Tee et al alleging that aoTee is the la*+ul *i+e o+ the deceased *hom he married in China. he trialcourt rendered decision in +avor o+ the opposition. "n appeal & the Court o+Appeals rendered a decision& modi+ying the decision declaring the marriage

o+ Sy Tiat to ao Tee as not has #een proven valid in accordance *ith thela*s o+ China. 5ence& #oth parties moved +or reconsideration to *hich theSupreme Court granted.

ISS E!

HK, the marriage o+ ao Tee and Sy Tiat is valid in accordance *ith:hilippine la*s.

5E D!

Hell@esta#lished in this urisdiction is the principle that:hilippine courts cannot ta>e udicial notice o+ +oreign la*s. hey must #ealleged and proven as any other +act. o esta#lish the validity o+ marriage&the e istence o+ +oreign la* as a ;uestion o+ +act and the alleged marriagemust #e proven #y clear and convincing evidence.

For +ailure to prove the +oreign la* or custom and conse;uently o+ themarriage& the marriage #et*een ao Tee and Sy Tiat in China cannot #erecogni3ed in the urisdiction o+ :hilippine courts.

Page 118: Nat Res Digested

8/10/2019 Nat Res Digested

http://slidepdf.com/reader/full/nat-res-digested 118/173

G.-. ,o. %6(' March %)& $'1

MA E" CA-IV" vs. 5E I,S A- G"2E-,ME,

Facts!

Carino is an Igorot o+ the :rovince o+ Benguet& *here the land lies /led +or*rit o+ error #ecause the CFI and SC dismissed his petition +or application.For more than )1 years #e+ore the reaty o+ :aris& April $$& $6''& he and hisancestors had held the land as recogni3ed o*ners #y the Igorots. CariUo

inherited the land in accordance *ith Igorot custom. 5e tried to have theland ad usted under the Spanish land la*s& #ut no document issued +rom theSpanish Cro*n. In $'1$& CariUo o#tained a possessory title to the land underthe Spanish Mortgage a*. he ,orth American colonial government&ho*ever& ignored his possessory title and #uilt a pu#lic road on the landprompting him to see> a orrens title to his property in the land registrationcourt.

Issue!

HK, the petitioner is granted o*nership o+ the land.5eld!

es. he petitioner is entitled to o*nership o+ said land.

he petitioner=s possession *as not unla*+ul& and no attempt at any suchproceedings against him or his +ather ever *as made. And also under the

Page 119: Nat Res Digested

8/10/2019 Nat Res Digested

http://slidepdf.com/reader/full/nat-res-digested 119/173

Spanish a*! OHhere such possessors shall not #e a#le to produce titledeeds& it shall #e su cient i+ they shall sho* that ancient possession& as avalid title #y prescription.O For cultivated land& %1 years& uninterrupted& isenough. For uncultivated& 41.

G.-. ,os. @4$(((& @4$(( and @4$((6 April 41& $' '

E:A, " vs. D M ,G

Facts!

he director o+ lands /led a criminal case against the de+endants on theground o+ misrepresentation and +alse data and in+ormation. he de+endantsin the three cases /led an amended oint ans*er *ith counterclaim to thecomplaint in intervention. he de+endants /led a motion to dismiss the same

on the ground that the accused had complied *ith all the legal re;uirementsin the ac;uisition o+ their patents *hich *ere duly issued #y the Director o+ands and that they are not guilty o+ the alleged +alsi/cation o+ pu#licdocuments.

Issue!

HK, the de+endants are entitled to o*nership o+ the land.

Page 120: Nat Res Digested

8/10/2019 Nat Res Digested

http://slidepdf.com/reader/full/nat-res-digested 120/173

5eld!

es. he De+endants are entitled to o*nership o+ the land in ;uestion.

Section 00 o+ the and Act in its second paragraph states!

A mem#er o+ the national cultural& minorities *ho has continuously occupiedand cultivated& either #y himsel+ or through his predecessors@in@ interest& atract or tracts o+ land& *hether disposa#le or not since July 0& $'))& shall #eentitled to the right granted in the preceding paragraph o+ this section!:-"2IDED& that at the time he /les his +ree patent application& he is not theo*ner o+ any real property secured or disposa#le under this provision o+ the:u#lic and a*.

It is +or this reason Q that is& to give these national cultural minorities *ho*ere driven +rom their ancestral a#odes& a +air chance to ac;uire lands o+ thepu#lic domain.

G.-. ,o. @4 (6% March %'& $' 0

-E: B IC "F 5E :5I I::I,ES vs. 5",. :ED-" SAMS", A,IMAS

Facts!

Page 121: Nat Res Digested

8/10/2019 Nat Res Digested

http://slidepdf.com/reader/full/nat-res-digested 121/173

here *as a land in General Santos City *hich *as claimed #y Isagani Du im#ol. he land covered #y the +ree patent and title in ;uestion *asoriginally applied +or #y :recila Soria& *ho trans+erred her rights to the landand its improvements to de+endant Isagani Du im#ol *ho /led hisapplication as a trans+eree +rom :recila Soria.

Bureau o+ Forestry& /led a complaint in the Court o+ First Instance o+Cota#ato& Branch I& General Santos City to declare the +ree patent in thename o+ de+endant Isagani Du im#ol null and void a# initio and to order thereversion o+ the land in ;uestion to the mass o+ pu#lic domain. he action is#ased on the ground that the land covered there#y is a +orest or tim#er land*hich is not disposa#le under the :u#lic and Act. And claimed that the saidpatent and title *ere o#tained +raudulently as private respondent Isagani Du

im#ol never occupied and cultivated the land applied +or. he case *asdismissed.

Issue!

HK, 5on. Animas< decision is correct.

5eld!

,o. he First decision *as incorrect.

he complaint alleges that applicant Isagani Du im#ol<s actions constitutesas +raud.

A certi/cate o+ title that is void may #e ordered cancelled. A title *ill #econsidered void i+ it is procured through +raud& as *hen a person applies +orregistration o+ the land under his name although the property #elongs toanother. In the case o+ disposa#le pu#lic lands& +ailure on the part o+ thegrantee to comply *ith the conditions imposed #y la* is a ground +or holdingsuch title void.

Page 122: Nat Res Digested

8/10/2019 Nat Res Digested

http://slidepdf.com/reader/full/nat-res-digested 122/173

G- ,o. $$('& Decem#er %%& $'66

Sangalang E . Al v. IAC&

Facts!

Sangalang /led an action to en+orce #y speci/c per+ormance restrictiveeasement upon property& speci/cally the Bel@ Air 2illage su#division inMa>ati& Metro Manila& pursuant to stipulations em#odied in the deeds o+ salecovering the su#division& and +or damages. Bel @ Air itsel+ had #rought itso*n complaints& +our in num#er& li>e*ise +or speci/c per+ormance anddamages to en+orce the same =deed restrictions.= he court ruled in +avor o+the planti s. he IAC reversed the decision o+ the court.

Issue!

HK, the IAC erred in the decision o+ the case at #ar.

5eld!

,o. he IAC did not commit any error in the ruling o+ the case at #ar.

he petitioners have not sho*n *hy *e should hold other*ise other than +orthe supposed Onon@impairmentO guaranty o+ the Constitution& *hich& as *ehave declared& is secondary to the more compelling interests o+ general*el+are. he "rdinance has not #een sho*n to #e capricious or ar#itrary orunreasona#le to *arrant the reversal o+ the udgments so appealed. In thatconnection& *e /nd no reversi#le error to have #een committed #y the Courto+ Appeals.

Page 123: Nat Res Digested

8/10/2019 Nat Res Digested

http://slidepdf.com/reader/full/nat-res-digested 123/173

G.-. ,o. @04%14 July %'& $'

J"SE C. C-IS "BA vs. A EJA,D-" ME C5"- and FEDE-IC" A-CA A

Facts!

he plainti *as +ormerly employed as a private secretary in the :resident=s:rivate " ce& MalacaUang& Manila.

Five o+ the employees *ho *ere separated not including the herein plainti/led a civil and *ere reinstatement and the payment o+ their salaries. heplainti sent a letter to the " ce o+ the :resident re;uesting reinstatementto his +ormer position and the payment o+ salary #ut the re;uest *as deniedrepeatedly until he received a letter *hich declared the matter Lde/nitelyclosed . he plainti then /led a complaint against the E ecutive Secretary&Ale andro Melchor and Federico Arcala& cash dis#ursing o cer& " ce o+ the:resident o+ the :hilippines. he de+endants argued that the plainti had nocause o+ action as he is deemed to have a#andoned his o ce +or +ailure toinstitute the proper proceedings to assert his right *ithin one year +rom thedate o+ separation pursuant to Sec. $(& -ule (( o+ the -ules o+ Court& he

having come to court only a+ter the lapse o+ more than nine years& there#y ine ect ac;uiescing to his separation& and there+ore he is not entitled to anysalary +rom termination o+ his employment. he complaint *as dismissed.

Issue!

HK, the court erred in dismissing the case.

Page 124: Nat Res Digested

8/10/2019 Nat Res Digested

http://slidepdf.com/reader/full/nat-res-digested 124/173

5eld!

es. he court made an error in dismissing the case.

here *as no ac;uiescence to or inaction on the part o+ Jose Cristo#al

amounting to a#andonment o+ his right to reinstatement in o ce. AlthoughCristo#al +ailed to /le his complaint *ithin one year +rom the date o+separation #ut& it is claimed& he allo*ed almost nine years passing #e+orecoming to court #y reason o+ *hich he is deemed to have ac;uiesced to hisremoval. he Court stated that in a general sense& laches is +ailure orneglect& +or an unreasona#le and une plained length o+ time& to do that*hich& #y e ercising due diligence& could or should have #een done earlier? itis negligence or omission to assert a right *ithin a reasona#le time&*arranting a presumption that the party entitled to assert it either hasa#andoned it or declined to assert it. And it is the doctrine o+ laches *hich isinvo>ed to de+eat Jose Cristo#al=s suit& there are e ceptional circumstancesattending *hich ta>e this case out o+ the rule enunciated a#ove and lead usto grant relie+ to appellant. hese are!

@ here *as no ac;uiescence to or inaction on the part o+ Jose Cristo#alamounting to a#andonment o+ his right to reinstatement in o ce.

@It *as an act o+ the government through its responsi#le o cials moreparticularly then E ecutive Secretary Amelito Mutuc and his successors*hich contri#uted to the alleged delay in the /ling o+ Cristo#al=s present

complaint +or reinstatement.@ he dismissal o+ appellant Cristo#al *as contrary to la* on the strength o+this Court=s Decision.

Hhere+ore& the court ordered the reinstatement and payment o+ #ac> *ageso+ the plainti .

G.-. ,o. @4($0% March 4$& $' 4

JA2E A,A 2S. A,

Facts!

In $' 4& Marcos ordered the immediate implementation o+ the ne* $' 4Constitution. Javellana& a Filipino and a registered voter sought to en oin theE ec Sec and other ca#inet secretaries +rom implementing the said

Page 125: Nat Res Digested

8/10/2019 Nat Res Digested

http://slidepdf.com/reader/full/nat-res-digested 125/173

constitution. Javellana averred that the said constitution is void #ecause thesame *as initiated #y the president. 5e argued that the :resident is *Kopo*er to proclaim the rati/cation #y the Filipino people o+ the proposedconstitution. Further& the election held to rati+y such constitution is not a +reeelection there #eing intimidation and +raud.

ISS E!

HK, the SC must give due course to the petition.

5E D!

he SC ruled that they cannot rule upon the case at #ar. Ma ority o+ the SC ustices e pressed the vie* that they *ere concluded #y the ascertainmentmade #y the president o+ the :hilippines& in the e ercise o+ his politicalprerogatives. Further& there #eing no competent evidence to sho* such +raudand intimidation during the election& it is to #e assumed that the people hadac;uiesced in or accepted the $' 4 Constitution. he ;uestion o+ the validityo+ the $' 4 Constitution is a political ;uestion *hich *as le+t to the people intheir sovereign capacity to ans*er. heir rati/cation o+ the same had sho*nsuch ac;uiescence.

Page 126: Nat Res Digested

8/10/2019 Nat Res Digested

http://slidepdf.com/reader/full/nat-res-digested 126/173

G.-. ,o. 6$4$$ June 41& $'66

TA:A I-A, ,G MGA ,AG I I,GT"D SA :AMA5A AA, 2S. A,

Facts!

E" % 4 *as issued #y the :resident o+ the :hilippines *hich amended the-evenue Code& adopting the value@added ta 72A 8 e ective $ January $'66.Four petitions assailed the validity o+ the 2A a* +or #eing #eyond the:resident to enact? +or #eing oppressive& discriminatory& regressive& andviolative o+ the due process and e;ual protection clauses& among others& o+the Constitution. he Integrated Customs Bro>ers Association particularlycontend that it unduly discriminate against customs #ro>ers 7Section $14 r 8as the amended provision o+ the a Code provides that Lservice per+ormedin the e ercise o+ pro+ession or calling 7e cept custom #ro>ers8 su# ect to

occupational ta under the ocal a Code& and pro+essional servicesper+ormed #y registered general pro+essional partnerships are e empt +rom2A.

Issue!

Hhether the E@2A la* discriminates against customs #ro>ers.

5eld!

he phrase Le cept custom #ro>ers is not meant to discriminate against

custom #ro>ers #ut to avert a potential con ict #et*een Sections $1% and$14 o+ the a Code& as amended. he distinction o+ the customs #ro>ers+rom the other pro+essionals *ho are su# ect to occupation ta under theocal a Code is #ased upon material di erences& in that the activities o+customs #ro>ers parta>e more o+ a #usiness& rather than a pro+ession and*ere thus su# ected to the percentage ta under Section $ 0 o+ the a Codeprior to its amendment #y E" % 4. E" % 4 a#olished the percentage ta

Page 127: Nat Res Digested

8/10/2019 Nat Res Digested

http://slidepdf.com/reader/full/nat-res-digested 127/173

and replaced it *ith the 2A . I+ the Association did not protestthe classi/cation o+ customs #ro>ers then& there is no reason *hy it shouldprotest no*.

G.-. ,o. @%4$4( August %(& $' 0

MA 5A et al vs. 5E C",S" IDA ED BA,T A,D - S C"M:A, et al

Facts!

he plainti /led a class suit against the de+endant #an> on the ground o+#reach o+ contract #et*een the plainti and de+endant #an> and O+alsely

certi/ed to the calling o+ a special stoc>holders= meeting allegedly pursuantto due notice and call o+ De+endant Ban>O *ithout noti+ying the plainti s andother stoc>holders. he de+endants moved +or the dismissal o+ the action onthe ground o+ the plainti s@appellants had no legal standing or capacity toinstitute the alleged class suit? that the complaint did not state a su cientand valid cause o+ action? and that plainti s@appellants= complaint againstthe increase o+ the num#er o+ directors did not li>e*ise state a cause o+action. hus the court dismissed the case. he plainti s appealed.

Issue!

HK, the plainti s have su cient cause o+ action.

5eld!

,o& there *as no su cient cause o+ action.It having #een sho*n that the complaint +ailed to state ultimate +acts to

Page 128: Nat Res Digested

8/10/2019 Nat Res Digested

http://slidepdf.com/reader/full/nat-res-digested 128/173

constitute a cause o+ action& it #ecomes unnecessary to discuss the otherassignments o+ errors.

G.-. ,o. @0)'6 May )& $'4'

5E :E": E "F 5E :5I I::I,ES vs. CAA

Facts!

In $'4 & there e ists a la* 7Act $(4'8 *hich #ars native non@ Christians +romdrin>ing gin or any other li;uor outside o+ their customary alcoholic drin>s.Cayat& a native o+ the Cordillera& *as caught *ith an A@$@$ gin in violation o+this Act. 5e *as then charged and sentenced to pay :).11 and to #eimprisoned in case o+ insolvency. Cayat admitted his guilt #ut he challengedthe constitutionality o+ the said Act. 5e averred& among others& that itviolated his right to e;ual protection a orded #y the constitution. 5e saidthis attempt to treat them *ith discrimination or Lmar> them as in+erior orless capa#le race and less entitled *ill meet *ith their instant challenge.

Page 129: Nat Res Digested

8/10/2019 Nat Res Digested

http://slidepdf.com/reader/full/nat-res-digested 129/173

he la* sought to distinguish and classi+y native non@Christians +romChristians.

ISS E! HK, the said Act violates the e;ual protection clause.

5E D! he SC ruled that Act $(4' is valid +or it met the re;uisites o+ areasona#le classi/cation. he SC emphasi3ed that it is not enough that themem#ers o+ a group have the characteristics that distinguish them +romothers. he classi/cation must& as an indispensa#le re;uisite& not #ear#itrary. he re;uisites to #e complied *ith are?

7$8 Must rest on su#stantial distinctions?

7%8 Must #e germane to the purposes o+ the la*?

748 Must not #e limited to e isting conditions only? and

708 Must apply e;ually to all mem#ers o+ the same class.

Act ,o. $(4' satis/es these re;uirements. he classi/cation rests on real orsu#stantial& not merely imaginary or *himsical& distinctions. It is not #asedupon Laccident o+ #irth or parentage. he la*& then& does not see> to mar>the non@Christian tri#es as Lan in+erior or less capa#le race. "n thecontrary& all measures thus +ar adopted in the promotion o+ the pu#lic policyto*ards them rest upon a recognition o+ their inherent right to e;uality in theen oyment o+ those privileges no* en oyed #y their Christian #rothers. But as

there can #e no true e;uality #e+ore the la*& i+ there is& in +act& no e;uality ineducation& the government has endeavored& #y appropriate measures& toraise their culture and civili3ation and secure +or them the #ene/ts o+ theirprogress& *ith the ultimate end in vie* o+ placing them *iththeir Christian #rothers on the #asis o+ true e;uality.

Page 130: Nat Res Digested

8/10/2019 Nat Res Digested

http://slidepdf.com/reader/full/nat-res-digested 130/173

G.-. ,o. @$01 6 March & $'$'

- BI& E A . 7manguianes8 vs. 5E :-"2I,CIA B"A-D "F MI,D"-"

Facts!

he provincial #oard o+ Mindoro adopted resolution ,o. %) *herein non@Christian inha#itants 7uncivili3ed tri#es8 *ill #e directed to ta>e up theirha#itation on sites on unoccupied pu#lic lands. It is resolved that undersection %1 o+ the Administrative Code& 611 hectares o+ pu#lic land in thesitio o+ ig#ao on ,au an a>e #e selected as a site +or the permanentsettlement o+ Mangyanes in Mindoro. Further& Mangyans may only solicithomesteads on thisreservation providing that saidhomestead applications are previously recommended #y the provincialgovernor.

In that case& pursuant to Section %$0) o+ the -evised Administrative Code& allthe Mangyans in the to*nships o+ ,au an and :ola and the Mangyans east o+the Baco -iver including those in the districts o+ Dulangan and -u#i=s place inCalapan& *ere ordered to ta>e up their ha#itation on the site o+ ig#ao&,au an a>e. Also& that any Mangyan *ho shall re+use to comply *ith thisorder shall upon conviction #e imprisoned not e ceed in si ty days& inaccordance *ith section % )' o+ the revised Administrative Code.

Said resolution o+ the provincial #oard o+ Mindoro *ere claimed as necessarymeasures +or the protection o+ the Mangyanes o+ Mindoro as *ell as theprotection o+ pu#lic +orests in *hich they roam& and to introduce civili3edcustoms among them.

It appeared that -u#i and those living in his rancheria have not / ed theird*elling *ithin the reservation o+ ig#ao and are lia#le to #e punished.

It is alleged that the Manguianes are #eing illegally deprived o+ their li#erty#y the provincial o cials o+ that province. -u#i and his companions are said

to #e held on the reservation esta#lished at ig#ao& Mindoro& against their*ill& and one Da#alos is said to #e held under the custody o+ theprovincial sheri in the prison at Calapan +or having run a*ay +romthe reservation.

Issue! Hhether or ,ot Section %$0) o+ the Administrative Code deprives a

Page 131: Nat Res Digested

8/10/2019 Nat Res Digested

http://slidepdf.com/reader/full/nat-res-digested 131/173

person o+ his li#erty o+ a#ode. hus& H", Section %$0) o+ the AdministrativeCode o+ $'$ is constitutional.

5eld! he Court held that section %$0) o+ the Administrative Code does notdeprive a person o+ his li#erty o+ a#ode and does not deny to him the e;ualprotection o+ the la*s& and that con/nement in reservations in accordance*ith said section does not constitute slavery and involuntary servitude. heCourt is +urther o+ the opinion that section %$0) o+ the Administrative Code isa legitimate e ertion o+ the police po*er. Section %$0) o+ the AdministrativeCode o+ $'$ is constitutional.

Assigned as reasons +or the action!

7$8 Attempts +or the advancement o+ the non@Christian people o+ theprovince? and

7%8 he only success+ully method +or educating the Manguianes *as to o#ligethem to live in a permanent settlement. he Solicitor@General adds the+ollo*ing?

748 he protection o+ the Manguianes?

708 he protection o+ the pu#lic +orests in *hich they roam?

7)8 he necessity o+ introducing civili3ed customs among the Manguianes.

"ne cannot hold that the li#erty o+ the citi3en is unduly inter+ered *ithout*hen the degree o+ civili3ation o+ the Manguianes is considered. hey arerestrained +or their o*n good and the general good o+ the :hilippines.

L i#erty regulated #y la*O! Implied in the term is restraint #y la* +or thegood o+ the individual and +or the greater good o+ the peace and order o+society and the general *ell@#eing. ,o man can do e actly as he pleases.

,one o+ the rights o+ the citi3en can #e ta>en a*ay e cept #y due process o+la*.

here+ore& petitioners are not unla*+ully imprisoned or restrained o+ theirli#erty. 5a#eas corpus can& there+ore& not issue.

Page 132: Nat Res Digested

8/10/2019 Nat Res Digested

http://slidepdf.com/reader/full/nat-res-digested 132/173

G.-. ,o. @)$ 4 May $(& $'61

. C" . -"D-IG" S. DE G RMA, and :E": E "F 5E :5I I::I,ESvs. M ,ICI:A CI-C I J DGE MA-CE I," M. ESCA ",A& F "-E, I,"-"D-IG"& and MA-IA," DA DA .

Facts!

he de+endants Florentino -odrigo and Mariano Dayday *ere charged *ithOIllegal :ossession o+ E plosive locally >no*n as =dinamita=. Hhile in thesea*aters o+ the Ce#u& con+ederating and mutually helping *ith one another&*ithout authority o+ the la* and *ithout proper permit +rom authorities& didthen and there *ill+ully& unla*+ully& and +eloniously possess& >eep ane plosive& locally >no*n as =DI,AMI A= in their #anca purposely +or use o+illegal /shing and three 748 #ottles o+ e plosives& t*o 7%8 paddles& t*o 7%8/shnets locally >no*n as OSIB" O and one 7$8 #anca *ere recovered +rom

Page 133: Nat Res Digested

8/10/2019 Nat Res Digested

http://slidepdf.com/reader/full/nat-res-digested 133/173

Page 134: Nat Res Digested

8/10/2019 Nat Res Digested

http://slidepdf.com/reader/full/nat-res-digested 134/173

es. Considering that the Municipal Circuit Court lac>ed competent urisdiction over the su# ect matter o+ the criminal complaint against theaccused respondents.

:residential Decree ,o. $1)6 is an amendatory decree& *hich increased the

penalties +or certain +orms o+ illegal /shing and +or other acts madepunisha#le under :residential Decree ,o. 10 or the OFisheries Decree o+$' )O. he pertinent portion o+ Section 44 o+ :residential Decree ,o. 10& asamended #y :residential Decree ,o. $1)6 reads!

Sec. 44. Illegal /shing? illegal possession o+ e plosives intended +or illegal/shing? dealing in illegally caught /sh or /sheryKa;uatic products. @ It shall #eunla*+ul +or any person to catch& ta>e or gather or cause to #e caught& ta>engathered /sh or /sheriesKa;uatic products in :hilippine *aters *ith the useo+ e plosives& o#no ious or poisonous su#stance& or #y the use o+ electricityas de/ned in paragraphs 7$8& 7m8 and 7d8& respectively& o+ Section 4 hereo+!:rovided& that possession o+ such e plosives *ith intent to use the same +orillegal /shing as herein de/ned shall #e punisha#le as hereina+terprovided. ... 7Emphasis supplied8.

Section 46& su#section a 7$8 o+ :residential Decree ,o. 10& as amended #y:residential Decree ,o. $1)6& correspondingly provides!

7$8 By the penalty o+ imprisonment ranging +rom t*elve 7$%8 years tot*enty@/ve 7%)8 years in the case o+ mere possession o+ e plosives intended

+or illegal /shing. ... 7Emphasis supplied8.As correctly pointed out #y the Solicitor General in the Comment he /led +orpetitioner :eople o+ the :hilippines& respondent Judge=s re+erence to:residential Decree ,o. ' is misplaced +or& indeed& there is no mention at allo+& nor any re+erence to& :residential Decree ,o. ' in the Complaint.

Page 135: Nat Res Digested

8/10/2019 Nat Res Digested

http://slidepdf.com/reader/full/nat-res-digested 135/173

G.-. ,o. @%)040 July %)& $' )

5","-AB E A-SE,I" ,. -" DA,& J-.& in his capacity as ActingCommissioner& :hilippine Fisheries Commission& and 5E :5I I::I,E ,A2

vs.5","-AB E F-A,CISC" A-CA& as :residing Judge o+ the Court o+ FirstInstance o+ Manila 7Branch $8 and M"-ABE& DE G RMA, W C"M:A,

Facts!

"n August ) or (& $'()& the t*o /shing #oats& ony e 2I and ony e III&also respectively called Srta. Hinnie and Srta. Agnes& *ere actually sei3ed +orillegal /shing *ith dynamite. Fish caught *ith dynamite and stic>s o+dynamite *ere then +ound a#oard the t*o vessels.

It *as alleged that at the time o+ the sei3ure o+ the /shing #oats in issue& thesame *ere engaged in legitimate /shing operations o the coast o+ :ala*an?that #y virtue o+ the o er o+ compromise dated Septem#er $4& $'() #yrespondent company to the Secretary o+ Agriculture and ,atural -esources&the numerous violations o+ the Fishery a*s& i+ any& #y the cre* mem#ers o+the vessels *ere settled. 5o*ever& the ships *ere apprehended *ithout*arrant +or alleged violations o+ some provisions o+ the Fisheries Act and therules and regulations promulgated there under.

-espondent /led *ith the Court against petitioner Fisheries CommissionerArsenio ,. -oldan& Jr.& +or the recovery o+ /shing vessel ony e 2I 7one o+t*o /shing #oats in ;uestion8 *hich had #een sei3ed and impounded #ypetitioner Fisheries Commissioner through the :hilippine ,avy. he courtdismissed the complaint +or +ailure o+ the petitioner to prosecute and +ailureo+ the de+endant to appear.

Page 136: Nat Res Digested

8/10/2019 Nat Res Digested

http://slidepdf.com/reader/full/nat-res-digested 136/173

It *as held that 5on. -oldan acted *ithout urisdiction and *ith grave a#useo+ discretion.

Issue!

HK, is it la*+ul to apprehend /shing #oats *ithout *arrant.5eld!

es. In the case at #ar& it is la*+ul to apprehend the /shing #oats *ithout*arrant.

he *ord #oat in its ordinary sense& means any *ater cra+t& the /shing #oats ony e III and ony e 2I are li>e*ise vessels *ithin the meaning o+ theterm vessel. the accepted de/nition o+ vessel includes Oevery description o+*ater cra+t& large or small& used or capa#le o+ #eing used as a means o+

transportation on *aterOSearch and sei3ure *ithout search *arrant o+ vessels and aircra+ts +orviolations o+ the customs la*s have #een the traditional e ception to theconstitutional re;uirement o+ a search *arrant& #ecause the vessel can #e;uic>ly moved out o+ the locality or urisdiction in *hich the search *arrantmust #e sought #e+ore such *arrant could #e secured? hence it is notpractica#le to re;uire a search *arrant #e+ore such search or sei3ure can #econstitutionally e ected.

Since the cre* o+ certain /shing vessels *ere caught& in agrante& illegally/shing *ith dynamite and *ithout the re;uisite license& their apprehension*ithout a *arrant o+ arrest and the sei3ure o+ the vessel& as *ell as itse;uipment and the dynamites +ound therein& as an incident to a la*+ul arrest*as held to #e la*+ul.

G.-. ,o. @'('' August %(& $'$)

5E ,I ED S A ES vs. J A, 5E-,A,DER& E A .

Facts!

In $'0 & i#erato Jimene3 *as appointed as a temporary legal investigator inthe :hilippine 2eterans Board 7:2B8. In $'0'& he *as promoted as theChie+ o+ the Investigation Section #ut still in a temporary capacity #ecausehe is not civil service eligi#le. In $')1& he too> a promotional civil servicee am. In July $')$& Jimene3 received a letter +rom :2B Chairman Gen.Guillermo Francisco advising him that he is #eing replaced #y a civil service

Page 137: Nat Res Digested

8/10/2019 Nat Res Digested

http://slidepdf.com/reader/full/nat-res-digested 137/173

eligi#le. In Septem#er $')$& Jimene3 received the results o+ the civil servicee am he too> in $')1? he passed. 5e then appealed his separation +romservice.

ISS E!

HK, Jimene3 should #e reinstated.

5E D!

,o. In +act& he should have #een separated +rom the service even #e+ore$')$. nder the la*& he *as supposed to only hold such temporaryappointment +or three months *hile the appointing po*er is still loo>ing +or acivil service eligi#le. 5is e tended stay in the service is only upon the graceo+ the appointing po*er. Further& there is no la* *hich provides that atemporary appointment may ripen to a permanent one. Hhen he met thecivil service eligi#ility& Jimene3 did not #ecome entitled to a permanentposition in the :2D. he po*er to appoint is in essence discretionary on thepart o+ the proper authority& in this case the head o+ the department. heappointing po*er has the right o+ choice *hich he may e ercise +reelyaccording to his udgment& deciding +or himsel+ *ho is #est ;uali/ed +or anycompetitive position in the Civil Service. Mere certi/cation as a civil serviceeligi#le does not amount to an appointment. he Civil Service Commissiondoes not insure any appointment? it only certi/es an eligi#le to #e possessedo+ the ;uali/cation as re;uired +or a position classi/ed under its rules.

Page 138: Nat Res Digested

8/10/2019 Nat Res Digested

http://slidepdf.com/reader/full/nat-res-digested 138/173

G.R. No. 1=2644 F%/r&ar0 1", 2""#

%OHN ERIC ONE , STE$EN PAU REID ( #. HERNANDE> , %'('(o %rs,

6 v%rs&s 6

PEOP E O3 THE PHI IPPINES , R%s+o )% '.

FACTS:%'('(o %rs ?o Er(c Lo %0, S'%v% a&- R%(), a ) %)ro . %r a )%3 ar% ' %r%s()% ' a ) C (% E$%c&'(v% O (c%r, S% (or Ma a %r, a ) R%s()% ' Ma a %r or

M( ( O+%ra'(o s, r%s+%c'(v%-0, o Marco++%r M( ( Cor+ora'(o = Marco++%r;>, acor+ora'(o % a %) ( ( ( ( ' % +rov( c% o Mar( )&4&%.

Page 139: Nat Res Digested

8/10/2019 Nat Res Digested

http://slidepdf.com/reader/full/nat-res-digested 139/173

Page 140: Nat Res Digested

8/10/2019 Nat Res Digested

http://slidepdf.com/reader/full/nat-res-digested 140/173

T %r% (s )&+-(c('0 =or &-'(+-(c('0> o c ar %s % as) g5/ I ;o*+(t)o '(*g/s +o*/t'( o / o;;/ s/. U )%r S%c'(o !=%>, R&-% 117 o ' % 1*85 R&-%s o Cr( ( a-roc%)&r%, )&+-(c('0 o o % s%s ( a s( -% ( or a'(o (s a ro& ) 'o 4&as ' %

I or a'(o . T % R&-%s +ro (/(' ' % (-( o s&c I or a'(o 'o avo() co &s( ' %acc&s%) ( +r%+ar( (s )% % s%. %r%, o %v%r, ' % +ros%c&'(o c ar %) %ac

+%'('(o %r (' o&r o % s%s, (' /( ' I ;o*+(t)o '(*g) g o 5 o / o;;/ s/ . T &s,+%'('(o %rs %rro %o&s-0 ( vo<% )&+-(c('0 o c ar %s as a ro& ) 'o 4&as ' %I or a'(o s. O ' (s scor% a-o %, ' % +%'('(o )%s%rv%s o&'r( ' )% (a-.

G.R. No. 1!*598. D%c% /%r 22, 2"""

MARCO ER MINING COR ORATION, petitioner,

vs.

AL ERTO G. UMOLO %' a-., , respondents .

FACTS:

MARCO ER MINING COR ORATION r% (s'%r%) ('s ( ( c-a( s ( ao, Bas(/&,N&%va (3ca0a (' ' % DENR ro F%/r&ar0 "2,1*82 'o Oc'o/%r 12, 1*82. r(va'%r%s+o )% 's A-/%r'o G. & o-o a ) o' %rs r% (s'%r%) ' %(r ( ( c-a( s ( ' % sa %ar%a ro 28 ?&-0 1*81 'o 22 S%+'% /%r 1*88, (c c-a( s %r% s&/s%4&% '-0co v%r'%) ( 'o M( %ra- ro)&c'(o S ar( A r%% % 's =M SA>.

Page 141: Nat Res Digested

8/10/2019 Nat Res Digested

http://slidepdf.com/reader/full/nat-res-digested 141/173

O Marc 12, 1*82 +%'('(o %r % '%r%) ( 'o O+'(o A r%% % 's ov%r ' % ( ( . U )%r ' % A r%% % 's, +%'('(o %r as ra '%) ' % %$c-&s(v% a ) (rr%voca/-% r( ' 'o %$+-or%' % ( ( c-a( s or ' r%% =!> 0%ars (' +rov(s(o or %$'% s(o .

O D%c% /%r 2!, 1*82 a ) Marc 2#, 1*87 +%'('(o %r (-%) ros+%c'( %r (' A++-(ca'(o s = A> (' ' % &r%a& o For%s' D%v%-o+ % ', DENR, o ' % a--% %)ro& ) ' a' a +or'(o o ' % ar%a cov%r%) /0 ' % ( ( c-a( s as (' ( ' % Ma a'R(v%r For%s' R%s%rva'(o & )%r roc. 57! o ?& % 2#, 1*#* a ) (' DAR o acco& 'o a--% %) cov%ra % o ' % o' %r +or'(o (' ( ' % N&%va (3ca0a6@&(r( o C(v(-R%s%rva'(o & )%r roc. 19*8 o 11 S%+'% /%r 1*75.

O 15 ?&-0 1**1 E$%c&'(v% D(r%c'or L%o ar)o A. aa' r% %c'%) +%'('(o %rJs ros+%c'(%r (' A++-(ca'(o = A> o ' % ro& ) ' a' ' % M% ora )& o ?&-0 "8, 1**1

% )ors%) /0 ' % R% (o a- T%c (ca- D(r%c'or or M( %s r%v%a-%) ' a' ' % ar%a cov%r%)

as o&'s()% ov%r % ' r%s%rva'(o ' a' ' % +ros+%c' c-a( as ( co -(c' (' %$(s'(c-a( s a ), ' a' ' % ar%a a) /%% %$'% s(v%-0 %$+-or%) ( ' % %ar-0 1*8" s.

%'('(o %r ov%) or r%co s()%ra'(o . R% (o a- E$%c&'(v% D(r%c'or Sa &%- ara asr%co % )%) 'o ' % DENR S%cr%'ar0 ' a' +%'('(o %r s r%4&%s' or r%co s()%ra'(o /%)% (%) ' a' ' % %$(s'( r( 's o ( ( c-a( o-)%rs /% r%s+%c'%) a ), ' a' ' % +r(or -% a- r( 's o M SAKF( a c(a- a ) T%c (ca- Ass(s'a c% A r%% % ' a++-(ca 's ov%r s&/ %c' ar%a /% r%co (3%).

As r% ar)s +%'('(o %r s A (-%) (' ' % DAR, (' a++%ar%) ' a' (' as (ss&%) ac-%ara c% 'o +ros+%c' or s($ =#> o ' s ro D%c% /%r 11, 1**5.

O A& &s' 15, 1**7 +%'('(o %r a++%a-%) 'o +&/-(c r%s+o )% ' M( %s A) &)(ca'(o oar)=MA >. %'('(o %r a( 'a( %) ' a' s&/ %c' ar%a as (' ( ' % Ma a' R(v%r For%s'R%s%rva'(o . O ?& % 11, 1**8 ' % r% %c'(o o ' % A as a (r %) %r%as ' %( ( c-a( s o r%s+o )% 's A-/%r'o G. & o-o %' a-. ' a' a) /%% co v%r'%) ( 'o aM SA, s&/ %c' 'o co +-(a c% (' R.A. 7*92 a ) DAO No. *#69", %r% (v% )&%co&rs%.

%'('(o %r ov%) or r%co s()%ra'(o . R%s+o )% ' MA )% (%) +%'('(o %rJs o'(o .

ISSUE:

%' %r r%s+o )% ' MA %rr%) ( ( )( ' a' ' % ar%a s&/ %c' o ' % A as o&'s()%' % Ma a' R(v%r For%s' R%s%rva'(o .

ELD:

R%s+o )% ' MA corr%c'-0 &+ %-) ' % ra'(oc( a'(o o R% (o a- E$%c&'(v% D(r%c'or

Page 142: Nat Res Digested

8/10/2019 Nat Res Digested

http://slidepdf.com/reader/full/nat-res-digested 142/173

ara as ( )% 0( +%'('(o %r s A.

T % )(sa++rova- o Marco++%rJs A or%ov%r, )() o' % a a'% ro a s( -%r%co % )a'(o o ' % RTD or M( %s. R%cor)s o&-) s o ' a' as %ar-0 as Ma0 !1,1*8* $ $ $ ' % & o-o ro&+ o D 9#! c-a( s (c Marco++%r as %v% '&a--0s&rro& )%) /0 (-( ('s o AO 16!" ro&+ o c-a( s $ $ $ $ as co (r %) /0 ' %For%s' E ( %%r( S%c'(o o ' % r% (o 'o /% o&'s()% +roc-a( %) a'%rs %) ar%as,(-)%r %ss, a'(o a- +ar<s a ) %$(s'( ov%r % ' r% or%s'a'(o +ro %c's $ $ $ $

I o' %r or)s, ' % c(rc& s'a c% ' a' ' % ar%a cov%r%) /0 +%'('(o %r s A (s o&'s()% ' %Ma a' R(v%r For%s' R%s%rva'(o as /%% a)%4&a'%-0 %s'a/-(s %) /0 ' % o--o (%v()% c%: =a> co (r a'(o as %ar-0 as !1 Ma0 1*8* /0 ' % For%s' E ( %%r( S%c'(oo T& &% arao, Ca a0a =/> ' % 8 ?&-0 1**1 M% ora )& R%+or' o R% (o a-T%c (ca- D(r%c'or & sa- ?r. a ), =c> +-o''( +rov()%) /0 ' % Na'(o a- Ma++( a )R%so&rc%s I or a'(o A&' or('0 +%r ('s 2 ?& % 1**5 ( )ors% % ' o ' % a+s 'o ' %o (c% o ' % R% (o a- E$%c&'(v% D(r%c'or. %'('(o %r co '%s's ' % %$c-&s(o o ' % ar%as&/ %c' o ('s A (' ( ' % Ma a' R(v%r For%s' R%s%rva'(o /as%) %r%-0 o ' %

a--% %) '0+o ra+ (ca- %rror co (''%) /0 so %/o)0 ( ' % E ( %%r( S%c'(o o ' %DENR. As()% ro ' % ac' ' a' ' % a--% a'(o )o%s o' av% a 0' ( 'o s&++or' (', ' %a or% % '(o %) )oc& % 's (c ' % R% (o a- E$%c&'(v% D(r%c'ors r%-(%) &+o ()% 0( ' % A a) a-r%a)0 s%''-%) ' % (ss&%.

F&r' %r or%, r%s+o )% ' MA %v% or'( (%) ' % /as%s or ' % r% %c'(o o +%'('(o %r sA. As +-o''%) /0 ' % La )s Ma a % % ' S%c'or o DENR R% (o 2 co 'a( %) ( ' %

s<%'c +-a o 11 Nov% /%r 1**# a ) as s o ( ' % La ) Us% a+ o ' % Co & ('0E v(ro % ' a ) Na'&ra- R%so&rc%s O (c% o D&+a$, N&%va (3ca0a, ' % ar%a cov%r%)& )%r ' % A (s ( )%%) o&'s()% a 0 ov%r % ' r%s%rva'(o .

G.R. No. *8!!2 ?a &ar0 1#, 1**5

MINERS ASSOCIATION OF T E ILI INES, INC., petitioner ,

vs .

ON. FULGENCIO S. FACTORAN, ?R., S%cr%'ar0 o E v(ro % ' a ) Na'&ra-R%so&rc%s, a ) ?OEL D. MU CO, D(r%c'or o M( %s a ) G%osc(% c%s &r%a&,respondents .

FACTS:&rs&a ' 'o S%c'(o # o E$%c&'(v% Or)%r No. 27*, a&' or(3( ' % DENR S%cr%'ar0 'o% o'(a'% a ) co c-&)% o( ' v% '&r%, co6+ro)&c'(o , or +ro)&c'(o 6s ar( a r%% % 'sor ' % %$+-ora'(o , )%v%-o+ % ' a ) &'(-(3a'(o o ( %ra- r%so&rc%s, a ) +r%scr(/(' % &()%-( %s or s&c a r%% % 's a ) ' os% a r%% % 's ( vo-v( '%c (ca- or ( a c(a- ass(s'a c% /0 or%( 6o %) cor+ora'(o s or -ar %6sca-% %$+-ora'(o ,

)%v%-o+ % ', a ) &'(-(3a'(o o ( %ra-s, ' % DENR S%cr%'ar0 (ss&%) DENR

Page 143: Nat Res Digested

8/10/2019 Nat Res Digested

http://slidepdf.com/reader/full/nat-res-digested 143/173

A) ( (s'ra'(v% Or)%r No. 57, s%r(%s o 1*8*, % '('-%) G&()%-( %s o M( %ra- ro)&c'(oS ar( A r%% % ' & )%r E$%c&'(v% Or)%r No. 27*. U )%r ' % 'ra s('or0 +rov(s(o o sa() DENR A) ( (s'ra'(v% Or)%r No. 57, % /o)(%) ( ('s Ar'(c-% *, a-- %$(s'( ( (-%as%s or a r%% % 's (c %r% ra '%) a '%r ' % % %c'(v('0 o ' % 1*87 Co s'('&'(o+&rs&a ' 'o E$%c&'(v% Or)%r No. 211, %$c%+' s a-- sca-% ( ( -%as%s a ) ' os%

+%r'a( ( 'o sa ) a ) rav%- a ) 4&arr0 r%so&rc%s cov%r( a ar%a o ' % '0 =2">%c'ar%s or -%ss, s a-- /% co v%r'%) ( 'o +ro)&c'(o 6s ar( a r%% % 's (' ( o % =1>0%ar ro ' % % %c'(v('0 o ' %s% &()%-( %s.

T % S%cr%'ar0 o ' % DENR ' % &r' %r (ss&%) DENR A) ( (s'ra'(v% Or)%r No. 82,s%r(%s o 1**", -a0( )o ' % roc%)&ra- G&()%-( %s o ' % A ar) o M( %ra-ro)&c'(o S ar( A r%% % ' =M SA> ' ro& N% o'(a'(o .

T % (ss&a c% a ) ' % ( +% )( ( +-% % 'a'(o /0 ' % DENR o A) ( (s'ra'(v% Or)%r Nos. 57 a ) 82 a '%r ' %(r r%s+%c'(v% % %c'(v('0 )a'%s co +%--%) ' % M( %rs Assoc(a'(oo ' % (-(++( %s, I c. 'o (-% ' % ( s'a ' +%'('(o assa(-( ' %(r va-()('0 a )

co s'('&'(o a-('0 /% or% ' (s Co&r'.%'('(o %r M( %rs Assoc(a'(o o ' % (-(++( %s, I c., a( -0 co '% )s ' a' ' %

a) ( (s'ra'(v% or)%rs )o o' co or (' E$%c&'(v% Or)%r Nos. 211 a ) 27*, +%'('(o %r co '% )s ' a' /o' or)%rs v(o-a'% ' % o 6( +a(r % ' o co 'rac' +rov(s(o & )%r Ar'(c-%III, S%c'(o 1" o ' % 1*87 Co s'('&'(o o ' % ro& ) ' a' A) ( (s'ra'(v% Or)%r No. 57& )&-0 +r%6'%r ( a'%s %$(s'( ( ( -%as%s a ) o' %r ( ( a r%% % 's a )a&'o a'(ca--0 co v%r's ' % ( 'o +ro)&c'(o 6s ar( a r%% % 's (' ( o % =1> 0%ar ro ('s % %c'(v('0 )a'%. O ' % o' %r a ), A) ( (s'ra'(v% Or)%r No. 82 )%c-ar%s ' a'a(-&r% 'o s&/ (' L%''%rs o I '% ' a ) M( %ra- ro)&c'(o 6S ar( A r%% % 's (' ( ' o=2> 0%ars ro ' % )a'% o % %c'(v('0 o sa() &()%-( % or o ?&-0 17, 1**1 s a-- ca&s% ' %

a/a )o % ' o ' %(r ( ( , 4&arr0 a ) sa ) rav%- +%r ('s.%'('(o %r ar &%) ' a' E$%c&'(v% Or)%r No. 27* )o%s o' co '% +-a'% a&'o a'(c

co v%rs(o o ( ( -%as% a r%% % 's ( 'o ( ( +ro)&c'(o 6s ar( a r%% % ' as+rov()%) & )%r Ar'(c-% *, A) ( (s'ra'(v% Or)%r No. 57 a )Kor ' % co s%4&% 'a/a )o % ' o ( ( c-a( s or a(-&r% 'o s&/ (' LOIs a ) M SAs & )%r S%c'(o !,

A) ( (s'ra'(v% Or)%r No. 82 /%ca&s% S%c'(o 1 o sa() E$%c&'(v% Or)%r No. 27*% +o %rs ' % DENR S%cr%'ar0 'o % o'(a'% a ) % '%r ( 'o vo-& 'ar0 a r%% % 's (c&s' s%' or' ' % ( ( & '%r s a ) co )('(o s +rov()%) & )%r S%c'(o 2 ' %r%o .Mor%ov%r, +%'('(o %r co '% )s ' a' ' % +o %r 'o r% &-a'% a ) % '%r ( 'o ( (a r%% % 's )o%s o' ( c-&)% ' % +o %r 'o +r%'%r ( a'% %$(s'( ( ( -%as%a r%% % 's.

ISSUE:

%' %r or o' DENR A) ( (s'ra'(v% Or)%r Nos. 57 a ) 82 (ss&%) /0 ' % DENRS%cr%'ar0 ar% & co s'('&'(o a-.

ELD:

Page 144: Nat Res Digested

8/10/2019 Nat Res Digested

http://slidepdf.com/reader/full/nat-res-digested 144/173

NO. DENR A) ( (s'ra'(v% Or)%r Nos. 57 a ) 82 ar% o' & co s'('&'(o a-.

T % 4&%s'(o %) a) ( (s'ra'(v% or)%rs ar% r%aso a/-0 )(r%c'%) 'o ' % acco +-(s % ' o ' % +&r+os%s o ' % -a & )%r (c ' %0 %r% (ss&%) a ) %r% ( '% )%) 'o s%c&r% ' %+ara o& ' ( '%r%s' o ' % +&/-(c, ' %(r %co o (c ro ' a ) %- ar%. T % va-()('0 a )

co s'('&'(o a-('0 o A) ( (s'ra'(v% Or)%r Nos. 57 a ) 82 &s' /% s&s'a( %), a ) ' %(r orc% a ) % %c' &+ %-).

A) ( (s'ra'(v% Or)%r No. 57 a++-(%s o -0 'o a-- %$(s'( ( ( -%as%s or a r%% % 's(c %r% ra '%) a '%r ' % % %c'(v('0 o ' % 1*87 Co s'('&'(o +&rs&a ' 'o E$%c&'(v%

Or)%r No. 211. I' /%ars % '(o ' a' & )%r ' % '%$' o E$%c&'(v% Or)%r No. 211, ' %r% (sa r%s%rva'(o c-a&s% (c +rov()%s ' a' ' % +r(v(-% %s as %-- as ' % '%r s a )co )('(o s o a-- %$(s'( ( ( -%as%s or a r%% % 's ra '%) a '%r ' % % %c'(v('0 o ' %1*87 Co s'('&'(o , +&rs&a ' 'o E$%c&'(v% Or)%r No. 211, s a-- /% s&/ %c' 'o a 0 a ) a--o)( (ca'(o s or a-'%ra'(o s (c Co r%ss a0 a)o+' +&rs&a ' 'o Ar'(c-% II, S%c'(o 2o ' % 1*87 Co s'('&'(o . % c%, ' % s'r(c'&r%s o ' % o 6( +a(r % ' o co 'rac' c-a&s%

& )%r Ar'(c-% III, S%c'(o 1" o ' % 1*87 Co s'('&'(o )o o' a++-0 'o ' % a or%sa() ( (-%as%s or a r%% % 's ra '%) a '%r ' % % %c'(v('0 o ' % 1*87 Co s'('&'(o , +&rs&a ' 'oE$%c&'(v% Or)%r No. 211. T %0 ca /% a % )%), o)( (%) or a-'%r%) /0 a s'a'&'%+ass%) /0 Co r%ss 'o ac (%v% ' % +&r+os%s o Ar'(c-% II, S%c'(o 2 o ' % 1*87Co s'('&'(o .

Mor%ov%r, o %r% ( A) ( (s'ra'(v% Or)%r No. 57 (s ' %r% a 0 +rov(s(o (c o&-)-%a) &s 'o co c-&)% ' a' ' % 4&%s'(o %) or)%r a&' or(3%s ' % a&'o a'(c co v%rs(o o ( ( -%as%s a ) a r%% % 's ra '%) a '%r ' % % %c'(v('0 o ' % 1*87 Co s'('&'(o ,+&rs&a ' 'o E$%c&'(v% Or)%r No. 211, 'o +ro)&c'(o 6s ar( a r%% % 's. T % +rov(s(o( Ar'(c-% * o A) ( (s'ra'(v% Or)%r No. 57 ' a' a-- s&c -%as%s or a r%% % 's s a-- /%

co v%r'%) ( 'o +ro)&c'(o s ar( a r%% % 's (' ( o % =1> 0%ar ro ' % % %c'(v('0 o ' %s% &()%-( %s co&-) o' +oss(/-0 co '% +-a'% a & (-a'%ra- )%c-ara'(o o ' % +ar' o ' % Gov%r % ' ' a' a-- %$(s'( ( ( -%as%s a ) a r%% % 's ar% a&'o a'(ca--0co v%r'%) ( 'o +ro)&c'(o 6s ar( a r%% % 's. O ' % co 'rar0, ' % &s% o ' % '%r+ro)&c'(o 6s ar( a r%% % ' ( ' % sa % +rov(s(o ( +-(%s % o'(a'(o /%' %% ' %Gov%r % ' a ) ' % a++-(ca 's, ( ' %0 ar% so ( )%). N% o'(a'(o % a'%s co +&-s(oor a&'o a'(c co v%rs(o as s& %s'%) /0 +%'('(o %r ( ' % ( s'a ' +%'('(o . A ( %ra-+ro)&c'(o 6s ar( a r%% % ' =M SA> r%4&(r%s a %%'( o ' % ( )s o ' % +ar'(%sa '%r % o'(a'(o s arr(v%) a' ( oo) a(' a ) ( accor)a c% (' ' % +roc%)&r% -a())o ( ' % s&/s%4&% ' A) ( (s'ra'(v% Or)%r No. 82.

Page 145: Nat Res Digested

8/10/2019 Nat Res Digested

http://slidepdf.com/reader/full/nat-res-digested 145/173

OL M IC MINES AND DE ELO MENT COR ., %'('(o %r,

6 v%rs&s 6 LATINUM GROU METALS COR ORATION, R%s+o )% '. CITINIC?E MINES AND DE$E OPMENT CORPORATION, %'('(o %r,

6 v%rs&s 6

HON. %UDGE #IEN$ENIDO C. # ANCA3 OR, ) ')s ( ( )t (s t'/ P*/s) ) g%u g/ o; t'/ R/g)o (5 T*)(5 Cou*t o; P(5( ( , #*( ' @=, Pu/*to P*) /s( C)t ,P(5( ( , ( P ATINUM GROUP META CORPORATION,

R%s+o )% 's

P ATINUM GROUP META S CORPORATION,

Page 146: Nat Res Digested

8/10/2019 Nat Res Digested

http://slidepdf.com/reader/full/nat-res-digested 146/173

%'('(o %r,

6 v%rs&s 6 CITINIC?E MINES AND DE$E OPMENT CORPORATION, ( t) g ;o* )ts o

) t/*/st ( o /'(5; o; O MPIC MINES AND DE$E OPMENT CORPORATION,R%s+o )% '.P ATINUM GROUP META S CORPORATION, %'('(o %r, 6 v%rs&s 6

COURT O3 APPEA S ( PO C. D , R%s+o )% 's

FACTS:I 1*71 a ) 1*8", O-0 +(c as ra '%) M( ( L%as% Co 'rac's; /0 ' % S%cr%'ar0 o ' % !E$% cov%r( ( ( ar%as -oca'%) ( ' % & (c(+a-('(%s o Narra a ) Es+a o-a,a-a a .

O ?&-0 18, 2""!, O-0 +(c % '%r%) ( 'o a O /*(t) g Ag*//+/ t (' -a'( & , /0v(r'&% o (c -a'( & as (v% ' % %$c-&s(v% r( ' 'o co 'ro-, +oss%ss,a a %Ko+%ra'%, a ) co )&c' ( ( o+%ra'(o s, a ) 'o ar<%' or )(s+os% ( (

+ro)&c's o ' % Toro 'o N(c<%- M( % ( ' % M& (c(+a-('0 o Narra. I r%'&r , -a'( &o&-) +a0 O-0 +(c a ro0a-'0 %% o 2P o ' % ross r%v% &%s.

O-0 +(c a ) -a'( & a++-(%) or, a ) %r% s&/s%4&% '-0 ra '%) ' % %c%ssar0ov%r % ' +%r ('s a ) % v(ro % 'a- co +-(a c% c%r'( (ca'%s.

O A+r(- 29, 2""#, O-0 +(c s% ' a -%''%r 'o -a'( & , ( or ( ' % -a''%r o ' %

( %)(a'% '%r ( a'(o o ' % O+%ra'( A r%% % ' o acco& ' o -a'( & Js rossv(o-a'(o s o ('s '%r s, a ) )(r%c'( -a'( & 'o ( %)(a'%-0 s&rr% )%r +oss%ss(o o ' % s&/ %c' ( ( ar%as & )%r ' % O+%ra'( A r%% % '.

O-0 +(c ( s'('&'%) a ac'(o or ' % (ss&a c% o a ( & c'(v% r(' /% or% ' % RTC o &%r'o r( c%sa a a( s' -a'( & . I ('s +ra0%r, O-0 +(c so& ' 'o % o( -a'( & roco )&c'( ( ( o+%ra'(o s o ' % s&/ %c' ( ( ar%as, a ) a-so 'o r%cov%r +oss%ss(o ' %r%o . T % RTC )(s (ss%) O-0 +(cJs co +-a( '.

Page 147: Nat Res Digested

8/10/2019 Nat Res Digested

http://slidepdf.com/reader/full/nat-res-digested 147/173

O-0 +(c ' % (-%) ' o cas%s (' ' % rov( c(a- M( ( R% &-a'or0 oar) = &M%B> or ' % r%voca'(o o ' % SSM s o -a'( & , o ' % ro& ) o O-0 +(cJs '%r ( a'(o o ' %O+%ra'( A r%% % ' /%ca&s% o ' % a--% %) ross v(o-a'(o s ' %r%o /0 -a'( & . T (s

as )(s (ss%) a ) OA or ' % ca c%--a'(o o ' % O+%ra'( A r%% % ' a ) ' %r%voca'(o o ' % SSM s o -a'( & . T (s cas% as s&/s%4&% '-0 (' )ra /0 .

(-% ' %s% ' o a) ( (s'ra'(v% cas%s %r% +% )( , O-0 +(c 'ra s %rr%) ('s a++-(ca'(o sor ( %ra- a r%% % 's, ( c-&)( ('s r( 's & )%r ' % O+%ra'( A r%% % ', 'o C('( (c<%-

via a D%%) o , (' o&' ' % < o -%) % or co s% ' o -a'( & . T (s ass( % ' as' %r%a '%r a++rov%) /0 ' % R% (o a- D(r%c'or o ' % M( %s a ) G%osc(% c%s &r%a&=M"B >.

A '%r ' % ass( % ', C('( (c<%- (-%) C(v(- Cas% No. "#6"185 /% or% ' % RTC o araQa4&%, o ?& % 21, 2""#, s%%<( 'o ( va-()a'% ' % O+%ra'( A r%% % ' /as%) o-a'( & Js a--% %) v(o-a'(o o ('s '%r s. T (s ac'(o as a-so )(s (ss%) /0 ' % 'r(a-co&r', c('( or& s o++( a ) ( +ro+%r v% &% as a o ' % ro& )s or )(s (ssa-.C('( (c<%- )() o' /o' %r 'o a++%a- ' (s )(s (ssa-, o+'( ( s'%a) 'o ( ) o' %r r% %)(%s.

C('( (c<%- ' %r%a '%r (-%) ' r%% a) ( (s'ra'(v% cas%s: MR Cas% No. ""26"#, DENRE v(ro % 'a- Ma a % % ' &r%a& = EMB > Cas% No. 825!, a ) OA Cas% No. 2""#6"26 .

C(v(- Cas% No. 91** ( vo-v%) a co +-a( ' or 4&(%'( o '('-%, )a a %s, /r%ac o co 'rac', a ) s+%c( (c +%r or a c% (-%) /0 -a'( & a a( s' O-0 +(c /% or% ' % RTC o &%r'o r( c%sa, a-a a , ra c *5 o ?& % 19, 2""#.

O-0 +(c so& ' ' % )(s (ssa- o -a'( & Js C(v(- Cas% No. 91** ' ro& a o'(o 'o)(s (ss %r% O-0 +(c a--% %) ' a' ' % 'r(a- co&r' as (' o&' &r(s)(c'(o 'o r&-% o ' %(ss&%s ra(s%) ( ' % cas%. 'lympic contended that the case involved a mining disputerequiring the technical e pertise o the &'*+ accordingly, urisdiction should e with the&'

ISSUE:

(c /o)0 as ' % a&' or('0 'o %ar a ) )%c()% ' % )(s+&'% /%' %% O-0 +(cKC('( (c<%-a ) -a'( & , as +ar'(%s 'o ' % o+%ra'( a r%% % '.

Page 148: Nat Res Digested

8/10/2019 Nat Res Digested

http://slidepdf.com/reader/full/nat-res-digested 148/173

Page 149: Nat Res Digested

8/10/2019 Nat Res Digested

http://slidepdf.com/reader/full/nat-res-digested 149/173

G.R. No. 1#!5"*

ICO RESOURCES, INC.,+%'('(o %r,

6 v%rs&s 6

ASE METALS MINERAL RESOURCES COR ORATION a ) T E MINES AD?UDICATION OARD, r%s+o )% 's.

FACTS:

Page 150: Nat Res Digested

8/10/2019 Nat Res Digested

http://slidepdf.com/reader/full/nat-res-digested 150/173

C% 'ra- M( )a ao M( ( a ) D%v%-o+ % ' Cor+ora'(o =CMMCI or /r%v('0> % '%r%)( 'o a M( %s O+%ra'( A r%% % ' =A r%% % ' or /r%v('0> (' a a a M( ( a )D%v%-o+ % ' Cor+ora'(o = a a a M( ( or /r%v('0> %r%/0 ' % -a''%r a r%%) 'o ac'

as M( % O+%ra'or or ' % %$+-ora'(o , )%v%-o+ % ', a ) %v% '&a- co %rc(a- o+%ra'(oo CMMCIJs %( '%% =18> ( ( c-a( s -oca'%) ( A &sa )%- S&r.

&rs&a ' 'o ' % '%r s o ' % A r%% % ', a a a M( ( (-%) a++-(ca'(o s or M( (L%as% Co 'rac's ov%r ' % ( ( c-a( s (' ' % &r%a& o M( %s. So ' a' a a aM( ( as (ss&%) a M( %s T% +orar0 %r (' a&' or(3( (' 'o %$'rac' a ) )(s+os% o +r%c(o&s ( %ra-s o& ) (' ( ('s ( ( c-a( s. U+o ('s %$+(ra'(o , ' % '% +orar0+%r (' as s&/s%4&% '-0 r% % %) ' r(c% /0 ' % &r%a& o M( %s, ' % -as' /%( o ?& %28, 1**1.

S( c% a +or'(o o a a a M( ( Js ( ( c-a( s as -oca'%) ( +%'('(o %r ICO Js-o ( co c%ss(o ( A &sa )%- S&r, a a a M( ( a ) +%'('(o %r ICO % '%r%)( 'o a M% ora )& o A r%% % ', %r%/0, ( &'&a- r%co ('(o o %ac o' %rJs r( ''o ' % ar%a co c%r %), +%'('(o %r ICO a--o %) a a a M( ( a acc%ssKr( ' o a0 'o ('s ( ( c-a( s. a a a M( ( co v%r'%) ('s ( ( c-a( s 'o a++-(ca'(o sor M( %ra- ro)&c'(o S ar( A r%% % 's =M SA or /r%v('0>.

(-% ' % M SA %r% +% )( , a a a M( ( , o D%c% /%r 18, 1**#, )%c()%) 'os%--Kass( ('s r( 's a ) ( '%r%s's ov%r ' (r'06s%v% =!7> ( ( c-a( s ( avor o +r(va'%r%s+o )% ' as% M%'a-s M( %ra- R%so&rc%s Cor+ora'(o = as% M%'a-s or /r%v('0>. T %'ra s %r ( c-&)%) ( ( c-a( s %-) /0 a a a M( ( ( ('s o r( ' as c-a( o %r,as %-- as ' os% cov%r%) /0 ('s ( ( o+%ra'( a r%% % ' (' CMMCI.

U+o /%( ( or %) o ' % )%v%-o+ % ', CMMCI, as c-a( o %r, ( %)(a'%-0a++rov%) ' % ass( % ' a)% /0 a a a M( ( ( avor o +r(va'% r%s+o )% ' as%M%'a-s, ' %r%/0 r%co (3( +r(va'% r%s+o )% ' as% M%'a-s as ' % % o+%ra'or o ('sc-a( s.

O Marc 1", 1**7, +r(va'% r%s+o )% ' as% M%'a-s a % )%) a a a M( ( Js+% )( M SA a++-(ca'(o s (' ' % &r%a& o M( %s 'o s&/s'('&'% ('s%- as a++-(ca 'a ) 'o s&/ (' a))('(o a- )oc& % 's ( s&++or' o ' % a++-(ca'(o . Ar%a c-%ara c%s ro' % DENR R% (o a- D(r%c'or a ) S&+%r( '% )% ' o ' % A &sa Mars a ) (-)-( %Sa c'&ar0 %r% s&/ (''%), as r%4&(r%).

O Oc'o/%r 7, 1**7, +r(va'% r%s+o )% ' as% M%'a-sJ a % )%) M SA a++-(ca'(o s%r% +&/-(s %) ( accor)a c% (' ' % r%4&(r% % 's o ' % M( ( Ac' o 1**5.

O Nov% /%r 18, 1**7, +%'('(o %r ICO (-%) (' ' % M( %s G%o6Sc(% c%s &r%a&=MG >, Cara a R% (o a- O (c% No. III a A)v%rs% C-a( a )Kor O++os('(o 'o +r(va'%r%s+o )% ' as% M%'a-sJ a++-(ca'(o . A '%r ' % s&/ (ss(o o ' %(r r%s+%c'(v% +os('(o+a+%r, ' % a %- Ar/('ra'or (ss&%) a Or)%r )(sa++rov( +r(va'% r%s+o )% ' as%M%'a-sJ M SA o ' % r%aso s ' a' a)v%rs% c-a( as (-%) o '( %, ' a' ' % ra '( o

Page 151: Nat Res Digested

8/10/2019 Nat Res Digested

http://slidepdf.com/reader/full/nat-res-digested 151/173

' % M SA a++-(ca'(o o ar%a s&/ %c' o a IFMA or TLA (c (s cov%r%) /0 ar%s()% '(a- arra '0, ' % +a %- /%-(%v%s (' ca o', & -%ss ' % ra '%% co s% 's ' %r%'o,(' o&' ' % ra '%%Js co s% ', ' % ar%a (s co s()%r%) c-os%) 'o ( ( -oca'(o =s%c. 1*>=/> =No. 2>, DAO No. *#69"> a ) ' a' ' % ( ( -oca'(o ( or%s' or '( /%r-a ) (sa--o %) o -0 ( s&c or%s' or '( /%r-a ) (s o' -%as%) /0 ' % ov%r % ' 'o a 4&a-( (%)

+%rso or % '('0 a ) ( (' (s -%as%) ' % co s% ' o ' % -%ssor (s %c%ssar0, ( a))('(o 'o' % ar%a c-%ara c% 'o /% (ss&%) /0 ' % a % c0 co c%r %) /% or% (' (s s&/ %c'%) 'o( ( o+%ra'(o .

-a 'a'(o (s co s()%r%) c-os%) 'o ( ( -oca'(o s /%ca&s% (' (s o 'a % ' 'o ( ( .o' ar% %$'r% %s. T %0 ca o' %$(s' a' ' % sa % '( %. T % o' %r &s' %c%ssar(-0s'o+ /% or% ' % o' %r o+%ra'%.

r(va'% r%s+o )% ' as% M%'a-s (-%) a No'(c% o A++%a- (' +&/-(c r%s+o )% ' MA ,' % -a''%r r% )%r%) ' % assa(-%) )%c(s(o s%''( as()% ' % a %- Ar/('ra'orJs or)%r. T %Co&r' o A++%a-s &+ %-) ' % )%c(s(o o ' % MA .

% c% ' (s +%'('(o .

ICO +r%s% 's ' % o--o ( (ss&%s: =1> ' % 2,75# %c'ar%s s&/ %c' o as% M%'a-sJM SA ar% c-os%) 'o ( ( o+%ra'(o s %$c%+' &+o ICO Js r(''% co s% ' +&rs&a ''o %$(s'( -a s, r&-%s a ) r% &-a'(o s a ) /0 v(r'&% o ' % r%s()% '(a- arra '0 =2> ('sr%s()% '(a- arra '0 (s +ro'%c'%) /0 ' % o 6( +a(r % ' c-a&s% o ' % Co s'('&'(o a )=!> (' )o%s o' ra(s% % (ss&%s ( ('s +%'('(o .

ICO ass%r's ' a' ('s co c%ss(o ar%as ar% c-os%) 'o ( ( o+%ra'(o s as ' %s% ar%(' ( ' % A &sa 6S&r( ao6Davao or%s' r%s%rv% %s'a/-(s %) & )%r roc-a a'(o No.!#* o ' % Gov. G% . D ( ' Dav(s. T % ar%a (s a--% %)-0 a-so +ar' o +%r a % ' or%s'%s'a/-(s %) & )%r R%+&/-(c Ac' No. !"*2 =RA !"*2>, a ) ov%r-a+s ' % (-)%r %ss ar%a%r% ( ( a++-(ca'(o s ar% %$+r%ss-0 +ro (/('%) & )%r RA 758#. % c%, ' % ar%a (s

c-os%) 'o ( ( o+%ra'(o s & )%r S%c. 1*= > o RA 7*92.

ISSUE: %' %r or o' ' % ar%a cov%r%) /0 as% M%'a-sJ M SA (s, /0 -a , c-os%) 'o ( (

ac'(v('(%s

%' %r or o' ' % r%s()% '(a- arra '0 (s a co 'rac' +ro'%c'%) /0 ' % o 6( +a(r % 'c-a&s% o ' % 1*87 Co s'('&'(o .

HE D

A / t t'/ ;)*st )ssu/, t'/ Cou*t *u5/ t'(t t'/ ar%a cov%r%) /0 as% M%'a-sJ M SA(s, /0 -a , o' c-os%) 'o ( ( ac'(v('(%s.

Page 152: Nat Res Digested

8/10/2019 Nat Res Digested

http://slidepdf.com/reader/full/nat-res-digested 152/173

T %r% (s o %v()% c% ( ' (s cas% ' a' ' % ar%a cov%r%) /0 as% M%'a-sJ M SA as/%% +roc-a( %) as a'%rs %) or%s' r%s%rv%s.

Ev% ra '( ' a' ' % ar%a cov%r%) /0 ' % M SA (s +ar' o ' % A &sa 6Davao6S&r( ao

For%s' R%s%rv%, s&c )o%s o' %c%ssar(-0 s( ( 0 ' a' ' % ar%a (s a/so-&'%-0 c-os%) 'o( ( ac'(v('(%s. Co 'rar0 'o ICO Js o/v(o&s (sr%a)( o o&r )%c(s(o ( *pe Mining Co., -nc. v. "arcia, supra, 'o ' % % %c' ' a' ( %ra- a r%% % 's ar% o' a--o %)( ' % or%s' r%s%rv% %s'a/-(s %) & )%r roc-a a'(o !#*, ' % Co&r' ( ' a' cas% ac'&a--0r&-%) ' a' +&rs&a ' 'o D 9#! as a % )%) /0 D 1!85, o % ca ac4&(r% ( ( r( 's(' ( or%s' r%s%rv%s, s&c as ' % A &sa 6Davao6S&r( ao For%s' R%s%rv%, /0 ( ('(a--0a++-0( or a +%r (' 'o +ros+%c' (' ' % &r%a& o For%s' a ) D%v%-o+ % ' a )s&/s%4&% '-0 or a +%r (' 'o %$+-or% (' ' % &r%a& o M( %s a ) G%osc(% c%s.

Mor%ov%r, S%c. 18 RA 7*92 a--o s ( ( %v% ( '( /%r-a ) or or%s''0 s&/ %c' 'o%$(s'( r( 's a ) r%s%rva'(o s. S( (-ar-0, S%c. 97 o D 7"5 +%r ('s ( ( o+%ra'(o s

( or%s' -a )s (c ( c-&)% ' % +&/-(c or%s', ' % +%r a % ' or%s' or or%s' r%s%rv%s,a ) or%s' r%s%rva'(o s

(' r% ar) 'o ' % s%co ) (ss&%, ' % Co&r' )o o' s&/scr(/% 'o ICO Js ar & % ' ' a'' % r%s()% '(a- arra '0 )a'%) S%+'% /%r 25, 1*#8 (s a co 'rac' +ro'%c'%) /0 ' % o 6( +a(r % ' c-a&s% o ' % 1*87 Co s'('&'(o . A %$a ( a'(o o ' % r%s()% '(a-arra '0 a' o c% r%v%a-s ' a' (' s( +-0 r%ass&r%s ICO o ' % ov%r % 'Js

co (' % ' 'o &+ o-) ' % '%r s a ) co )('(o s o ('s '( /%r -(c% s% a ) &ara '%%sICO Js +%ac% &- a ) a)%4&a'% +oss%ss(o a ) % o0 % ' o ' % ar%as (c ar% ' %/as(c so&rc%s o ra a'%r(a-s or ('s oo) +roc%ss( co +-%$. T % arra '0 cov%rso -0 ' % r( ' 'o c&', co--%c', a ) r% ov% '( /%r ( ('s co c%ss(o ar%a, a ) )o%s o'%$'% ) 'o ' % &'(-(3a'(o o o' %r r%so&rc%s, s&c as ( %ra- r%so&rc%s, occ&rr( (' (' % co c%ss(o .

T % r%s()% '(a- arra '0 ca o' /% co s()%r%) a co 'rac' )(s'( c' ro TLANo. 97 a ) IFMA No. !5. I' (s %r%-0 a co--a'%ra- & )%r'a<( (c ca o' a +-( 0ICO Js r( 's & )%r ('s '( /%r -(c% s%. S) / t)+ /* 5) / s/s (*/ ot o t*( ts, t'/o ")+ ()*+/ t 5(us/ ( ot / ) 9o:/ .

Page 153: Nat Res Digested

8/10/2019 Nat Res Digested

http://slidepdf.com/reader/full/nat-res-digested 153/173

RO CO ER MINING COR ORATION, +%'('(o %r,

versus

MINES AD?UDICATION OARD6DE ARTMENT OF EN IRONMENT AND NATURALRESOURCES, ET AL, r%s+o )% '.

FACTS:

%'('(o %r (s a cor+ora'(o )&-0 or a (3%) a ) %$(s'( & )%r (-(++( % -a s % a %) (' % /&s( %ss o ( ( . O !1 Marc 2""", +%'('(o %rJs A++-(ca'(o or M( %ra-ro)&c'(o S ar( A r%% % ' =M SA>, or ' % %$+-ora'(o , )%v%-o+ % ' a )

co %rc(a- &'(-(3a'(o o c%r'a( +0r('% or% a ) o' %r ( %ra- )%+os('s ( a 9,!#".716%c'ar% -a ) ( Daso-, a as( a , as a++rov%) a ) M SA No. 15!62"""61 as(ss&%) ( ('s avor.

r(va'% r%s+o )% ' (s a-so a cor+ora'(o or a (3%) a ) %$(s'( & )%r ' % -a s o ' %(-(++( %s a ) % a %) ( ' % /&s( %ss o ( ( . r(va'% r%s+o )% ' (-%) a

A++-(ca'(o or E$+-ora'(o %r (' (' MG cov%r( ' % sa % +ro+%r'(%s cov%r%) /0a ) )&r( ' % s&/s(s'% c% o A SA6SF6""""8* a ) M SA No. 15!62"""61 o

Page 154: Nat Res Digested

8/10/2019 Nat Res Digested

http://slidepdf.com/reader/full/nat-res-digested 154/173

+%'('(o %r. I '&r , +%'('(o %r (-%) a %r( (%) ro'%s'KO++os('(o 'o ' % A++-(ca'(o or E$+-ora'(o %r (' o ' % +r(va'% r%s+o )% '. I' as a--% %)-0 (-%) (' ' % a %- o

Ar/('ra'ors o !" A& &s' 2""5 a ) as r%c%(v%) /0 ' % -a''%r o 5 S%+'% /%r 2""5.

r(or, o %v%r, 'o +%'('(o %rJs (-( o ('s %r( (%) ro'%s'KO++os('(o 'o ' % +r(va'%

r%s+o )% 'Js A++-(ca'(o or E$+-ora'(o %r (', +%'('(o %rJs M SA No. 15!62"""61 asca c%--%), a Mo'(o or R%co s()%ra'(o as -(<% (s% )% (%).

T % MG (ss&%) E No. "56""1 'o +r(va'% r%s+o )% '.

a %- o Ar/('ra'ors )(s (ss%) motu proprio ' % %r( (%) ro'%s'KO++os('(o o +%'('(o %r.%'('(o %r %-%va'%) /0 a++%a- 'o ' % MA (c as a-so )(s (ss%).

T % cas% as %-%va'%) 'o ' % Co&r' o a++%a-s /&' &) % ' as r% )%r%) a a( s' ' %+%'('(o %r.

% c%, ' (s +%'('(o .

ISSUE:

%' %r ' % a %- o Ar/('ra'ors as &r(s)(c'(o 'o ca c%-, )% 0 a )Kor r%vo<% E No."56""1 (ss&%) /0 MG 'o +r(va'% r%s+o )% '.

ELD:

NO. T % a %- o Ar/('ra'ors as o &r(s)(c'(o 'o ca c%-, )% 0 a )Kor r%vo<% E No."56""1 (ss&%) /0 MG 'o +r(va'% r%s+o )% '

S%c'(o 77 o R%+&/-(c Ac' No. 7*92 %s'a/-(s %s ' % &r(s)(c'(o o ' % a %- o Ar/('ra'ors, ' &s:

S%c. 77. &anel o *r itrators. $ $ $. (' ( ' (r'0 =!"> or<( )a0s, a '%r ' %s&/ (ss(o o ' % cas% /0 ' % +ar'(%s or )%c(s(o , t'/ ( /5 s'(55 '(9/ /! 5us)9/ (o*)g) (5 Bu*)s ) t)o to '/(* ( / ) / o t'/ ;o55o ) g

1. D)s ut/s ) 9o59) g *)g'ts to +) ) g (*/(s

2. D)s ut/s ) 9o59) g +) /*(5 (g*//+/ ts o* /*+)ts

!. D(s+&'%s ( vo-v( s&r ac% o %rs, occ&+a 's a )c-a( o-)%rsKco c%ss(o a(r%s a )

Page 155: Nat Res Digested

8/10/2019 Nat Res Digested

http://slidepdf.com/reader/full/nat-res-digested 155/173

Page 156: Nat Res Digested

8/10/2019 Nat Res Digested

http://slidepdf.com/reader/full/nat-res-digested 156/173

O 19 Oc'o/%r 1*77, r%s()% '(a- D%cr%% No. 1219 as (ss&%), r%4&(r( o-)%rs o s&/s(s'( a ) va-() +a'% 'a/-% ( ( c-a( s -oca'%) & )%r ' % +rov(s(o s o ' %

(-(++( % (-- o 1*"2 'o (-% a ( ( -%as% a++-(ca'(o (' ( o % =1> 0%ar ro ' %a++rova- o ' % D%cr%%. %'('(o %r accor)( -0 (-%) a ( ( -%as% a++-(ca'(o , /&'& )%r +ro'%s' , o 1! Oc'o/%r 1*78, (' a r%s%rva'(o a o'a'%) o ' % /ac< o ('s

a++-(ca'(o ' a' (' (s o' a(v( ('s r( 's ov%r ('s ( ( c-a( s & '(- ' % va-()('0 o r%s()% '(a- D%cr%% No. 1219 s a-- av% /%% +ass%) &+o /0 ' (s Co&r'.

O 1" Oc'o/%r 1*78, +%'('(o %r (-%) ' (s s+%c(a- c(v(- ac'(o orcertiorari a ) +ro (/('(o ,a--% ( ' a' (' as o o' %r +-a( , s+%%)0 a ) a)%4&a'% r% %)0 ( ' % or)( ar0 co&rs%o -a 'o +ro'%c' ('s r( 's =%$c%+' /0 sa() +%'('(o >. %'('(o %r assa(-s r%s()% '(a-D%cr%% No. 1219 as & co s'('&'(o a- ( ' a' (' a o& 's 'o a )%+r(va'(o o +ro+%r'0(' o&' )&% +roc%ss o -a .

%'('(o %r av%rs ' a' ('s ( '0 =5"> ( ( c-a( s a) a-r%a)0 /%% )%c-ar%) as ('s o+r(va'% a ) %$c-&s(v% +ro+%r'0 ( ( a- &) % 's. T % r%s+o )% 's, o ' % o' %r a ),

a--% % ' a' +%'('(o %r as o s'a )( 'o (-% ' % ( s'a ' +%'('(o as (' a(-%) 'o &--0%$ a&s' a) ( (s'ra'(v% r% %)(%s.

ISSUE:

%' %r or o' r%s()% '(a- D%cr%% No. 1219 (s co s'('&'(o a-.

ELD:

r%s()% '(a- D%cr%% No. 1219 (s o' & co s'('&'(o a-.

I' (s a va-() %$%rc(s% o ' % sov%r%( +o %r o ' % S'a'%, as o %r, ov%r -a )s o ' %+&/-(c )o a( , o (c +%'('(o %r s ( ( c-a( s s'(-- or a +ar', a ) ov%r ' %+a'r( o 0 o ' % a'(o , o (c ( %ra- )%+os('s ar% a va-&a/-% ass%'. I' a0 /%& )%rscor%), ( ' (s co %c'(o , ' a' ' % D%cr%% )o%s o' cov%r a-- ( ( c-a( s-oca'%) & )%r ' % (-. (-- o 1*"2, /&' o -0 ' os% c-a( s ov%r (c ' %(r -oca'ors a)a(-%) 'o o/'a( a +a'% '. A ) %v% ' % , s&c -oca'ors a0 s'(-- ava(- o ' % r% % a/-%' % '06 (v% 0%ar =25> -%as% +r%scr(/%) /0 r%s. D%c. No. 9#!, ' % M( %ra- D%v%-o+ % 'R%so&rc%s D%cr%% o 1*79.

r%s()% '(a- D%cr%% No. 1219 (s ( accor) (' S%c. 8, Ar'. I o ' % 1*7! Co s'('&'(o .

%'('(o (s )(s (ss%).

Page 157: Nat Res Digested

8/10/2019 Nat Res Digested

http://slidepdf.com/reader/full/nat-res-digested 157/173

G.R. No. 1!51*". A+r(- !, 2""2

SOUT EAST MINDANAO GOLD MINING COR ORATION, petitioner , vs . ALITEORTAL MINING COO ERATI E a ) o' %rs s( (-ar-0 s('&a'%) a ) T E ONORA LE

ANTONIO CERILLES, ( (s ca+ac('0 as S%cr%'ar0 o ' % D%+ar' % ' o E v(ro % 'a ) Na'&ra- R%so&rc%s =DENR>, RO INCIAL MINING REGULATOR OARD OFDA AO = MR 6Davao>, respondents .

FACTS:T % ( s'a ' cas% ( vo-v%s a r(c 'rac' o ( %ra- -a ) s('&a'%) ( ' % A &sa 6Davao6S&r( ao For%s' R%s%rv% < o as ' % D( a- a- Go-) R&s Ar%a.; Loca'%) a' M'.D( a'a ( ' % & (c(+a-('(%s o Mo <a0o a ) Ca'%%- ( Davao D%- Nor'%, ' % -a ) as/%% % /ro(-%) ( co 'rov%rs0 s( c% ' % ()68"Js )&% 'o ' % scra /-% ov%r o-))%+os('s o& ) (' ( ('s /o %-s.

O Marc 1", 1*88, Marco++%r M( ( Cor+ora'(o =Marco++%r> as ra '%)E$+-ora'(o %r (' No. 1!! =E No. 1!!> ov%r 9,9*1 %c'ar%s o -a ), (c ( c-&)%)' % o'-06co '%s'%) D( a- a- ar%a.

Page 158: Nat Res Digested

8/10/2019 Nat Res Digested

http://slidepdf.com/reader/full/nat-res-digested 158/173

No' -o ' %r%a '%r, Co r%ss % ac'%) o ?& % 27, 1**1 R%+&/-(c Ac' No. 7"7#, or ' %%o+-%Js S a--6Sca-% M( ( Ac'. T % -a %s'a/-(s %) a %o+-%Js S a--6Sca-% M( (ro ra 'o /% ( +-% % '%) /0 ' % S%cr%'ar0 o ' % DENR a ) cr%a'%) ' % rov( c(a-M( ( R% &-a'or0 oar) = MR > & )%r ' % DENR S%cr%'ar0Js )(r%c' s&+%rv(s(o a )

co 'ro-. T % s'a'&'% a-so a&' or(3%) ' % MR 'o )%c-ar% a ) s%' as()% s a--6sca-%( ( ar%as s&/ %c' 'o r%v(% /0 ' % DENR S%cr%'ar0 a ) a ar) ( ( co 'rac's 'os a--6sca-% ( %rs & )%r c%r'a( co )('(o s.

O D%c% /%r 21, 1**1, DENR S%cr%'ar0 F&- % c(o S. Fac'ora (ss&%) D%+ar' % ' A) ( (s'ra'(v% Or)%r =DAO> No. ##, )%c-ar( 72* %c'ar%s o ' % D( a- a- ar%a as

o 6 or%s' -a ) o+% 'o s a--6sca-% ( ( . T % (ss&a c% as a)% +&rs&a ' 'o ' %+o %rs v%s'%) ( ' % DENR S%cr%'ar0 /0 roc-a a'(o No. !#*, (c %s'a/-(s %) ' %

A &sa 6Davao6S&r( ao For%s' R%s%rv%.

O ?& % 29, 1**7, ' % DENR S%cr%'ar0 (ss&%) M% ora )& Or)%r No. *76"! (c

)(r%c's ' % DENR 'o study thoroughly and e haustively the option o direct stateutili1ation o the mineral resources in the !iwalwal "old2%ush *rea. S&c study shall include, /&' s a-- o' /% -( ('%) 'o, s'&)0( a ) weighing the easi ility o entering intomanagement agreements or operating agreements, or /o' , (' ' % a++ro+r(a'%ov%r % ' ( s'r& % 'a-('(%s or +r(va'% % '('(%s, or /o' , ( carr0( o&' ' % )%c-ar%)+o-(c0 o ra'(o a-(3( ' % ( ( o+%ra'(o s ( ' % D( a- a- Go-) R&s Ar%a s&cagreements shall include provisions or pro it2sharing /%' %% ' % s'a'% a ) ' % sa()+ar'(%s, including pro it2sharing arrangements with small2scale miners, as %-- as ' %+a0 % ' o ro0a-'(%s 'o ( )( % o&s c&-'&ra- co & ('(%s, a o o' %rs. T %U )%rs%cr%'ar0 or F(%-) O+%ra'(o s, as %-- as ' % U )%rs%cr%'ar0 or L% a- a )L% (s-a'(v% A a(rs a ) A''ac %) A % c(%s, a ) ' % D(r%c'or o ' % M( %s a ) G%o6

sc(% c%s &r%a& ar% %r%/0ordered to underta#e such studies. $ $ $%'('(o %r (-%) a s+%c(a- c(v(- ac'(o orcertiorari, prohi ition a ) mandamus /% or% ' %Co&r' o A++%a-s a a( s' MR 6Davao, ' % DENR S%cr%'ar0 a ) a-('% Co & a-or'a- M( ( Coo+%ra'(v% = C MC>, (c r%+r%s% '%) a-- ' % OT ra '%%s. I' +ra0%)or ' % &--( (ca'(o o ' % a/ov%64&o'%) M% ora )& Or)%r No. *76"! o ' % ro& )' a' ' % )(r%c' s'a'% &'(-(3a'(o ; %s+o&s%) ' %r%( o&-) % %c'(v%-0 ( +a(r ('s v%s'%)r( 's & )%r E No. 1!!.

T % Co&r' o A++%a-s )(s (ss%) ' % +%'('(o . I' r&-%) ' a' ' % DENR S%cr%'ar0 )() o'a/&s% (s )(scr%'(o ( (ss&( M% ora )& Or)%r No. *76"! s( c% ' % sa % as%r%-0 a )(r%c'(v% 'o co )&c' s'&)(%s o ' % var(o&s o+'(o s ava(-a/-% 'o ' % ov%r % 'or so-v( ' % D( a- a- co -(c'.

ISSUE:

Page 159: Nat Res Digested

8/10/2019 Nat Res Digested

http://slidepdf.com/reader/full/nat-res-digested 159/173

%' %r or o' ' % Co&r' o A++%a-s %rr%) % (' co c-&)%) ' a' ' % assa(-%)% ora )& or)%r )() o' a)o+' ' % )(r%c' s'a'% &'(-(3a'(o sc % %; ( r%so-v( ' %D( a- a- )(s+&'%.

%-):% a r%% (' ' % Co&r' o A++%a-sJ r&-( ' a' ' % c a--% %) MO *76"! )() o'

co c-&s(v%-0 a)o+' )(r%c' s'a'% &'(-(3a'(o ; as a +o-(c0 ( r%so-v( ' % D( a- a- )(s+&'%.T % '%r s o ' % % ora )& c-%ar-0 ( )(ca'% ' a' a' as )(r%c'%) ' %r%& )%r as%r%-0 astudy o ' (s o+'(o a ) o' ( %-s%. Co 'rar0 'o +%'('(o %rJs co '% '(o , (' )()o' ra ' a 0 a a % % 'Ko+%ra'( or +ro ('6s ar( a r%% % ' 'o s a--6sca-% ( %rsor 'o a 0 +ar'0, or ' a' a''%r, /&' s( +-0 ( s'r&c'%) ' % DENR o (c(a-s co c%r %) 'o& )%r'a<% s'&)(%s 'o )%'%r ( % ('s %as(/(-('0.

G.R. No. #***7. S%+'% /%r !", 1*87

UNGA MALO AGO MINES, INC., petitioner , vs . ON. INTERMEDIATE A ELLATECOURT, DIRECTOR OF LANDS, GREGORIA OLANOS, AUREA ARAO?O,GER ACIO ARAO?O, MARIA ERNAL, FELI DETECIO, ?ESUS ASUNCION,MELANIO ASUNCION a ) IEN ENIDO ASUNCION, respondents .

FACTS:

O ?&-0 2", 1*#2, ' % r%s()% ' o ' % (-(++( %s ra '%) ( ( +a'% 's o ( %ra-c-a( s -oca'%) a' U a0 Ma-o/a o, Ra+&6Ra+&, A-/a0 'o %r%( +%'('(o %rs a ) o' %r +r(va'% ( )(v()&a-s.

a0 /ac< o Oc'o/%r !", 1*5*, ?o Ca so , ?r. a ) Car-os S'(-(a o+&-os ass( %)' %(r r( 's 'o ' %(r ( ( c-a( s ( avor o ' % +%'('(o %r. T % ass( % ' o r( 's asr%cor)%) ( ' % O (c% o ' % M( ( R%cor)%r o A-/a0 o D%c% /%r 2, 1*5*.

Page 160: Nat Res Digested

8/10/2019 Nat Res Digested

http://slidepdf.com/reader/full/nat-res-digested 160/173

Page 161: Nat Res Digested

8/10/2019 Nat Res Digested

http://slidepdf.com/reader/full/nat-res-digested 161/173

r%so&rc%s s a-- /% ra '%) or a +%r(o) %$c%%)( ' % '06 (v% 0%ars, r% % a/-% or a o' %r ' % '06 (v% 0%ars, %$c%+' as 'o a'%r r( 's or (rr( a'(o , a'%r s&++-0, (s %r(%s,or ( )&s'r(a- &s%s o' %r ' a ' % )%v%-o+ % ' o a'%r +o %r, ( (c cas%s /% % (c(a-&s% a0 /% ' % %as&r% a ) ' % -( (' o ' % ra '. =E + as(s s&++-(%)>

T %r% or%, a++-0( ' % a or%4&o'%) +rov(s(o 'o ' % cas% a' /ar, % co c-&)% ' a' ' %(ss&a c% o ' % -o)% +a'% 's o ( %ra- c-a( s /0 ' % r%s()% ' o ' % (-(++( %s (1*#2 ( avor o ' % +%'('(o %r ra '%) 'o (' o -0 ' % r( ' 'o %$'rac' or &'(-(3% ' % ( %ra-s(c a0 /% o& ) o or & )%r ' % s&r ac% o ' % -a ). O ' % o' %r a ), ' %

(ss&a c% o ' % r%% +a'% 's /0 ' % r%s+o )% ' D(r%c'or o La )s ( 1*7* ( avor o ' %+r(va'% r%s+o )% 's ra '%) 'o ' % ' % o %rs (+ a ) ' % r( ' 'o &s% ' % -a ) or a r(c&-'&ra- +&r+os%s /&' %$c-&)( ' % o %rs (+ o , a ) ' % r( ' 'o %$'rac' or &'(-(3%,' % ( %ra-s (c a0 /% o& ) o or & )%r ' % s&r ac%.

T %r% (s o /as(s ( ' % r%cor)s or ' % +%'('(o %r s s'a ) ' a' (' ac4&(r%) ' % r( ' 'o ' %( %ra- -a )s +r(or 'o ' % % %c'(v('0 o ' % 1*!5 Co s'('&'(o , ' &s, a<( s&c

ac4&(s('(o o&'s()% ('s +&rv(% a ) sco+%. A % ' ' % s%co ) (ss&%, ' % +%'('(o %r as o +%rso a-('0 'o ( s'('&'% ' % ac'(o /%-o or a &- % ' a ) ca c%--a'(o o +a'% 's. T % ( %ra- -a )s ov%r (c (' as a r( ' 'o%$'rac' ( %ra-s r% a( %) +ar' o ' % ( a-(% a/-% -a )s o ' % +&/-(c )o a( a ) ' &s,o -0 ' % So-(c('or G% %ra- or ' % +%rso ac'( ( (s s'%a) ca /r( a ac'(o or r%v%rs(o .

LOCAL GO ERNANCE CASES:

G.R. No. 11"29*. A& &s' 21, 1**7

ALFREDO TANO, ET AL, petitioners,

vs .

GO . SAL ADOR . SOCRATES ET AL respondents.

3acts4O D%c% /%r 15, 1**2, ' % Sa & (a a -& so) &%r'o r( c%sa C('0 % ac'%)Or)( a c% No. 156*2 (c 'oo< % %c' o ?a &ar0 1, 1**! % '('-%): AN ORDINANCEANNING T E S I MENT OF ALL LI E FIS AND LO STER OUTSIDE UERTORINCESA CIT FROM ?ANUAR 1, 1**! TO ?ANUAR 1, 1**8 AND RO IDINGE EM TIONS, ENALTIES AND FOR OT ER UR OSES T EREOF.

Page 162: Nat Res Digested

8/10/2019 Nat Res Digested

http://slidepdf.com/reader/full/nat-res-digested 162/173

To ( +-% % ' sa() c('0 or)( a c%, ' % Ac'( C('0 Ma0or A a)o L. L&c%ro (ss&%) O (c%Or)%r No. 2!, S%r(%s o 1**! )a'%) ?a &ar0 22, 1**! (c r%a)s as o--o s:

I ' % ( '%r%s' o +&/-(c s%rv(c% a ) or +&r+os%s o C('0 Or)( a c% No. D92#619679,o' %r (s% < o as AN ORDINANCE RE@UIRING AN ERSON ENGAGED OR

INTENDING TO ENGAGE IN AN USINESS, TRADE, OCCU ATION, CALLING ORROFESSION OR A ING IN IS OSSESSION AN OF T E ARTICLES FORIC A ERMIT IS RE@UIRED TO E AD, TO O TAIN FIRST A MA ORJS

ERMIT; a ) C('0 Or)( a c% No. 156*2, AN ORDINANCE ANNING T E S I MENTOF ALL LI E FIS AND LO STER OUTSIDE UERTO RINCESA CIT FROM?ANUAR 1, 1**! TO ?ANUAR 1, 1**8, 0o& ar% %r%/0 a&' or(3%) a ) )(r%c'%) 'oc %c< or co )&c' %c%ssar0 ( s+%c'(o s o car o%s co 'a( ( -(v% (s a ) -o/s'%r /%( s (++%) o&' ro ' % &%r'o r( c%sa A(r+or', &%r'o r( c%sa ar or a' a 0+or' (' ( ' % &r(s)(c'(o o ' % C('0 'o a 0 +o( ' o )%s'( a'(o s s(c %(' %r v(a a(rcra 'or s%acra '.

O F%/r&ar0 1*, 1**!, ' % Sa & (a a -a-a ( a , rov( c(a- Gov%r % ' o a-a a % ac'%) R%so-&'(o No. !! % '('-%): A RESOLUTION RO I ITING T ECATC ING, GAT ERING, OSSESSING, U ING, SELLING AND S I MENT OFLI E MARINE CORAL D ELLING A@UATIC ORGANISMS, TO IT: FAMIL:SCARIDAE =MAMENG>, E INE ELUS FASCIATUS =SUNO>. CROMILE TES

ALTI ELIS = ANT ER OR SENORITA>, LO STER ELO 2"" GRAMS ANDS A NING, TRADACNA GIGAS =TABLO O>, INCTADA MARGARITEFERA=MOT ER EARL, O STERS, GIANT CLAMS AND OT ER S ECIES>, ENAEUSMONODON =TIGER RA N6 REEDER SIZE OR MOT ER>, E INE ELUSSUILLUS =LO A OR GREEN GROU ER> AND FAMIL: ALISTIDAE =TRO ICAL

A@UARIUM FIS ES> FOR A ERIOD FI E =5> EARS IN AND COMING FROM

ALA AN ATERS,a ),

ORDINANCE NO. 2, S%r(%s o 1**!

E '('-%), Or)( a c% ro (/('( ' % ca'c ( , a' %r( , +oss%ss( , /&0( , s%--( a )s (+ % ' o -(v% ar( % cora- ) %--( a4&a'(c or a (s s, 'o (': 1. Fa (-0: Scar()a%=Ma % >, 2. E+( %+ %-&s Fasc(a'&s =S& o>, !. Cro (-%+'%s a-'(v%-(s = a ' %r or S% or('a>, -o/s'%r /%-o 2"" ra s a ) s+a ( >, 9. Tr()ac a G( as =Ta<-o/o>, 5.( c'a)a Mar ar%'% %ra =Mo' %r %ar-, O0s'%rs, G(a ' C-a s a ) o' %r s+%c(%s>, #.% a%&s Mo o)o =T( %r ra 6/r%%)%r s(3% or o' %r>, 7. E+( %+ %-&s S&(--&s =Lo/aor Gr%% Gro&+%r> a ) 8. Fa (-0: a-(s'()a% =To+(ca- A4&ar(& F(s %s> or a +%r(o) o (v% =5> 0%ars ( a ) co ( ro a-a a a'%rs.

R%s+o )% 's ( +-% % '%) ' % sa() or)( a c%s, ' %r%/0 )%+r(v( a-- ' % (s %r % o ' % o-% +rov( c% o a-a a a ) ' % C('0 o &%r'o r( c%sa o ' %(r o -0 %a s o -(v%-( oo) a ) ' % +%'('(o %rs A(r-( % S (++%rs Assoc(a'(o o a-a a a ) o' %r ar( %%rc a 's ro +%r or ( ' %(r -a &- occ&+a'(o a ) 'ra)%.

Page 163: Nat Res Digested

8/10/2019 Nat Res Digested

http://slidepdf.com/reader/full/nat-res-digested 163/173

%'('(o %rs (-%) ' (s +%'('(o )(r%c'-0 (' ' % COURT a--% ( ' a' ' % Or)( a c%s)%+r(v%) ' % o )&% +roc%ss o -a , ' %(r -(v%-( oo), a ) & )&-0 r%s'r(c'%) ' % ro' % +rac'(c% o ' %(r 'ra)%, ( v(o-a'(o o S%c'(o 2, Ar'(c-% II a ) S%c'(o s 2 a ) 7 o

Ar'(c-% III o ' % 1*87 Co s'('&'(o ' a' ' % O (c% Or)%r No. 2! co 'a( %) o r% &-a'(oor co )('(o & )%r (c ' % Ma0orJs +%r (' co&-) /% ra '%) or )% (%) ( o' %r

or)s, ' % Ma0or a) ' % a/so-&'% a&' or('0 'o )%'%r ( % %' %r or o' 'o (ss&% +%r ('a ) ' a' Or)( a c% No. 2 o ' % rov( c% o a-a a a-'o %' %r +ro (/('%) ' %ca'c ( , a' %r( , +oss%ss(o , /&0( , s%--( a ) s (++( o -(v% ar( % cora-) %--( or a (s s, (' o&' a 0 )(s'( c'(o %' %r (' as ca& ' or a' %r%) ' ro&-a &- (s ( %' o),; ' % Or)( a c% 'oo< a a0 ' % r( ' o +%'('(o %rs6 (s %r % 'o%ar ' %(r -(v%-( oo) ( -a &- a0s a ) ( so ar as +%'('(o %rs6 % /%rs o A(r-( %S (++%rs Assoc(a'(o ar% co c%r %), ' %0 %r% & )&-0 +r%v% '%) ro +&rs&( ' %(r voca'(o a ) % '%r( ( 'o co 'rac's (c ar% +ro+%r, %c%ssar0, a ) %ss% '(a- 'o carr0o&' ' %(r /&s( %ss % )%avors 'o a s&cc%ss &- co c-&s(o .;

A ) ( a--0, 'o )%c-ar% Or)( a c% No. 2 o ' % Sa & (a a -a-a ( a as &-- a )

vo(),ISSUE:

ET ER OR NOT T E ASSAILED ORDINANCES AREUNCONSTITUTIONAL.

ELD:

I' (s o co&rs% s%''-%) ' a' -a s =( c-&)( or)( a c%s % ac'%) /0 -oca- ov%r % ' & ('s>% o0 ' % +r%s& +'(o o co s'('&'(o a-('0. To ov%r' ro ' (s +r%s& +'(o , ' %r% &s' /%a c-%ar a ) & %4&(voca- /r%ac o ' % Co s'('&'(o , o' %r%-0 a )o&/' &- or ar & % 'a'(v% co 'ra)(c'(o . I s or', ' % co -(c' (' ' % Co s'('&'(o &s' /% s o/%0o ) r%aso a/-% )o&/'. %r% )o&/' %$(s's, %v% ( %-- o& )%), ' %r% ca /% o( )( o & co s'('&'(o a-('0. To )o&/' (s 'o s&s'a( .

A '%r a scr& ('0 o ' % c a--% %) Or)( a c%s a ) ' % +rov(s(o s o ' % Co s'('&'(o+%'('(o %rs c-a( 'o av% /%% v(o-a'%), % ( ) +%'('(o %rsJ co '% '(o s /as%-%ss a ) soo-) ' a' ' % or %r )o o' s& %r ro a 0 ( (r ('0, /o' & )%r ' % Co s'('&'(o a )a++-(ca/-% -a s.

U )%r ' % % %ra- %- ar% c-a&s% o ' % LGC, -oca- ov%r % ' & ('s av% ' % +o %r,inter alia , 'o % ac' or)( a c%s 'o % a c% ' % r( ' o ' % +%o+-% 'o a /a-a c%) %co-o 0.I' -(<% (s% s+%c( (ca--0 v%s's & (c(+a-('(%s (' ' % +o %r 'o ra ' (s %r0 +r(v(-% %s (& (c(+a- a'%rs, a ) ( +os% r% 'a-s, %%s or c ar %s ' %r% or 'o +% a-(3%, /0

a++ro+r(a'% or)( a c%s, ' % &s% o %$+-os(v%s, o$(o&s or +o(so o&s s&/s'a c%s,%-%c'r(c('0,muro2ami , a ) o' %r )%-%'%r(o&s %' o)s o (s ( a ) 'o +ros%c&'% o' %r

Page 164: Nat Res Digested

8/10/2019 Nat Res Digested

http://slidepdf.com/reader/full/nat-res-digested 164/173

%' o)s o (s ( a ) 'o +ros%c&'% a 0 v(o-a'(o o ' % +rov(s(o s o a++-(ca/-% (s (-a s. F( a--0, (' ( +os%s &+o ' % sangguniang ayan, ' % sangguniang panlungsod,a ) ' % sangguniang panlalawigan ' % )&'0 'o % ac' or)( a c%s 'o + ro'%c' ' %% v(ro % ' a ) ( +os% a++ro+r(a'% +% a-'(%s or ac's (c % )a %r ' % % v(ro % 's&c as )0 a ('% (s ( a ) o' %r or s o )%s'r&c'(v% (s ( a ) s&c o' %r

ac'(v('(%s (c r%s&-' ( +o--&'(o , acc%-%ra'(o o %&'ro+ (ca'(o o r(v%rs a ) -a<%s or o %co-o (ca- ( /a-a c%.;

G.R. No. L69"29!. Marc 11, 1**2

CELESTINO TATEL, petitioner , vs . MUNICI ALIT OF IRAC, SAL ADOR A.SURTIDA, ( (s ca+ac('0 as Ma0or o (rac, Ca'a )&a %s GA INO . GUERRERO, ((s ca+ac('0 as (c%6Ma0or o (rac, Ca'a )&a %s ?OSE T. UE OS, ( (s ca+ac('0 asCo& c(-or o (rac, Ca'a )&a %s ANGELES TA LIZO, ( (s ca+ac('0 as Co& c(-or o (rac, Ca'a )&a %s EL IDIO T. ZAFE, ( (s ca+ac('0 as Co& c(-or o (rac, Ca'a 6)&a %s MARIANO AL ERTO, ( (s ca+ac('0 as Co& c(-or o (rac, Ca'a )&a %s?ULIA A. GARCIA, ( %r ca+ac('0 as Co& c(-or o (rac, Ca'a )&a %s a ) EDRO A.GUERRERO, ( (s ca+ac('0 as Co& c(-or o (rac, Ca'a )&a %s, respondents .

FACTS:

C%-%s'( o Ta'%-, a /&s( %ss a , (s % a %) ( ' % ( +or' a ) %$+or' o a/aca a ) o' %r +ro)&c's. % as a ar% o&s% ( /arr(o S'a. E-% a.

R%s()% 's o /arr(o S'a. E-% a (-%) a co +-a( ' a a( s' +%'('(o %r. T %0 a--% %) ' a' ' %)(s'&r/a c% ca&s%) /0 ' % o+%ra'(o o ' % a/aca /a(-( ac ( % ( s()% ' % ar% o&s%

Page 165: Nat Res Digested

8/10/2019 Nat Res Digested

http://slidepdf.com/reader/full/nat-res-digested 165/173

o +%'('(o %r a %c'%) ' % +%ac% a ) 'ra 4&(-('0 o ' % %( /or oo) )&% 'o ' % s o<%,o/ o$(o&s o)or a ) )&s' % (''%) /0 ' % ac ( %. A co (''%% as a++o( '%) /0 ' %& (c(+a- co& c(- o (rac 'o ( v%s'( a'% ' % a''%r. T % co (''%% o'%) ' % cro )%)a'&r% o ' % %( /or oo) (' arro roa)s a ) ' % s&rro& )( r%s()% '(a- o&s%s,so &c so ' a' a acc()% 'a- (r% (' ( ' % ar% o&s% o +%'('(o %r occas(o %) /0 a

co '( &a c% o ' % ac'(v('0 ( s()% ' % ar% o&s% a ) ' % s'or( o ( -a a/-%a'%r(a-s cr%a'%) a )a %r 'o ' % -(v%s a ) +ro+%r'(%s o ' % +%o+-% (' ( ' %%( /or oo).

R%s&-'a '-0, R%so-&'(o No. 2* as +ass%) /0 ' % M& (c(+a- Co& c(- o (rac o A+r(- 22,1*## )%c-ar( ' % ar% o&s% o %) a ) o+%ra'%) /0 +%'('(o %r a +&/-(c &(sa c%(' ( ' % +&rv(% o Ar'(c-% #*9 o ' % N% C(v(- Co)%.

(s o'(o or r%co s()%ra'(o av( /%% )% (%) /0 ' % M& (c(+a- Co& c(- o (rac.

%'('(o %r ( s'('&'%) ' % +r%s% ' +%'('(o or +ro (/('(o (' +r%-( ( ar0 ( & c'(o .

ISSUES:

ET ER OR NOT +%'('(o %r s ar% o&s% (s a &(sa c% (' ( ' % %a ( o Ar'(c-%#*9 o ' % C(v(- Co)% a ) %' %r Or)( a c% No. 1!, S. 1*52 o ' % M& (c(+a-('0 o (rac (s & co s'('&'(o a- a ) vo().

ELD:Or)( a c% No. 1!, s%r(%s o 1*52, as +ass%) /0 ' % M& (c(+a- Co& c(- o (rac ( ' %%$%rc(s% o ('s +o-(c% +o %r. I' (s a s%''-%) +r( c(+-% o -a ' a' & (c(+a- cor+ora'(o sar% a % c(%s o ' % S'a'% or ' % +ro o'(o a ) a( '% a c% o -oca- s%- 6 ov%r % 'a ) as s&c ar% % )o %) (' +o-(c% +o %rs ( or)%r 'o % %c'(v%-0 acco +-(s a )carr0 o&' ' % )%c-ar%) o/ %c's o ' %(r cr%a'(o . I's a&' or('0 % a a'%s ro ' % % %ra-%- ar% c-a&s% & )%r ' % A) ( (s'ra'(v% Co)%.

For a or)( a c% 'o /% va-(), (' &s' o' o -0 /% (' ( ' % cor+ora'% +o %rs o ' %& (c(+a-('0 'o % ac' /&' &s' a-so /% +ass%) accor)( 'o ' % +roc%)&r% +r%scr(/%) /0

-a , a ) &s' /% ( co so a c% (' c%r'a( %-- %s'a/-(s %) a ) /as(c +r( c(+-%s o as&/s'a '(v% a'&r%. T %s% +r( c(+-%s r%4&(r% ' a' a & (c(+a- or)( a c% =1> &s' o'co 'rav% % ' % Co s'('&'(o or a 0 s'a'&'% =2> &s' o' /% & a(r or o++r%ss(v% =!> &s'o' /% +ar'(a- or )(scr( ( a'or0 =9> &s' o' +ro (/(' /&' a0 r% &-a'% 'ra)% =5> &s' /%% %ra- a ) co s(s'% ' (' +&/-(c +o-(c0, a ) =#> &s' o' /% & r%aso a/-%. Or)( a c%No. 1!, S%r(%s o 1*52, %%'s ' %s% cr('%r(a.

Page 166: Nat Res Digested

8/10/2019 Nat Res Digested

http://slidepdf.com/reader/full/nat-res-digested 166/173

LAND TENURE CASES:

G.R. No. 8#88*. D%c% /%r 9, 1**"

LUZ FARMS, petitioner , vs . T E ONORA LE SECRETAR OF T E DE ARTMENTOF AGRARIAN REFORM,respondent .

FACTS:

O ?& % 1", 1*88, ' % r%s()% ' o ' % (-(++( %s a++rov%) R.A. No. ##57, (c

( c-&)%s ' % ra(s( o -(v%s'oc<, +o&-'r0 a ) s ( % ( ('s cov%ra %.O ?a &ar0 2, 1*8*, ' % S%cr%'ar0 o A rar(a R% or +ro &- a'%) ' % G&()%-( %s a )roc%)&r%s I +-% % '( ro)&c'(o a ) ro (' S ar( as % /o)(%) ( S%c'(o s 1!a ) !2 o R.A. No. ##57.

O ?a &ar0 *, 1*8*, ' % S%cr%'ar0 o A rar(a R% or +ro &- a'%) ('s R&-%s a )R% &-a'(o s ( +-% % '( S%c'(o 11 o R.A. No. ##57 =Co %rc(a- Far s>.

Page 167: Nat Res Digested

8/10/2019 Nat Res Digested

http://slidepdf.com/reader/full/nat-res-digested 167/173

L&3 Far s, +%'('(o %r ( ' (s cas%, (s a cor+ora'(o % a %) ( ' % -(v%s'oc< a ) +o&-'r0/&s( %ss a ) 'o %' %r (' o' %rs ( ' % sa % /&s( %ss a--% %)-0 s'a )s 'o /%a)v%rs%-0 a %c'%) /0 ' % % orc% % ' o S%c'(o !=/>, S%c'(o 11, S%c'(o 1!, S%c'(o1#=)> a ) 17 a ) S%c'(o !2 o R.A. No. ##57 o' %r (s% < o as Co +r% % s(v%

A rar(a R% or La a ) o ' % G&()%-( %s a ) roc%)&r%s I +-% % '( ro)&c'(o

a ) ro (' S ar( & )%r R.A. No. ##57 +ro &- a'%) o ?a &ar0 2, 1*8* a ) ' % R&-%sa ) R% &-a'(o s I +-% % '( S%c'(o 11 ' %r%o as +ro &- a'%) /0 ' % DAR o?a &ar0 *, 1*8*.

ISSUE:

%' %r or o' S%c'(o s !=/>, 11, 1! a ) !2 o R.A. No. ##57, ' % G&()%-( %s a )roc%)&r%s I +-% % '( ro)&c'(o a ) ro (' S ar( & )%r R.A. No. ##57 a ) ' %R&-%s a ) R% &-a'(o s I +-% % '( S%c'(o 11 ar% & co s'('&'(o a-.

ELD:

S%c'(o s !=/>, 11, 1! a ) !2 o R.A. No. ##57 ( so ar as ' % ( c-&s(o o ' % ra(s( o -(v%s'oc<, +o&-'r0 a ) s ( % ( ('s cov%ra % as %-- as ' % I +-% % '( R&-%s a )G&()%-( %s +ro &- a'%) ( accor)a c% ' %r% (' , ar% &-- a ) vo() or /%(& co s'('&'(o a-.

S%c'(o II o R.A. ##57 (c ( c-&)%s +r(va'% a r(c&-'&ra- -a )s )%vo'%) 'o co %rc(a--(v%s'oc<, +o&-'r0 a ) s ( % ra(s( ; ( ' % )% ( ('(o o co %rc(a- ar s (s ( va-(), 'o' % %$'% ' ' a' ' % a or%c('%) a ro6( )&s'r(a- ac'(v('(%s ar% a)% 'o /% cov%r%) /0 ' %a rar(a r% or +ro ra o ' % S'a'%. T %r% (s s( +-0 o r%aso 'o ( c-&)% -(v%s'oc<a ) +o&-'r0 -a )s ( ' % cov%ra % o a rar(a r% or . T % 'ra scr(+'s o ' % )%-(/%ra'(o s

o ' % Co s'('&'(o a- Co (ss(o o 1*8# o ' % %a ( o ' % or) a r(c&-'&ra-,c-%ar-0 s o ' a' (' as %v%r ' % ( '% '(o o ' % ra %rs o ' % Co s'('&'(o 'o ( c-&)%-(v%s'oc< a ) +o&-'r0 ( )&s'r0 ( ' % cov%ra % o ' % co s'('&'(o a--06 a )a'%) a rar(ar% or +ro ra o ' % Gov%r % '.

T % r%4&(r% % ' ( S%c'(o s 1! a ) !2 o R.A. ##57 )(r%c'( cor+ora'% ar s; (c( c-&)% -(v%s'oc< a ) +o&-'r0 ra(s%rs 'o %$%c&'% a ) ( +-% % ' +ro)&c'(o 6s ar(+-a s; =+% )( ( a- r%)(s'r(/&'(o o ' %(r -a ) o-)( s> %r%/0 ' %0 ar% ca--%) &+o'o )(s'r(/&'% ro ' r%% +%rc% ' =! > o ' %(r ross sa-%s a ) '% +%rc% ' =1" > o ' %(r %' +ro ('s 'o ' %(r or<%rs as a))('(o a- co +% sa'(o (s & r%aso a/-% or /%(

co (sca'or0, a ) ' %r% or% v(o-a'(v% o )&% +roc%ss.

Page 168: Nat Res Digested

8/10/2019 Nat Res Digested

http://slidepdf.com/reader/full/nat-res-digested 168/173

G.R. No. 78792. ?&-0 19, 1*8*

ASSOCIATION OF SMALL LANDO NERS IN T E ILI INES, INC., ?UANITO D.

GOMEZ, GERARDO . ALARCIO, FELI E A. GUICO, ?R., ERNARDO M. ALMONTE,CANUTO RAMIR . CA RITO, ISIDRO T. GUICO, FELISA I. LLAMIDO, FAUSTO ?.SAL A, RE NALDO G. ESTRADA, FELISA C. AUTISTA, ESMENIA ?. CA E,TEODORO . MADRIAGA, AUREA ?. RESTOSA, EMERENCIANA ?. ISLA,FELICISIMA C. A RESTO, CONSUELO M. MORALES, EN?AMIN R. SEGISMUNDO,CIRILA A. ?OSE V NA OLEON S. FERRER, petitioners, vs. ONORA LESECRETAR OF AGRARIAN REFORM, respondent .

G.R. No. 7*!1". ?&-0 19, 1*8*

ARSENIO AL. ACUWA, NE TON ?ISON, ICTORINO FERRARIS, DENNIS ?EREZA,

ERMINIGILDO GUSTILO, AULINO D. TOLENTINO a ) LANTERS COMMITTEE,INC., (c'or(as M(-- D(s'r(c', (c'or(as, N% ros Occ()% 'a-, petitioners , vs . ?OBER ARRO O, ILI E. ?UICO a ) RESIDENTIAL AGRARIAN REFORM COUNCIL,respondents .

G.R. No. 7*799. ?&-0 19, 1*8*

Page 169: Nat Res Digested

8/10/2019 Nat Res Digested

http://slidepdf.com/reader/full/nat-res-digested 169/173

INOCENTES A ICO, petitioner , vs . ON. ILI E. ?UICO, SECRETAR OF T EDE ARTMENT OF AGRARIAN REFORM, ON. ?OBER ARRO O, E ECUTI ESECRETAR OF T E OFFICE OF T E RESIDENT, a ) M%ssrs. SALADORTALENTO, ?AIME A OGADO, CONRADO AANCEWA, a ) RO ERTO TAA,respondents .

G.R. No. 7*777. ?&-0 19, 1*8*

NICOLAS S. MANAA a ) AGUSTIN ERMANO, ?R., petitioners , vs . ON. ILIELLA ?UICO, as S%cr%'ar0 o A rar(a R% or , a ) LAND ANB OF T EILI INES, respondents .

FACTS:

G.R. No. 7*777S4&ar%-0 ra(s%) ( ' (s +%'('(o (s ' % co s'('&'(o a-('0 o .D. No. 27, E.O. Nos. 228 a )22*, a ) R.A. No. ##57.

T % s&/ %c's o ' (s +%'('(o ar% a *6 %c'ar% r(c%-a ) or<%) /0 o&r '% a 's a ) o %)/0 +%'('(o %r N(co-as Ma aa0 a ) (s ( % a ) a 56 %c'ar% r(c%-a ) or<%) /0 o&r '% a 's a ) o %) /0 +%'('(o %r A &s'( %r a o, ?r. T % '% a 's %r% )%c-ar%) &--o %rs o ' %s% -a )s /0 E.O. No. 228 as 4&a-( (%) ar %rs & )%r .D. No. 27.

T % +%'('(o %rs ar% 4&%s'(o ( .D. No. 27 a ) E.O. Nos. 228 a ) 22* o ro& )s ( '%r

a-(a o s%+ara'(o o +o %rs, )&% +roc%ss, %4&a- +ro'%c'(o a ) ' % co s'('&'(o a--( ('a'(o ' a' o +r(va'% +ro+%r'0 s a-- /% 'a<% or +&/-(c &s% (' o&' &s'co +% sa'(o .

T %0 co '% ) ' a' r%s()% ' A4&( o &s&r+%) -% (s-a'(v% +o %r % s % +ro &- a'%)E.O. No. 228. T % sa() %as&r% (s ( va-() a-so or v(o-a'(o o Ar'(c-% III, S%c'(o 9, o ' % Co s'('&'(o , or a(-&r% 'o +rov()% or r%'% '(o -( ('s or s a-- -a )o %rs.Mor%ov%r, (' )o%s o' co or 'o Ar'(c-% I, S%c'(o 25=9> a ) ' % o' %r r%4&(s('%s o ava-() a++ro+r(a'(o .

T % +%'('(o %rs a-so a( 'a( ' a' ( )%c-ar( ' % /% % (c(ar(%s & )%r .D. No. 27 'o /%

' % o %rs o ' % -a )s occ&+(%) /0 ' % , E.O. No. 228 ( or%) &)(c(a- +r%ro a'(v%sa ) so v(o-a'%) )&% +roc%ss. ors%, ' % %as&r% o&-) o' so-v% ' % a rar(a +ro/-%/%ca&s% %v% ' % s a-- ar %rs ar% )%+r(v%) o ' %(r -a )s a ) ' % r%'% '(o r( 's&ara '%%) /0 ' % Co s'('&'(o .

G.R. No. 7*!1"

Page 170: Nat Res Digested

8/10/2019 Nat Res Digested

http://slidepdf.com/reader/full/nat-res-digested 170/173

T % +%'('(o %rs %r%( ar% -a )o %rs a ) s& ar +-a '%rs ( ' % (c'or(as M(-- D(s'r(c',(c'or(as, N% ros Occ()% 'a-. Co6+%'('(o %r -a '%rs Co (''%%, I c. (s a or a (3a'(oco +os%) o 1,9"" +-a '%r6 % /%rs. T (s +%'('(o s%%<s 'o +ro (/(' ' % ( +-% % 'a'(oo roc. No. 1!1 a ) E.O. No. 22*.

T % +%'('(o %rs c-a( ' a' ' % +o %r 'o +rov()% or a Co +r% % s(v% A rar(a R% orro ra as )%cr%%) /0 ' % Co s'('&'(o /%-o s 'o Co r%ss a ) o' ' % r%s()% '. A-' o& ' %0 a r%% ' a' ' % r%s()% ' co&-) %$%rc(s% -% (s-a'(v% +o %r & '(- ' %Co r%ss as co v% %), s % co&-) )o so o -0 'o % ac' % %r % c0 %as&r%s )&r(' % 'ra s('(o +%r(o). A' ' a', %v% ass& ( ' a' ' % ( '%r( -% (s-a'(v% +o %r o ' %r%s()% ' as +ro+%r-0 %$%rc(s%), roc. No. 1!1 a ) E.O. No. 22* o&-) s'(-- av% 'o /%a &--%) or v(o-a'( ' % co s'('&'(o a- +rov(s(o s o &s' co +% sa'(o , )&% +roc%ss,a ) %4&a- +ro'%c'(o .

T %0 co '% ) ' a' 'a<( &s' /% s( &-'a %o&s (' +a0 % ' o &s' co +% sa'(o as ('(s 'ra)('(o a--0 & )%rs'oo), (.%., (' o %0 a ) ( &--, /&' o s&c +a0 % ' (s

co '% +-a'%) ( S%c'(o 5 o ' % E.O. No. 22*G.R. No. 7*799

T % +%'('(o %r a--% %s ' a' ' % ' % S%cr%'ar0 o D%+ar' % ' o A rar(a R% or , (v(o-a'(o o )&% +roc%ss a ) ' % r%4&(r% % ' or &s' co +% sa'(o , +-ac%) (s-a ) o-)( & )%r ' % cov%ra % o O+%ra'(o La ) Tra s %r C%r'( (ca'%s o La )Tra s %r %r% s&/s%4&% '-0 (ss&%) 'o ' % +r(va'% r%s+o )% 's, o ' % r% &s%)+a0 % ' o -%as% r% 'a-s 'o ( .

O S%+'% /%r !, 1*8#, ' % +%'('(o %r +ro'%s'%) ' % %rro %o&s ( c-&s(o o (s s a--

-a ) o-)( & )%r O+%ra'(o La ) Tra s %r a ) as<%) or ' % r%ca-- a ) ca c%--a'(o o ' % C%r'( (ca'%s o La ) Tra s %r ( ' % a % o ' % +r(va'% r%s+o )% 's. % c-a( s ' a'o D%c% /%r 29, 1*8#, (s +%'('(o as )% (%) (' o&' %ar( . O F%/r&ar0 17, 1*87,% (-%) a o'(o or r%co s()%ra'(o , (c a) o' /%% ac'%) &+o % E.O. Nos.228 a ) 22* %r% (ss&%). T %s% or)%rs r% )%r%) (s o'(o oo' a ) aca)% (c/%ca&s% ' %0 )(r%c'-0 % %c'%) ' % 'ra s %r o (s -a ) 'o ' % +r(va'% r%s+o )% 's.

T % +%'('(o %r o ar &%s ' a':

=1> E.O. Nos. 228 a ) 22* %r% ( va-()-0 (ss&%) /0 ' % r%s()% ' o ' % (-(++( %s.

=2> T % sa() %$%c&'(v% or)%rs ar% v(o-a'(v% o ' % co s'('&'(o a- +rov(s(o ' a' o +r(va'%+ro+%r'0 s a-- /% 'a<% (' o&' )&% +roc%ss or &s' co +% sa'(o .

=!> T % +%'('(o %r (s )% (%) ' % r( ' o a$( & r%'% '(o +rov()%) or & )%r ' % 1*87Co s'('&'(o .

G.R. No. 78792

Page 171: Nat Res Digested

8/10/2019 Nat Res Digested

http://slidepdf.com/reader/full/nat-res-digested 171/173

T % +%'('(o %rs ( ' (s cas% ( vo<% ' % r( ' o r%'% '(o ra '%) /0 .D. No. 27 'oo %rs o r(c% a ) cor -a )s o' %$c%%)( s%v% %c'ar%s as -o as ' %0 ar%c&-'(va'( or ( '% ) 'o c&-'(va'% ' % sa %. T %(r r%s+%c'(v% -a )s )o o' %$c%%) ' %s'a'&'or0 -( (' /&' ar% occ&+(%) /0 '% a 's o ar% ac'&a--0 c&-'(va'( s&c -a )s.

Accor)( 'o .D. No. !1#, (c as +ro &- a'%) ( ( +-% % 'a'(o o .D. No. 27:No '% a '6 ar %r ( a r(c&-'&ra- -a )s +r( ar(-0 )%vo'%) 'o r(c% a ) cor s a-- /% % %c'%)or r% ov%) ro (s ar o-)( & '(- s&c '( % as ' % r%s+%c'(v% r( 's o ' % '% a '6ar %rs a ) ' % -a )o %r s a-- av% /%% )%'%r ( %) ( accor)a c% (' ' % r&-%sa ) r% &-a'(o s ( +-% % '( .D. No. 27.

T % +%'('(o %rs c-a( ' %0 ca o' % %c' ' %(r '% a 's a ) so ar% & a/-% 'o % o0 ' %(r r( ' o r%'% '(o /%ca&s% ' % D%+ar' % ' o A rar(a R% or as so ar o' (ss&%) ' %( +-% % '( r&-%s r%4&(r%) & )%r ' % a/ov%64&o'%) )%cr%%. T %0 ' %r% or% as< ' %Co&r' or a r(' o mandamus 'o co +%- ' % r%s+o )% ' 'o (ss&% ' % sa() r&-%s.

ISSUEKS:

%' %r or o' R.A. No. ##57, .D. No. 27, roc. No. 1!1, a ) E.O. Nos. 228 a ) 22*ar% co s'('&'(o a-.

ELD:

R.A. No. ##57, S%c'(o 18 o ' % CAR La , .D. No. 27, roc. No. 1!1, a ) E.O. Nos.228 a ) 22* ar% co s'('&'(o a-.

T % Co&r' )%c-ar%) ' a' ' % co '% ' a ) a %r o ' % &s' co +% sa'(o +rov()%) or ( S%c'(o 18 o ' % CAR La (s o' v(o-a'(v% o ' % Co s'('&'(o .

E.O. No. 228, ca'% or(ca--0 s'a'%) ( ('s S%c'(o 1 ' a':

A-- 4&a-( (%) ar %r6/% % (c(ar(%s ar% o )%% %) &-- o %rs as o Oc'o/%r 21, 1*72 o

' % -a ) ' %0 ac4&(r%) /0 v(r'&% o r%s()% '(a- D%cr%% No. 27. =E + as(s s&++-(%).>(' as o/v(o&s-0 r% %rr( 'o -a )s a-r%a)0 va-()-0 ac4&(r%) & )%r ' % sa() )%cr%%, a '%r +roo o &--6 -%) %) % /%rs (+ ( ' % ar %rs coo+%ra'(v%s a ) &-- +a0 % ' o &s'co +% sa'(o . % c%, (' as a-so +%r %c'-0 +ro+%r or ' % Or)%r 'o a-so +rov()% ( ('sS%c'(o 2 ' a' ' % -%as% r% 'a-s +a() 'o ' % -a )o %r /0 ' % ar %r6/% % (c(ar0 a '%r Oc'o/%r 21, 1*72 =+% )( 'ra s %r o o %rs (+ a '%r &-- +a0 % ' o &s'co +% sa'(o >, s a-- /% co s()%r%) as a)va c% +a0 % ' or ' % -a ).

Page 172: Nat Res Digested

8/10/2019 Nat Res Digested

http://slidepdf.com/reader/full/nat-res-digested 172/173

T % CAR La , or ('s +ar', co )('(o s ' % 'ra s %r o +oss%ss(o a ) o %rs (+ o ' %-a ) 'o ' % ov%r % ' o r%c%(+' /0 ' % -a )o %r o ' % corr%s+o )( +a0 % ' or ' % )%+os(' /0 ' % DAR o ' % co +% sa'(o ( cas or L /o )s (' a acc%ss(/-%/a <. U '(- ' % , '('-% a-so r% a( s (' ' % -a )o %r. No o&'r( ' c a % o o %rs (+(s co '% +-a'%) %(' %r.

% c%, ' % ar & % ' ' a' ' % assa(-%) %as&r%s v(o-a'% )&% +roc%ss /0 ar/('rar(-0'ra s %rr( '('-% /% or% ' % -a ) (s &--0 +a() or &s' a-so /% r% %c'%).

T % ar & % ' o so % o ' % +%'('(o %rs ' a' roc. No. 1!1 a ) E.O. No. 22* s o&-) /%( va-()a'%) /%ca&s% ' %0 )o o' +rov()% or r%'% '(o -( ('s as r%4&(r%) /0 Ar'(c-% III,S%c'(o 9 o ' % Co s'('&'(o (s o -o %r '% a/-%. R.A. No. ##57 )o%s +rov()% or s&c-( ('s o ( S%c'(o # o ' % -a , (c ( ac' (s o % o ('s os' co 'rov%rs(a-+rov(s(o s. T (s s%c'(o )%c-ar%s:

R%'% '(o L( ('s. 6 E$c%+' as o' %r (s% +rov()%) ( ' (s Ac', o +%rso a0 o or

r%'a( , )(r%c'-0 or ( )(r%c'-0, a 0 +&/-(c or +r(va'% a r(c&-'&ra- -a ), ' % s(3% o (c s a--var0 accor)( 'o ac'ors ov%r ( a v(a/-% a (-06s(3%) ar , s&c as co o)('0+ro)&c%), '%rra( , ( ras'r&c'&r%, a ) so(- %r'(-('0 as )%'%r ( %) /0 ' % r%s()% '(a-

A rar(a R% or Co& c(- = ARC> cr%a'%) %r%& )%r, /&' ( o cas% s a-- r%'% '(o /0' % -a )o %r %$c%%) (v% =5> %c'ar%s. T r%% =!> %c'ar%s a0 /% a ar)%) 'o %acc (-) o ' % -a )o %r, s&/ %c' 'o ' % o--o ( 4&a-( (ca'(o s: =1> ' a' % (s a' -%as'( '%% =15> 0%ars o a % a ) =2> ' a' % (s ac'&a--0 '(--( ' % -a ) or )(r%c'-0 a a (' % ar rov()%), T a' -a )o %rs os% -a )s av% /%% cov%r%) /0 r%s()% '(a-D%cr%% No. 27 s a-- /% a--o %) 'o <%%+ ' % ar%a or( ( a--0 r%'a( %) /0 ' % ' %r%& )%r,&r' %r, T a' or( ( a- o %s'%a) ra '%%s or )(r%c' co +&-sor0 %(rs o s'(-- o ' %or( ( a- o %s'%a) a' ' % '( % o ' % a++rova- o ' (s Ac' s a-- r%'a( ' % sa % ar%as as

-o as ' %0 co '( &% 'o c&-'(va'% sa() o %s'%a).T % ar & % ' ' a' E.O. No. 22* v(o-a'%s ' % co s'('&'(o a- r%4&(r% % ' ' a' a /(-- s a--av% o -0 o % s&/ %c', 'o /% %$+r%ss%) ( ('s '('-%, )%s%rv%s o -0 s or' a''% '(o . I' (ss%''-%) ' a' ' % '('-% o ' % /(-- )o%s o' av% 'o /% a ca'a-o &% o ('s co '% 's a ) (--s& (c% ( ' % a''%rs % /o)(%) ( ' % '%$'.

Page 173: Nat Res Digested

8/10/2019 Nat Res Digested

http://slidepdf.com/reader/full/nat-res-digested 173/173

( - id., +. 2" 3a ian v. !esierto, s&+ra, +. 987.(( -d.((( See X 1, Ar'. III, Co s'('&'(o .(v 3a ian v. !esierto, supra , +. 98*.v - id. , +. 9*2 Metro Construction v. Chatham &roperties, supra , ++. 2262!.v( I's +r%c&rsors ar% C(rc&-ar No. 16*1, (c +r%scr(/%) ' % r&-%s ov%r ( a++%a-s 'o ' % CA ro ' % ( a-or)%rs or )%c(s(o o ' % Co&r' o Ta$ A++%a-s a ) 4&as(6 &)(c(a- a % c(%s a ) A) ( (s'ra'(v% C(rc&-ar No. 16*5,(c r%v(s%) ' % %ar-(%r c(rc&-ar.

v(( S%c'(o 1, R&-% 9! o ' % R&-%s o Co&r'.v((( Metro Construction v. Chatham &roperties, supra, +. 2".($ - id./ Prior to 2P 2lg# ' ?) the .urisdiction of the CA) under the Gudiciary Act of '?=@) ,as as follo,s6

SEC. 2*. 5urisdiction o the Court o *ppeals. 6 T % Co&r' o A++%a-s s a-- av% %$c-&s(v% a++%--a'% &r(s)(c'ov%r a-- cas%s, ac'(o s, a ) +roc%%)( s, o' % & %ra'%) ( s%c'(o s%v% '%% o ' (s Ac', +ro+%r-0 /ro& ' 'o (',

%$c%+' ( a- &) % 's or )%c(s(o s o Co&r' o F(rs' I s'a c% r% )%r%) a '%r 'r(a- o ' % %r('s ( ' % %$%rc(s% o a++%--a'% &r(s)(c'(o , (c a (r ( &-- ' % &) % ' or )%c(s(o o a & (c(+a- or c('0 co&r', ( (c cas%s ' %a r(%v%) +ar'0 a0 %-%va'% ' % a''%r 'o ' % Co&r' o A++%a-s o -0 o +%'('(o or r%v(% , 'o (c ' % Co&r' o

A++%a-s s a-- (v% )&% co&rs% o -0 % ' % +%'('(o s o s prima acie ' a' ' % co&r' as co (''%) %rrors o ac' or o ac' a ) -a ' a' o&-) arra ' r%v%rsa- or o)( (ca'(o o ' % &) % ' or )%c(s(o s so& ' 'o /%r%v(% %). T % )%c(s(o o ' % Co&r' o A++%a-s s a-- /% ( a-:&rovided, however, T a' ' % S&+r% % Co&r' ( ('s